mksap13-hematology and oncology

329
Hematology and Oncology:Question 1 A 67-year-old man in previously good health is hospitalized because of a 2- day history of fever and diminished consciousness. The patient responds inconsistently to verbal commands. His temperature is 39.5 °C (103.1 °F); he has tachycardia, and his blood pressure is 80/58 mm Hg. There is no bleeding. His hemoglobin is 12.1 g/dL, leukocyte count is 29,000/μL with 80% neutrophils, and platelet count is 20,000/μL. Which of the following studies should be obtained in this patient? A. Bone marrow aspiration and biopsy B. Factor VIII level C. Measurement of platelet-associated IgG D. Measurements of fibrin D-dimer and total fibrinogen E. Bleeding time Hematology and Oncology:Question 2 A 35-year-old woman has completed 6 months of anticoagulation therapy for pulmonary emboli. She had been using

Upload: iman

Post on 27-Apr-2015

1.301 views

Category:

Documents


3 download

DESCRIPTION

oncology

TRANSCRIPT

Page 1: MKSAP13-Hematology and Oncology

Hematology and Oncology:Question 1A 67-year-old man in previously good health is hospitalized because of a 2-day history of fever and diminished consciousness. The patient responds inconsistently to verbal commands. His temperature is 39.5 °C (103.1 °F); he has tachycardia, and his blood pressure is 80/58 mm Hg. There is no bleeding. His hemoglobin is 12.1 g/dL, leukocyte count is 29,000/μL with 80% neutrophils, and platelet count is 20,000/μL. Which of the following studies should be obtained in this patient? A. Bone marrow aspiration and biopsy B. Factor VIII level C. Measurement of platelet-associated IgG D. Measurements of fibrin D-dimer and total fibrinogen E. Bleeding time

Hematology and Oncology:Question 2 A 35-year-old woman has completed 6 months of anticoagulation therapy for pulmonary emboli. She had been using oral contraceptives for the past 10 years but stopped when the pulmonary emboli were detected. There is no family history of thrombosis. Two weeks after the discontinuation of warfarin, she undergoes coagulation studies. She is found to be heterozygous for the prothrom bin G2021 0A mutation. On the basis of her history and laboratory evaluation, what is the best

Page 2: MKSAP13-Hematology and Oncology

recommendation for this patient? A. Resume oral anticoagulation therapy at a target INR of 2 to 3 B. Resume oral anticoagulation at a target INR of 1 .5 to 2 and advise her that it is safe to begin taking oral contraceptives again C. Advise the patient to take 81 mg of aspirin each day D. Prescribe a combination of aspirin and dipyridamole E. Advise the patient not to resume oral contraceptives and that additional therapy is not required for the thrombotic event

Hematology and Oncology:Question 3 A 22-year-old man with sickle cell disease has four to six painful crises per year that require parenteral hydration and narcotics. Two weeks ago the patient developed an acute chest characterized by fever, dyspnea, and the radiographic appearance of pulmonary infiltrates. The patient’s condition rapidly responded to aggressive exchange transfusions and empiric broad-spectrum antibiotics; he has been asymptomatic for 1 week. Laboratory studies: Hemoglobin 10.4 g/dL Hematocrit 32% Leukocyte count 12,000/μL Platelet count 277,000/μL

Page 3: MKSAP13-Hematology and Oncology

Hemoglobin electrophoresis shows the following results. Hemoglobin A (α2 β2) 62% (normal > 97.5%) Hemoglobin A2(α2 δ2) 2% (normal< 2.5%) Hemoglobin F (α2γ2) 1% (normal 0%) Hemoglobin S (α2 βS2) 35% (normal 0%) What is the best next step in this patient’s management? A. Long-term transfusion therapy B. Hydroxyurea therapy C. Consultation for therapeutic gene therapy D. Consultation for therapeutic bone marrow transplantation E. Long-term oxygen therapy

Hematology and Oncology:Question 4 A 24-year-old woman is evaluated in the emergency department because of fever, chills, sore throat, and difficulty talking. Two months ago she was diagnosed with schizophrenia, and clozapine therapy was initiated, at which time her complete blood count was normal. On physical examination, her tonsils are swollen and red bilaterally; she has submandibular lymphadenopathy and high fever. Her temperature is 40 °C (104 °F), pulse rate 140/min, and blood pressure is 90/50 mm Hg. Her hemoglobin is 12 g/dL, hematocrit is 36%, leukocyte count is 2000/μL, and platelet count

Page 4: MKSAP13-Hematology and Oncology

is 180,000/μL. Throat and blood cultures are obtained; broad-spectrum intravenous antibiotic therapy is initiated and clozapine is discontinued. Which of the following is the most appropriate next step for this patient? A. Bone marrow aspiration and biopsy B. Initiation of subcutaneous granulocyte colony-stimulating factor daily C. Granulocyte transfusion D. Search for a suitable bone marrow donor E. Test for HIV

Hematology and Oncology:Question 5 A 46-year-old woman with alcoholism is evaluated because of a 4-day history of nausea and vomiting after a recent binge. She has diffuse epigastric pain. She has been hospitalized intermittently for years for medical complications related to alcohol abuse, including cirrhosis with ascites and bleeding esophageal varices. On physical examination, her temperature is 37.3 °C (99.1 °F), pulse rate is 68/min, and blood pressure is 135/70 mm Hg. She has jaundice and is icteric. Her lungs are clear to auscultation. The remainder of her physical examination is unremarkable. Laboratory studies: Hemoglobin 7.1 g/dL Leukocyte count 4500/μL

Page 5: MKSAP13-Hematology and Oncology

Platelet count 115,000/μL Mean corpuscular volume 107 fL Reticulocyte count 5.4% Serum alkaline phosphatase 592 U/L Serum aspartate aminotransferase 198 U/L Serum alanine aminotransferase 166 U/L Serum total bilirubin 9.2 mg/dL Serum indirect bilirubin6.4 mg/dL Serum albumin 2.2 g/dL Serum folate 12 ng/mL Which of the following diagnoses best explains the findings? A. Spur cell anemia B. Folic acid deficiency C. Hypersplenism D. Acanthocytosis

Hematology and Oncology:Question 6 A 62-year-old man is evaluated because of increasing fatigability. He previously felt fine. He has recently been found to have osteomyelitis underlying a diabetic foot ulcer and is being treated with ciprofloxacin and amoxicillin/clavulanate.

Laboratory studies: Hemoglobin 9.8 g/dL Mean corpuscular volume 79 fL Serum creatinine0.9 mg/dL Serum iron 38 μg/dL

Page 6: MKSAP13-Hematology and Oncology

Serum total iron-binding capacity 190 μg/dL Serum ferritin 298 ng/mL Which of the following constitutes appropriate additional therapy? A. Cyanocobalamin B. Ferrous sulfate C. Parenteral iron D. Recombinant human erythropoietin and ferrous sulfate

Hematology and Oncology:Question 7 A previously healthy, 59-kg (130-Ib), 35-year-old woman delivers her fifth child after an uncomplicated pregnancy and labor. Her blood type is group O, Rh-positive. Soon after the birth, vaginal bleeding is noted that requires emergency transfusion of four units of packed red blood cells. Results of an antibody screen and direct antiglobulin test were negative at 2 months gestation and again when the patient was hospitalized for the delivery. Seven days later, faint scleral icterus is noted. The hemoglobin is 9.5 g/dL (approximately 1 g/dL less than at discharge after delivery). Serum total bilirubin is 2.1 mg/dL, with a direct component of 0.3 mg/dL. Serum lactate dehydrogenase level is 200 U/L, and haptoglobin is 20 mg/dL. Direct antiglobulin (Coombs) testing shows IgG coating the erythrocytes. There is no hemoglobinemia, and the urine is negative for blood.

Page 7: MKSAP13-Hematology and Oncology

What is the most likely cause of hemolysis in this patient? A. Improper storage of the units of blood prior to transfusion B. Autoimmune hemolytic anemia C. Glucose-6-phosphate dehydrogenase deficiency D. Erroneous transfusion of group A blood E. Hemolytic transfusion reaction

Hematology and Oncology:Question 8 A 60-year-old woman is evaluated because of a 1-year history of fatigue. Her menses ended when she was 50 years old, and she has not had any blood loss since that time. She also notes that she is short of breath with exertion. She recalls that a bone marrow biopsy was done some years ago. She is not sure about her family history because her father died when she was young and her mother died of “liver complications at an early age. On physical examination, she has a bronze hue to her skin. The liver is enlarged 3 cm below the right costal margin. She has swelling of the first and second metacarpophalangeal joints of both hands. Laboratory studies: Hemoglobin 13.0 g/dL Hematocrit 40% Serum ferritin 1900 ng/mL Transferrin saturation 70%

Page 8: MKSAP13-Hematology and Oncology

Fasting plasma glucose 115 mg/dL Serum alanine aminotransferase 80 U/L Serum aspartate aminotransferase 35 U/L A copy of the bone marrow study that she mentioned reveals only trace macrophage iron stores. A polymerase chain reaction test of mutations in the HFE gene is performed on the basis of the clinical picture, and the patient is found to be a homozygote for the C282Y mutation. Which of the following is the next best step in this patients management? A. Liver biopsy B. Phlebotomy C. Follow-up of liver function and iron status D. Liver biopsy and phlebotomy

Hematology and Oncology:Question 9 A 37-year-old man is hospitalized because of fever and right-sided chest pain. He has been having fatigue and recently developed dyspnea on exertion and intermittent chills. At the age of 29 years he was diagnosed with stage III Hodgkin’s disease and treated with multiagent chemotherapy and radiation therapy. A year ago, he developed mild anemia with no obvious cause. He takes thyroid hormone replacement when he remembers and has used fexofenadine as needed for allergic rhinitis for the past 5 years. On physical examination, his temperature is

Page 9: MKSAP13-Hematology and Oncology

38.7 °C (101.7 °F), pulse rate is 112/min, and blood pressure is 110/70 mmHg. There is dullness at the right lower lung and egophony. The hemoglobin is 8.5 g/dL, hematocrit is 26 %, leukocyte count is 2200/μL, and platelet count is 70,000/μL. What is the most likely diagnosis? A. Relapsed Hodgkin’s disease B. Sepsis C. Hypothyroidism D. Secondary myelodysplastic syndrome E. Drug-induced bone marrow suppression

Hematology and Oncology:Question 10 A 43-year-old man with severe acquired aplastic anemia has not responded to immunosuppressive agents. He remains neutropenic and transfusion-dependent for platelets and red cells. He has an HLA-identical brother who has been cleared as a donor for his planned allogeneic stem cell transplant. They are both cytomegalovirus-seronegative. Which of the following would be prevented by using irradiated cellular blood products for this patient? A. Cytomegalovirus disease B. Alloimmunization C. Transfusion-related graft-versus-host disease D. Febrile nonhemolytic transfusion reaction

Page 10: MKSAP13-Hematology and Oncology

E. Hemolytic transfusion reaction

Hematology and Oncology:Question 11 A 40-year-old Chinese woman is evaluated because of pulmonary emboli that developed 1 week after she arrived in the United States after a 16-hour airplane trip. She has no other medical problems. She had taken oral contraceptives for several years and had one uncomplicated pregnancy but is not taking any medications at present. Her older brother and father both have a history of thrombosis. Results of a complete blood count, prothrombin time, and activated partial thromboplastin time are normal. Which of the following studies should be done at this time? A. Factor V Leiden and prothrombin G20210A mutation analyses B. Protein C, protein 5, and antithrombin III measurements C. Testing for a lupus anticoagulant D. Screening tests for cancer including CT scans of the chest, abdomen, and pelvis E. No testing is required

Hematology and Oncology:Question 12 A 34-year-old woman is evaluated because of progressive fatigue and recurrent attacks of abdominal pain. She weighs 61.2 kg (135 Ib). Her pulse rate is 110/min and her blood

Page 11: MKSAP13-Hematology and Oncology

pressure is 110/70 mm Hg. She is pale, and her spleen is palpable 4 cm below the costal margin. Laboratory studies: Hemoglobin 6 g/dL Hematocrit 20% Leukocyte count 2500/μL Platelet count 80,000/μL Reticulocyte count 10% Blood smear Anisocytosis and polychromatophilia Haptoglobin 0 mg/dL Serum lactate dehydrogenase 645 U/L There is high level of hemosiderin in the urine. What is the most likely cause of this patients pancytopenia? A. Autoimmune hemolytic anemia B. Aplastic anemia C. Myelodysplastic syndrome D. Paroxysmal nocturnal hemoglobinuria E. Acute myeloid leukemia

Hematology and Oncology:Question 13 A 45-year-old man with alcoholic cirrhosis is evaluated because of hematemesis from active bleeding due to esophageal varices. He receives eight units of uncrossmatched group O packed cells over 3 hours, but his blood pressure remains unstable and his bleeding continues.

Page 12: MKSAP13-Hematology and Oncology

When he was first evaluated in the emergency department, his prothrombin time and partial thromboplastin time were 34 s and 57 s, respectively. The blood bank never received a specimen for type and screen, and no pretransfusion blood is available for testing. What is the most appropriate transfusion product to treat this patients bleeding? A. Group O whole blood B. Group AB whole blood C. Group O fresh frozen plasma D. Group AB fresh frozen plasma

Hematology and Oncology:Question 14 A 24-year-old man who moved to this country from Sweden 15 years ago is evaluated because of severe weakness and shortness of breath. He has a 4-year history of HIV infection and has had intermittent infections, predominantly sinusitis, over the past year. Two months ago, his hemoglobin was 10.8 g/dL, leukocyte count was 3900/μL, and platelet count was 220,000/μL. He has been noncompliant with his antiretoviral medications because of the side effects, but admits to taking trimethoprim-sulfamethoxazole twice daily for the past week. His appetite is poor. Cn physical examination, he exhibits extreme pallor. His temperature is 37.5 °C (99.5 °F), his pulse rate is 110/min, and his blood pressure is

Page 13: MKSAP13-Hematology and Oncology

130/89 mm Hg. His respiratory rate is 22/min at rest, increasing to 33/min when walking. He has no skin lesions. His conjunctivae are pale. He has generalized shotty lymphadenopathy. Chest radiography is normal. The remainder of his physical examination is unremarkable. Laboratory studies: Hemoglobin 4 g/dL Leukocyte count 2400/μL Platelet count 129,000/μL Mean corpuscular volume 101 fL Reticulocyte count 0% Peripheral blood smear No abnormalities Serum total bilirubin 3.2 mg/dL Serum indirect bilirubin2.8 mgldL Serum vitamin B12 222 pg/mL Serum folate 14 ng/mL What is the most likely diagnosis? A. Drug-induced marrow suppression B. Parvoviral infection C. HIV-induced marrow dysplasia D. Autoimmune hemolytic anemia

Hematology and Oncology:Question 15 A 24-year-old black woman is evaluated at an urgent care clinic because of sore throat, temperature of 39.4 °C (103 °F), chills, and severe fatigue of several weeks duration. She does not take any prescription or over-the-counter medications and she has never been ill. She lives with her two sisters.

Page 14: MKSAP13-Hematology and Oncology

On physical examination she has pallor and swollen and red tonsils bilaterally. Her pulse rate is 120/min, and her blood pressure is 90/50 mm Hg. Her hemoglobin is 6 g/dL, leukocyte count 100/μL, platelet count 17,000/μL, and reticulocyte count 0%. After throat and blood cultures are obtained, broad-spectrum intravenous antibiotic therapy is administered, and the patient receives a transfusion of cytomegalovirus-negative irradiated red blood cells. Bone marrow aspirate and biopsy show scattered plasma cells and almost complete absence of myeloid elements. What is the most appropriate therapy for long-term survival of this patient? A. Intravenous immunoglobulin, 500 mg/kg daily for 4 days B. Allogeneic stem cell transplantation from an HLA-identical sister C. Oral melphalan and prednisone D. Antithymocyte globulin and cyclosporine E. Long-term transfusion and iron chelation therapy

Hematology and Oncology:Question 16 A 63.5-kg (140-Ib) 29-year-old woman underwent repeat thoracic spinal fusion surgery. She had extensive bleeding and received a total of 20 units of packed red blood cells. The surgery lasted 3 hours and required

Page 15: MKSAP13-Hematology and Oncology

the use of 7 L of Ringers lactated solution. As they started to close the incision, the surgeons noticed diffuse bleeding from the surgical site that was not easily controlled by cauterization. An emergency complete blood count showed a hemoglobin of 11.7 g/dL, hematocrit of 35%, a leukocyte count of 6300/μL, and a platelet count of 65,000/μL. What is the most appropriate choice of blood product to stop this patient’s bleeding? A. Platelet concentrates B. Whole blood C. Fresh frozen plasma D. Cryoprecipitate

Hematology and Oncology:Question 17 A 54-year-old woman is evaluated because of progressively worsening fatigue and inability to sustain her work hours. Metastatic breast cancer was diagnosed 6 months ago, and she is currently receiving multiagent systemic chemotherapy. On physical examination, she is afebrile. Her blood pressure is 120/80 mm Hg. She has total alopecia and a left mastectomy scar. Laboratory findings include a leukocyte count of 3200/μL, hemoglobin of 8.5 g/dL, hematocrit of 26%, platelet count of 142,000/μL, and serum ferritin level of 375 mg/dL.

Page 16: MKSAP13-Hematology and Oncology

Which of the following is the most appropriate management of this patient? A. Daily administration of granulocyte colony-stimulating factor subcutaneously B. Cessation of chemotherapy C. Weekly adminstration of erythopoietin subcutaneously D. Administration of androgens E. Administration of vitamin E

Hematology and Oncology:Question 18 A 46-year-old woman is referred for preoperative evaluation before undergoing resection of a newly diagnosed glioblastoma. Her hemoglobin is 12.5 g/dL, leukocyte count is 6700/μL, and platelet count is 193,000/μL. The surgeon is particularly interested in an opinion concerning her risk of bleeding. Which of the following will provide the best estimate of her surgical risk of bleeding? A. Bleeding time B. Platelet Function Analyzer-i 00 analysis C. Platelet aggregation studies D. Medical history, including outcomes of previous surgical procedures E. Prothrombin time and partial thromboplastin time

Hematology and Oncology:Question 19 A 42-year-old woman with rheumatoid arthritis is found be anemic, with a hemoglobin of 11.0

Page 17: MKSAP13-Hematology and Oncology

g/dL. Her serum iron level is 21μg/dL, and iron replacement with ferrous sulfate, 325 mg three times daily, is started. On a return visit 3 months later, the patient’s hemoglobin level remains unchanged. The patient claims to be taking ferrous sulfate as directed and reports mild epigastric discomfort and constipation since starting. On rectal examination, stool is black but negative for occult blood. There has been no change in the patient’s menstrual blood loss, which has typically been light. Which of the following is the next appropriate step in this patient’s management? A. Change to an iron polysaccharide complex B. Order a serum ferritin test C. Add ascorbic acid D. Administer parenteral iron

Hematology and Oncology:Question 20 A 67-year-old woman is evaluated because of a 3-month history of dry cough and progressive fevers. She denies having chills but notes intermittent drenching night sweats. She was treated 6 months ago with corticosteroids for polymyalgia rheumatica. She currently takes prednisone, 5 mg every other day, with reasonable symptom control. She smoked two packs of cigarettes daily for 20 years. Her family history is unremarkable, except that her husband was treated for tuberculosis 20 years

Page 18: MKSAP13-Hematology and Oncology

ago. On physical examination, her temperature is 38.6 °C (101 .5 °F), pulse rate is 86/min, and blood pressure is 132/80 mm Hg. She is anicteric. Abdominal examination reveals a palpable spleen tip. The remainder of her physical examination is unremarkable. Radiograph of the chest shows a diffuse micronodular pattern confirmed on CT scan of the chest. Abdominal CT scan confirms mild splenomegaly and shows a 1.5-cm periportal lymph node. Blood cultures (including fungal culture), urine cultures, and sputum cultures (including stains for acid-fast bacteria) show no growth. Bronchoscopy with bronchoalveolar lavage is nondiagnostic for infectious or neoplastic disorders. Laboratory studies: Hemoglobin 10.1 g/dL Leukocyte count 12,400/μL Platelet count 625,000/μL Mean corpuscular volume 89 fL Reticulocyte count 1.3% Differential 62% neutrophils, 10% bands, 3% metamyelocytes, 15% lymphocytes, 6% eosinophils, 2% nucleated erythrocytes, 2% monocytes Serum alkaline phosphatase 360 U/L Serum aspartate aminotransferase 112 U/L Serum alanine aminotransferase 178 U/L Serum total bilirubin 1.1 mg/dL Serum indirect bilirubin0.7 mg/dL

Page 19: MKSAP13-Hematology and Oncology

Serum albumin 3.2 g/dL Skin tests for tuberculosis and Trichophyton are negative; she is anergic. What would be the most appropriate next step in the management of this patient? A. Liver biopsy B. Bone marrow biopsy C. Splenectomy D. Abdominal (periportal) lymph node aspiration

Hematology and Oncology:Question 21 A 32-year-old woman with sickle cell disease is evaluated in the emergency department because of diffuse bone and joint pains typical of sickle cell crisis. In the past week, she has noted generalized malaise, a low-grade fever, and mild exertional dyspnea without cough. The patient states that her last painful crisis occurred more than 6 months ago and was treated with rehydration, intravenous narcotic analgesics, and two units of packed red blood cells. On physical examination, the patient appears uncomfortable. Her temperature is 38.4 °C (101.2 °F), pulse rate 108/min, respiratory rate 20/min, blood pressure 126/92 mm Hg. She is mildly icteric. Bibasilar crackles are audible on pulmonary examination. Her abdomen is soft and nontender; there is no palpable organomegaly. Transcutaneous pulse oximetry

Page 20: MKSAP13-Hematology and Oncology

with the patient breathing room air indicates 95% arterial oxygen saturation. Results of chest radiography are unremarkable. Laboratory studies: Hemoglobin 4.2 g/dL Hematocrit 11 .8% Mean corpuscular volume 88 fL Mean corpuscular hemoglobin 22 pg Mean corpuscular hemoglobin concentration 34 g/dL Leukocyte count 1 2,000/μL Platelet count 200,000/μL Reticulocyte count 0.2% What is the most likely cause of the patients current status? A. Parvovirus B19 infection B. Alloantibodies against transfused erythrocytes C. Folate deficiency D. Splenic sequestration

Hematology and Oncology:Question 22 A 67-year-old man who had peripheral vascular surgery 2 months ago is hospitalized because of substernal chest pain and electrocardiographic changes consistent with myocardial ischemia. His hemoglobin is 12.2 g/dL, leukocyte count is 7300/μL, and platelet count is 235,000/μL. An infusion of unfractionated heparin is initiated, but pain persists intermittently. Two days after admission, his repeat platelet count is

Page 21: MKSAP13-Hematology and Oncology

90,000/μL. What is the most appropriate management for this patient? A. Continue the heparin infusion, monitor the platelet count daily B. Continue the heparin infusion and order an ELISA test for heparin-platelet factor IV antibodies. C. Discontinue the heparin infusion D. Add an antiplatelet drug E. Discontinue the heparin infusion and institute thrombin-inhibitor therapy

Hematology and Oncology:Question 23 Four days ago, a 30-year-old woman with ulcerative colitis required transfusion with packed red cells for recurrent gastrointestinal bleeding during a flare of her disease. The blood bank distributed group O, Rh-negative units for her because they were the only antigen-negative units available. The antibody screen performed in the emergency department did not show any additional antibodies. This patient has received many red cell transfusions since her diagnosis 10 years ago. Her blood type is group A, and she is Rh-positive. She has a history of positive response to a red cell antibody screen, with anti-Kell, anti-Duffy(a), and anti-E identified. On the day after her present transfusion, the

Page 22: MKSAP13-Hematology and Oncology

patients hemoglobin level increases by 3 g/dL. The bleeding subsides after high-dose corticosteroid therapy but starts again as the dose is lowered. The patients blood pressure is stable, but her physician requests that additional units be prepared for her in case the bleeding becomes more active. The blood bank refuses the request until a new compatibility sample is sent. The new compatibility sample is needed to detect which of the following? A. New alloantibodies B. A hemolytic transfusion reaction C. Residual group O red cells D. New autoantibodies E. Antibodies missed in the first sample

Hematology and Oncology:Question 24 A 55-year-old man who is HIV-positive is evaluated because of weakness, shortness of breath, and a dry, nonproductive cough. He also reports dizziness, cold intolerance, and an inability to continue his regular daily jogging routine. Physical examination reveals pallor, a flow murmur, and bilateral crackles in the lungs. Laboratory studies: Hemoglobin 7 g/dL Mean corpuscular volume 75 fL Red cell distribution width 17 Reticulocyte count 0.8 %

Page 23: MKSAP13-Hematology and Oncology

Leukocyte count 8700/μL Serum ferritin 180 ng/mL Serum transferrin 180 mg/dL Serum iron 10 μg/dL A chest radiograph shows an interstitial infiltrate consistent withPneumocystis carinii pneumonia. Which of the following is the most appropriate approach to the anemia? A. Assume the patient has the anemia of chronic disease; direct diagnosis and treatment to the underlying inflammatory process B. Initiate treatment with ferrous sulfate and observe the patients response over the next several weeks C. Treat the patient with erythropoietin D. Perform a bone marrow aspirate and stain with iron stain; if negative, evaluate the gastrointestinal tract for evidence of bleeding

Hematology and Oncology:Question 25 In an office visit for an annual checkup, a 46-year-old man reports that he has had malaise and intermittent sweats for the past few months but has been able to continue his job as a high school teacher. Two years ago he was treated for stage III diffuse large-cell non-Hodgkin’s lymphoma with six cycles of cyclophosphamide, doxorubicin, vincristine, and prednisone (CHOP) and attained complete remission. He takes simvastatin for

Page 24: MKSAP13-Hematology and Oncology

hypercholesterolemia and hydrochlorothiazide for hypertension. On physical examination, he has lymphadenopathy: a 3-cm right axillary node and a 2-cm right supraclavicular node. His spleen tip is palpable. Laboratory evaluation shows mild normochromic, normocytic anemia and an elevated serum lactate dehydrogenase level. CT scans of his chest and abdomen reveal additional mediastinal and retroperitoneal lymphadenopathy. What is the best next step in this patient’s management? A. A 2-week course of oral antibiotics B. Referral for salvage chemotherapy and autologous stem cell transplantation C. A repeat physical examination and CT scans in 3 months D. Referral for treatment with investigational agents E. A repeat course of CHOP

Hematology and Oncology:Question 26 A 50-year-old woman requests hormone replacement therapy (HRT) for severe hot flashes. Family studies done after her daughter developed deep venous thrombosis in association with use of oral contraceptives showed the patient to be heterozygous for the factor V Leiden mutation. Subsequently, several other members of her large extended

Page 25: MKSAP13-Hematology and Oncology

family were found to have the factor V Leiden mutation, but none has yet had a thrombotic event, including several older than 65 years. The patient is healthy, and results of a physical examination are within normal limits. Which of the following regimens is most appropriate for management of this patient? A. Initiate HRT along with warfarin, with a target INR of 1.5 to 2 B. Initiate HRT along with enoxaparin and warfarin, with a target INR of 2 to 3 C. Initiate HRT along with warfarin, with a target INR of 2 to 3 D. Recommend against the use of HRT; initiate anticoagulant prophylaxis at times of increased thrombotic risk E. Recommend against the use of HRT and prescribe aspirin therapy

Hematology and Oncology:Question 27 A 37-year-old woman is referred for evaluation of the new onset of thrombocytopenia. She underwent orthotopic cardiac transplantation 4 months ago following the development of postpartum cardiomyopathy. Function of the allograft has been good with only one episode of mild rejection several weeks after transplant that was successfully managed by changing her immunosuppressive regimen. Her current medications include cyclosporine, tacrolimus, trimethoprim-sulfamethoxazole, and

Page 26: MKSAP13-Hematology and Oncology

furosemide. Physical examination shows no petechiae, purpura, or splenomegaly. Laboratory studies reveal a hemoglobin of 11.9 g/dL, leukocyte count of 5000/μL, and platelet count of 30,000/μL. Peripheral blood smear shows reduced platelets but is otherwise normal. What is the optimal management of this patient? A. Discontinuation of trimethoprim-sulfamethoxazole B. Bone marrow aspiration and biopsy C. Platelet transfusion D. Administration of epsilon-aminocaproic acid E. Discontinuation of immunosuppressive drugs

Hematology and Oncology:Question 28 A 25-year-old pregnant woman is evaluated at the end of her first trimester. Three years ago, she sustained deep venous thrombosis while taking an oral contraceptive preparation. Her medical history is otherwise unremarkable and she is not currently taking anticoagulants or other medications. She recently underwent screening for thrombophilia, and no abnormalities were identified. There is no family history of venous thrombosis. Which of the following is the best management recommendation? A. Monitor her pregnancy and initiate low-

Page 27: MKSAP13-Hematology and Oncology

molecular-weight heparin at prophylactic doses after delivery and continue for 6 weeks B. Immediately initiate prophylactic doses of low-molecular-weight heparin and continue prophylaxis for 6 weeks after delivery C. Immediately initiate therapeutic doses of low-molecular-weight heparin and continue therapy for 6 weeks after delivery D. Terminate the pregnancy because of the high risk of thrombosis and obstetric complications E. Do not administer anticoagulation before or after delivery unless she develops symptomatic venous thromboembolism

Hematology and Oncology:Question 29 A 27-year-old woman is found to have a platelet count of 50,000/μL. Her medical history is unremarkable; she has not had excessive bruising, epistaxis, or menorrhagia. The hemoglobin level is 13.2 g/dL and the leukocyte count is 50004iL. The physical examination is within normal limits. A peripheral blood smear shows decreased platelets, but no other abnormalities. What is the most appropriate management for this patient? A. Bone marrow aspiration and biopsy B. Platelet aggregation studies C. Measurement of the platelet count of her parents and siblings

Page 28: MKSAP13-Hematology and Oncology

D. Prednisone, 1 mg/kg per day E. No intervention, follow-up with monthly platelet counts

Hematology and Oncology:Question 30 A 61-year-old man is evaluated because of progressive fatigue, dyspnea on exertion, and headaches over the course of the past 6 months. He also notices intense itching after he takes a shower. He had successful right hip replacement surgery 3 years ago for severe degenerative arthritis. He takes a cholesterol-lowering agent for hypercholesterolemia. He does not smoke but drinks one or two shots of whiskey several days a week. On physical examination, his pulse rate is 84/min, and his blood pressure is 145/85 mm Hg. His face appears swollen and flushed. Examination of the chest reveals no abnormalities. The spleen is palpable 3 cm below the costal margin. He has no edema. Hematocrit is 62%, leukocyte count 17,200/μL, and platelet count 544,000/μL. Arterial blood gas results are normal, with an oxygen saturation of 97%. His bone marrow is hypercellular, but the karyotype does not reveal the presence of the Philadelphia chromosome. What is the best next step in this patient’s management? A. Splenectomy B. Phlebotomy

Page 29: MKSAP13-Hematology and Oncology

C. Cytarabine and idarubicin induction chemotherapy D. Discontinuation of alcohol intake

Hematology and Oncology:Question 31 A 29-year-old woman is evaluated because of thrombocytopenia. She is currently in her 31st week of an otherwise uncomplicated pregnancy. Her platelet count was normal until approximately week 26, after which it declined gradually. She has had no epistaxis or other bleeding complications and no personal or family history of hematologic disease. One previous pregnancy resulted in term delivery of a healthy child. Current medications include prenatal vitamins. On physical examination, her blood pressure is 120/80mm Hg. Her uterus is appropriately enlarged for gestational age. Laboratory studies: Hemoglobin 12.1 g/dL Leukocyte count 8200/μL Platelet count 93,000 /μLA peripheral blood smear shows only decreased platelets. Which of the following is the most likely cause of this patients thrombocytopenia? A. Disseminated intravascular coagulation B. Idiopathic (immune) thrombocytopenic purpura C. Incidental (gestational) thrombocytopenia

Page 30: MKSAP13-Hematology and Oncology

D. Preeclampsia E. Drug-induced thrombocytopenia

Hematology and Oncology:Question 32 A 29-year-old woman with systemic lupus erythematosus and end-stage renal disease from glomerulonephritis is dialysis-dependent. She has significant arthritis and polyserositis, which are only partially helped by anti-inflammatory therapy. She has anemia, and has been receiving weekly erythropoietin and intravenous iron for 2 months, without much response. Her physician wishes to order a transfusion to treat her increasing fatigue. She has never received any kind of blood product. She is cytomegalovirus-seropositive and has never been pregnant. There are no family members who are HLA matched as potential donors, and she is on the waiting list for a cadaveric kidney transplant. Which of the following would be prevented by using leukocyte-reduced blood for this patient? A. Cytomegalovirus disease B. HLA alloimmunization C. Transfusion graft-versus-host disease D. Febrile nonhemolytic reaction E. Hemolytic transfusion reaction

Hematology and Oncology:Question 33 A 55-year-old black American woman prepares

Page 31: MKSAP13-Hematology and Oncology

to tour South Africa and visit relatives. She begins malaria prophylaxis with primaquine shortly before she is scheduled to leave. Over the next week she develops weakness and notes dark urine. Two days later, she feels short of breath and goes to the emergency department. Four years ago, she had a cardiac valve replaced for severe aortic stenosis and takes warfarin. Her medical history is otherwise unremarkable. Her parents were both raised in South Africa but have no known history of anemia or other blood dyscrasias. On physical examination, her temperature is 37.3 °C (99.1 °F), pulse rate is 11 5/min, blood pressure is 155/100 mm Hg. Results of head, eyes, ears, nose, and throat examination are unremarkable. She has slight icterus. Cardiac examination is unremarkable except for mechanical valve sounds. The remainder of her physical examination is normal. Laboratory studies: Hemoglobin 9.2 g/dL Leukocyte count 1 0,400/μL Platelet count 325,000/μL Mean corpuscular volume 101 fL Reticulocyte count 7.3% Serum total bilirubin 2.0 mg/dL Serum indirect bilirubin 1.6 mg/dL Serum lactate dehydrogenase 667 U/L

Page 32: MKSAP13-Hematology and Oncology

Differential count 60% neutrophils, 15% bands, 3% metamyelocytes, 15% lymphocytes, 3% eosinophils, 2% nucleated erythrocytes, 2% monocytes Urine is negative for hemoglobin and positive for hemosiderin. Oxygen saturation is 97% with the patient breathing room air. Peripheral blood smear shows Heinz bodies and bite cells.

Which of the following statements about this patients condition is true? A. Long-term continuation of primaquine would lead to severe, uncompensated hemolysis. B. The leukoerythroblastic blood smear suggests a marrow infiltrative process. C. The absence of free hemoglobin in the urine excludes recent intravascular hemolysis. D. Weekly dosing of primaquine is preferable to low-dose daily administration. E. The patients ethnic background indicates she has a severe enzyme deficiency.

Hematology and Oncology:Question 34 A previously healthy 65-year-old man is evaluated because of a 3-day history of swelling, warmth, and erythema in the left calf; compression ultrasound testing reveals deep venous thrombosis extending into the popliteal vein. The patient is a heavy smoker. There is no history of recent surgery, travel, or

Page 33: MKSAP13-Hematology and Oncology

immobilization. His mother had phlebitis in her 80s. Rectal examination is unremarkable and a stool specimen is negative for occult blood. Results of a complete blood count, prothrombin time, partial thromboplastin time, routine serum chemistry studies, and prostate-specific antigen are normal. No lupus anticoagulant is detected and chest radiograph is within normal limits. Which of the following is the next best step in the evaluation of this patient? A. No further evaluation is required B. CT scans of the chest, abdomen, and pelvis C. CT scan of the chest; upper and lower gastrointestinal endoscopy D. Fasting plasma homocysteine level E. Evaluation for a hereditary thrombotic disorder (tests for factor V Leiden and prothrombin G2021 0A mutations; measurements of levels of antithrombin III, protein C, and protein S)

Hematology and Oncology:Question 35 A 50-year-old, previous healthy man presents with a 3-day history of low-grade fever and confusion.

Page 34: MKSAP13-Hematology and Oncology

Physical examination demonstrates pallor and ecchymoses. Laboratory studies reveal a hemoglobin of 8.3 g/dL, leukocyte count of 9000/μL, and platelet count of 12,000/μL. The peripheral blood smear shows schistocytes and decreased platelets. Which additional studies are of most use in diagnosing this disorder? A. Functional assessment of plasma ADAMTS 13 (von Willebrand factor-cleaving protease) activity B. Measurement of antibodies against ADAMTS 13 C. Measurement of serum lactate dehydrogenase D. Liver function studies E. CT scan of the brain

Hematology and Oncology:Question 36 A 54-year-old man presents to the office for an annual checkup. He feels well and exercises regularly. He has gained 2.3 kg (5 Ib) since last year. His only complaint is bloating and fullness after meals. He takes hydrochlorothiazide for hypertension and aspirin, 81 mg/d. On physical examination, his temperature is 36.5 °C (97.7 °F), pulse rate is 84/min, and his blood pressure is 140/85mm Hg. The spleen tip is palpable below the costal margin, but there is no abdominal tenderness. He has no lymphadenopathy, abdominal mass, or carotid

Page 35: MKSAP13-Hematology and Oncology

or abdominal bruit. Peripheral pulses are palpable and normal. There is no occult blood in the stool. Laboratory studies: Hemoglobin 13 g/dL Hematocrit 40% Mean corpuscular volume 90 fL Leukocyte count 37,000/μL Platelet count 470,000/μL Differential Mostly segmented and band neutrophils and circulating metamyelocytes and myelocytes What is the diagnostic test of choice? A. CT scan of the abdomen B. Cytogenetic analysis of peripheral blood mononuclear cells C. Transesophageal echocardiography D. Leukocyte alkaline phosphatase score E. Blood culture

Hematology and Oncology:Question 37 A 67-year-old woman has an uncomplicated myocardial infarction. She returns to the hospital 2 weeks later for cardiac catheterization, which shows three-vessel coronary artery disease. Coronary artery bypass grafting is recommended, and the patient undergoes an uncomplicated procedure 2 weeks later. Her platelet count immediately before her surgery is 325,000/μL, and the day after is 210,000/μL. She is discharged 3 days

Page 36: MKSAP13-Hematology and Oncology

after surgery with the platelet count not having been repeated. Four days later she is evaluated in the emergency department because of shortness of breath. She is afebrile, and radiography of the chest shows no infiltrates. Platelet count at this time is 85,000/μL, prothrombin time and activated partial thromboplastin time are both normal, and peripheral blood smear shows only decreased numbers of platelets. A ventilation-perfusion lung scan is indeterminate, and low-molecular-weight heparin therapy is empirically initiated to treat a presumed pulmonary embolism. The following morning, the patient’s platelet count is 25,000/μL, and the patient has developed acute ischemia of her right foot. What is the most likely cause of her thrombocytopenia? A. Idiopathic thrombocytopenic purpura B. Seq uela of bypass surgery C. Disseminated intravascular coagulation D. Heparin-induced thrombocytopenia E. Thrombotic thrombocytopenic purpura

Hematology and Oncology:Question 38 An 18-year-old black man is discovered to have a mild microcytic hypochromic anemia during a routine physical examination. The patient is unaware of any significant relevant family history. Physical examination reveals a normal-appearing youth who is anicteric; the spleen is

Page 37: MKSAP13-Hematology and Oncology

not palpable. Laboratory studies: Hemoglobin 13 g/dL Hematocrit 32.4% Mean corpuscular volume 64 fL Mean corpuscular hemoglobin 19 pg Mean corpuscular hemoglobin concentration 30 g/dL Reticulocyte count 4.0% Hemoglobin electrophoresis shows the following results. Hemoglobin A (α2β2) 95% (normal > 97.5%) Hemoglobin A2(α2δ2) 5% (normal <2.5%) Hemoglobin F (α2γ2 )0% (normal 0%) Hemoglobin S (α21βS2) 0% (normal 0%) Iron studies are pending. What is the most likely cause of this patient’s anemia? A. Iron deficiency B. β-Thalassemia trait C. α-Thalassemia trait D. Glucose-6-phosphate dehydrogenase deficiency E. Sickle cell disease

Hematology and Oncology:Question 39 A 78-year-old woman underwent partial colonic resection for repair of a diverticular abscess. During her 2-week hospitalization, she received a twice-daily injection of unfractionated heparin. The platelet count was 21 5,000/μL on

Page 38: MKSAP13-Hematology and Oncology

admission and remained stable at discharge. Two weeks after discharge she is readmitted to the hospital with a platelet count of 27,000/μL and a painful, dusky left lower extremity. Hemoglobin is 12.1 g/dL and leukocyte count is 6500/μL. Arteriogram of the left lower extremity shows a thrombus in the left common femoral artery. Review of the peripheral blood smear shows only decreased numbers of platelets and no schistocytes. The prothrombin time and partial thromboplastin time are normal. Which study is most likely to yield the cause of this patient’s thrombosis? A. Genetic assay for the factor V Leiden mutation B. Genetic assay for the prothrombin G2021 0A mutation C. 1 4C-serotonin release assay for heparin-platelet factor 4 antibodies D. Platelet aggregation studies E. Platelet-associated IgG study

Hematology and Oncology:Question 40 What virus is associated with the highest risk of transmission through blood transfusion? A. Human T-cell lymphotropic virus B. Hepatitis C virus C. Human immunodeficiency virus

Page 39: MKSAP13-Hematology and Oncology

D. Hepatitis B virus

Hematology and Oncology:Question 41 A 52-year-old woman is evaluated because of transient numbness on the right side of her face and blurred vision lasting up to 24 hours. The patient has a history of migraine headache, but her current symptoms are distinct from those associated with her usual migraines. Physical examination is unremarkable except for livedo reticularis, which she has had for the past decade. Neurologic examination is within normal limits. Results of MRI of the head, transesophageal echocardiography, and carotid ultrasound examination are all within normal limits. Her complete blood count and serum chemistry studies are also normal. The prothrombin time is within normal limits, but the partial thromboplastin time is 55 s (normal limit as high as 34 s). Results of a lupus anticoagulant panel are positive. The cardiolipin IgG is elevated at 50 GPL U/mL (reference range <16 GPL U/mL); 1gM antibody level is normal. Which of the following is the best therapy for this patient? A. Aspirin, 325 mg/d B. Warfarin, with a target INR of 2 to 3, and aspirin, 81 mg/d C. Warfarin, with a target INR of 3 to 4

Page 40: MKSAP13-Hematology and Oncology

D. Aspirin, 325 mg/d, and clopidogrel E. Continue medications that she takes for her migraine headaches

Hematology and Oncology:Question 42 A previously healthy 38-year-old woman is evaluated because of a 2-week history of fatigue and malaise. She has not had any unexplained weight loss and says she does not have fevers or night sweats. She states that she does have occasional bilateral hand stiffness that responds to ibuprofen. Physical examination shows slight scleral icterus and a mildly enlarged spleen. There is no lymphadenopathy. Laboratory studies: Hemoglobin 9.2 g/dL Leukocyte count 8200/μL Platelet count 335,000/μL Reticulocyte count 9.8% A peripheral blood smear shows polychromatophilia, occasional spherocytes, and a rare nucleated erythrocyte. What is the best initial therapy for this patient?

A. Prednisone B. Intravenous immunoglobulin C. Danazol D. Splenectomy E. Erythropoietin

Page 41: MKSAP13-Hematology and Oncology

Hematology and Oncology:Question 43 A 62-year-old black woman is evaluated after a recent emergency department visit for a transient ischemic attack. She has intermittent dizziness and headaches but not vision changes, weakness, or numbness in her extremities. She has not seen a physician for more than 5 years and describes herself as generally healthy. She has not had any recent change in weight. On physical examination, her temperature is 36.4 °C (97.5 °F), her pulse rate is 84/min, and her blood pressure is 135/82 mm Hg. She has no carotid bruits or cardiac murmur, no splenomegaly, and no edema. Stool is negative for occult blood. Hemoglobin is 12.5 g/dL, leukocyte count 7200/1iL, platelet count 1,100,000/μL, and fasting blood glucose 165 mg/dL. Results of iron studies are normal. Results of electrocardiogram and chest radiography are normal. Her bone marrow is hypercellular; fibrosis is not evident and karyotype is normal. What is the most appropriate treatment of this patients disorder?

A. Hydroxyurea and warfarin B. Low-dose aspirin C. Hydroxyurea and low-dose aspirin

Page 42: MKSAP13-Hematology and Oncology

D. Busulfan and aspirin

Hematology and Oncology:Question 44 A 55-year-old retired longshoreman is evaluated because he has 3+ edema in both legs. Eight months ago, he had a deep venous thrombosis in his left iliac vein associated with a central venous catheter. He was treated with low-molecular-weight heparin followed by warfarin. The warfarin was stopped after 3 months because of difficulty regulating his INR and intermittent melena. He refused colonoscopy at that time. Two months ago, he had a recurrent thrombosis in the iliac vein, and low-molecular-weight heparin has been administered twice daily since then. Two days ago, recurrence of gastrointestinal bleeding was detected but has now stopped. He required a transfusion of four units of blood. His systolic blood pressure is now stable at 100 mm Hg. His prothrombin time is 17 s and partial thromboplastin time is 33 s. Serum albumin level is 1.9 g/dL. Colonoscopy reveals an ulcerating polyp, and surgical resection is planned. What is the most appropriate reason to order transfusion of fresh frozen plasma before this patient undergoes surgery? A. Liver disease B. Hypovolemia C. Hypoproteinemia

Page 43: MKSAP13-Hematology and Oncology

D. Prolonged prothrombin time E. Low-molecular-weight heparin therapy

Hematology and Oncology:Question 45 A 34-year-old woman is evaluated because of easy bruisability. She has previously been in good health with no personal or family history of hematologic disease. On physical examination, occasional petechiae are evident over the distal lower extremities. She has no hepatosplenomegaly. Her hemoglobin is 12.8 g/dL, leukocyte count is 6200/μL, and platelet count is 18,000/μL. Examination of the peripheral blood smear shows decreased platelets, with some platelets slightly increased in size. What is the most appropriate management for this patient? A. Observation alone, with monthly platelet counts B. Intravenous immunoglobulin C. Intravenous anti-Rh(D) D. Prednisone followed by immediate splenectomy E. Prednisone

Hematology and Oncology:Question 46 A 45-year-old man develops symptomatic deep venous thrombosis of the left leg 1 week after arthroscopic surgery. Doppler ultrasound examination shows a thrombus in the left posterior tibial vein extending to within 2 mm

Page 44: MKSAP13-Hematology and Oncology

of the popliteal vein. The patient has mild hypertension, for which he takes an angiotensin-converting enzyme inhibitor. He has no other medical problems and is a nonsmoker. There is no family history of thromboembolic disease. Which of the following treatments is the most appropriate? A. Ibuprofen, 600 mg every 8 h, and obtain a repeat ultrasound in 1 week B. Warfarin, 10 mg for 2 nights; check the patients INR the following day and adjust the dosage if necessary C. Enoxaparin, 1 mg/kg body weight subcutaneously every 12 hours, and warfarin, 5 mg/d, adjusted to an lNRof 2 to 3 D. Placement of an inferior vena cava filter E. Enoxaparin, 30mg subcutaneously every 12 hours, and warfarin, 5 mg/d, adjusted to an INR of 2 to 3.

Hematology and Oncology:Question 47 A 43-year-old previously healthy woman is evaluated because she develops a severe headache and becomes confused. She has had fatigue, lethargy, and dyspnea on exertion for 2 days. Her temperature is 37.7 °C (99.8 °F), pulse rate 93/min, and blood pressure 110/80 mm Hg. Her conjunctivae and mucous membranes are pale. Neurologic examination reveals a

Page 45: MKSAP13-Hematology and Oncology

confused and disoriented patient without focal findings. Laboratory studies: Hemoglobin 7.3 g/dL Hematocrit 23% Leukocyte count 6500/μL Platelet count 37,000/μL Reticulocyte count 3.5% (elevated; absolute count 107,000 cells/μL) Serum haptoglobin < 6 mg/dL Serum lactate dehydrogenase 2130 U/L Serum electrolytes Normal Serum creatinine 2.3 mg/dL Blood urea nitrogen 43 mg/dL Urinalysis 3+ blood, 1-3 erythrocytes, 0 leukocytes/hpf; no casts Results of a direct antiglobulin test are negative. The laboratory identifies polychromasia, numerous schistocytes (fragmented erythrocytes), and nucleated erythrocytes on a peripheral blood smear. Prothrombin time and partial thromboplastin time are normal. What is the best initial therapy for this patient?

A. Plasmapheresis with saline and albumin as replacement fluid B. Plasmapheresis with fresh frozen plasma as replacement fluid C. Platelet transfusion

Page 46: MKSAP13-Hematology and Oncology

D. High-dose corticosteroids

Hematology and Oncology:Question 48 A 57-year-old man is referred by his orthopedist for recommendations regarding anticoagulation therapy before knee replacement surgery next month. He had an unprovoked deep venous thrombosis 6 months ago. He takes no medications except warfarin. In addition to stopping the warfarin, which of the following anticoagulation regimens is most appropriate for this patient? A. Administer antithrombotic prophylaxis for 1 week after surgery B. Provide adequate antithrombotic prophylaxis after surgery and continue anticoagulation for a total of 6 weeks C. Hospitalize the patient for intravenous administration of heparin when his INR is less than 2.0; stop the heparin before surgery and administer anticoagulant prophylaxis for 1 week after surgery D. Begin therapeutic doses of low-molecular-weight heparin, and give a prophylactic dose the morning of surgery; resume therapeutic doses of low-molecular-weight heparin 12 hours after surgery E. Use pneumatic compression boots during and after surgery and start aspirin therapy, 325 mg/d postoperatively for 10 days

Page 47: MKSAP13-Hematology and Oncology

Hematology and Oncology:Question 49 A 69-year-old homeless man is hospitalized because of chest pain and electrocardiographic changes, and a diagnosis of acute myocardial infarction is considered. He also has fever and dyspnea, and chest radiograph shows lung infiltrates consistent with aspiration pneumonia. Broad-spectrum antibiotics are administered intravenously, but his respiratory status worsens and he requires intubation and ventilatory support. His hemoglobin is 9.7 g/dL and his hematocrit is 32%. His serum albumin is 2.2 mg/dL on admission. His left ventricular ejection fraction is 35% by echocardiogram, and his systolic blood pressure has ranged from 70 to 110 mm Hg. Arterial blood gas measurement shows a Po2 of 75 mm Hg, Pco2 of 35 mm Hg, and pH of 7.35. Oxygen saturation is 94% while breathing 50% inspired oxygen of 50%. His pulmonary capillary wedge pressure is below normal. What is the most appropriate reason to order packed red cell transfusion for this patient? A. Hem atocrit below 36% B. Ventilatory support C. Hypoxia D. Hypotension E. Hypovolemia

Page 48: MKSAP13-Hematology and Oncology

Hematology and Oncology:Question 50 A 44-year-old man is evaluated because he has had foamy, nonbloody diarrhea for 3 weeks and has lost 4.5 kg (10 Ib). He reports five to six bowel movements a day, including at night. His diarrhea is associated with crampy abdominal pain. He has a history of chronic myeloid leukemia, for which he underwent allogeneic stem cell transplantation 9 months ago. He is taking prednisone, 10 mg/d; loperamide, two to three times daily, and trimethoprim-sulfamethoxazole twice a week. On physical examination, he has generalized hyperpigmentation, and there are lichenoid changes over the skin of his arms, chest, and back. He has generalized muscle atrophy. Laboratory results show a hematocrit of 33%, leukocyte count of 45004iL, platelet count of 110,000/μL, and total serum bilirubin of 1.1 mg/dL. Stool culture shows no growth of enteric pathogens. What is the most likely cause of his diarrhea? A. Pseudomembranous colitis B. Laxative abuse C. Chronic graft-versus-host disease D. Recurrence of chronic myeloid leukemia E. Shigella enteritis

Hematology and Oncology:Question 51 A 40-year-old woman developed deep venous thrombosis and pulmonary embolism 1 month

Page 49: MKSAP13-Hematology and Oncology

post partum. She was hospitalized for 2 weeks before the delivery because of preeclampsia; she required a caesarean section. Warfarin therapy is begun after delivery. Her brother had an episode of venous thrombosis at the age of 33 years and was found to have protein S deficiency. Her father was also found to have a protein S deficiency but is in his 60s and has not had venous thromboembolism. After 6 months of anticoagulation, her warfarin therapy is discontinued and coagulation studies are done. Two weeks later, she too is found to have protein S deficiency. Which of the following is appropriate therapy for this patient? A. Resume oral anticoagulation therapy for an indefinite period at a target INR of 2 to 3 B. Administer low-molecular-weight heparin and resume anticoagulation with warfarin at a fixed dose of 1 mg/d for 2 years C. Discontinue oral anticoagulation and counsel her about prophylactic anticoagulation in high-risk situations D. Initiate treatment with aspirin and clopidogrel

Hematology and Oncology:Question 52 A 74-year-old woman with a history of diverticulosis and hypertension presents for an urgent office visit. She has had progressive

Page 50: MKSAP13-Hematology and Oncology

fatigue and poor appetite, and generally does not feel well. She has been taking ramipril, 5 mg/d, for the past year and aspirin, 81 mg/d, for more than 10 years. On physical examination, she is pale. There is no abdominal tenderness, splenomegaly, or lymphadenopathy. She is afebrile, and her pulse rate is 100/min. Laboratory studies: Hemoglobin 7.5 g/dL Leukocyte count 2200/μL Mean corpuscular volume 105 fL Reticulocyte count 0.8% Platelet count 87,000/μL Peripheral blood smear shows ovalomacrocytes and a few nucleated erythrocytes with nuclear budding. Neutrophils have reduced segmentation and granulation, and there is pseudo-Pelger-Huet hypolobulation. What is the most likely diagnosis? A. Iron deficiency anemia B. Drug-induced aplastic anemia C. Myelodysplastic syndrome D. Vitamin Bl2deficiency E. Myelofibrosis

Hematology and Oncology:Question 53 A 65-year-old man is hospitalized because of an episode of epistaxis that lasted longer than 1 hour and some rectal bleeding. He has myelodysplastic syndrome and low blood

Page 51: MKSAP13-Hematology and Oncology

counts, including a platelet count of 1 5,0004/μL and an absolute neutrophil count of 600/μL (normal is greater than 2500/μL). The patient receives a transfusion of a pool of five platelet concentrates over 30 minutes through a leukocyte-reduction filter. The patient’s blood type is known to be group A, and the blood bank releases group AB platelets for his transfusion. A half-hour after the transfusion, the patient develops severe hypotension; his temperature is 38.9 °C (102 °F), and he has rigors and slight dyspnea. A pulse oximeter evaluation shows 93% saturation, and no wheezing or lymphedema is noted. Vascular collapse and acute renal failure follow within a few hours, despite intravenous administration of fluids and the use of pressors. What is the most likely cause of this adverse transfusion reaction? A. Bacterial contamination B. Anaphylaxis C. ABC incompatibility D. Patient leukoagglutinins E. Donor leukoagglutinins

Hematology and Oncology:Question 54 A 68-year-old woman is hospitalized for right total hip replacement. She receives daily injections of unfractionated heparin for prophylaxis of venous thrombosis. On

Page 52: MKSAP13-Hematology and Oncology

admission her hemoglobin is 11.9 g/dL, leukocyte count is 6500/μL, and platelet count is 286,000/μL. On day 6 of hospitalization, the platelet count has fallen to 105,000/μL and a proximal right lower extremity deep venous thrombosis is documented by venous ultrasound. A hematology consultant recommends initiating systemic anticoagulation therapy with argatroban after the result of a heparin-platelet factor 4 enzyme-linked immunosorbent assay is positive. Argatroban therapy is initiated at a dose of 2 μg/kg per minute, which is titrated to a final dose of 3 μg/kg per minute to maintain a stable partial thromboplastin time of 2 X control. What would be the best schedule for discontinuing this patient’s agatroban therapy and initiating warfarin therapy? A. Initiate warfarin immediately after discontinuing argatroban B. Initiate warfarin 24 h after discontinuing argatroban C. Initiate warfarin at 5 mg/d, and discontinue argatroban when the INR is greater than 2.0 D. Initiate warfarin at 5 mg/d, and discontinue argatroban when the INR is greater than 4.0 E. Initiate warfarin at a loading dose of 10 mg/d, and discontinue argatroban when the partial thromboplastin time is greater than 4 X

Page 53: MKSAP13-Hematology and Oncology

control

Hematology and Oncology:Question 55 A 60-year-old woman is evaluated because her left leg has become warm and swollen 3 weeks after total right hip replacement. Compression ultrasound shows proximal deep venous thrombosis of the symptomatic leg. She was treated postoperatively with prophylactic doses of low-molecular-weight heparin for 1 week. How long should the therapy be continued? A. 6 Weeks B. 3 Months C. 1 Year D. For the rest of her life

Hematology and Oncology:Question 56 After delivery of her sixth child, a 37-year-old woman has severe vaginal bleeding from uterine atony. She weighs 70 kg (154 Ib). She receives 12 units of packed red cells and four units of fresh frozen plasma over 3 hours, along with 4 L of normal saline. Her prothrombin time and partial thromboplastin time are normal, and the plasma fibrinogen level is 330 mg/dL. Her platelet count was 160,000/μL before delivery and is now 55,000/μL. The bleeding is unresponsive to uterine massage, adminstration of prostaglandins, and uterine curettage. An emergency hysterectomy is planned, and she

Page 54: MKSAP13-Hematology and Oncology

receives a five-pack of pooled platelet concentrates. Platelet count 10 min after transfusion is 60,000/μL. What is the most likely cause of the 5000/μL platelet-count increment? A. Active bleeding B. Gestational thrombocytopenia C. Disseminated intravascular coagulopathy D. HELLP syndrome E. Alloimmunization

Hematology and Oncology:Question 57 A 55-year-old man of Greek ancestry is evaluated because of fatigue. He has had anemia since childhood but has never received a blood transfusion. He drinks wine regularly in the evening with his meal but denies excessive use of alcohol. On physical examination his liver is palpable 3 cm below the right costal margin. Laboratory studies: Hemoglobin 10.2 g/dL Mean corpuscular volume 70 fL Reticulocyte count 1.5% Serum ferritin 1800 ng/mL Transferrin saturation 40% Serum alanine aminotransferase 60 U/L Serum aspartate aminotransferase 40 U/L Serum bilirubin (total) 2.1 mg/dL with an elevated indirect fraction Serum haptoglobin Decreased

Page 55: MKSAP13-Hematology and Oncology

Serologic studies for hepatitis B and C Negative

Results of a test for the C282Y mutation in the HFE gene are negative. What would be the most appropriate next step in managing this patients presumptive iron overload? A. Phlebotomy to remove one unit of blood weekly until the serum ferritin level is less than 20 ng/mL B. Diagnostic liver biopsy to include staining for iron and determination of quantitative iron concentration C. Bone marrow study to include staining the aspirate with Perls’ reagent D. Desferrioxamine therapy, 50 mg/kg body weight daily by subcutaneous infusion over 10 hours

Hematology and Oncology:Question 58 A 74-year-old woman has an annual checkup. She has a history of congestive heart failure, atrial fibrillation, hypothyroidism, diet-controlled diabetes mellitus, and degenerative arthritis. She reports dyspnea on exertion and bilateral knee pain. She has lost 2.3 kg (5 Ib) since last year. She takes ramipril, furosemide, digoxin, thyroid hormone replacement, aspirin (81 mg/d), and a nonsteroidal anti-inflammatory drug. On physical examination, her temperature is

Page 56: MKSAP13-Hematology and Oncology

36.5 °C (97.7 °F), her pulse rate is 72/min, and her blood pressure is 105/70mm Hg. The spleen tip is palpable below the costal margin. There is no occult blood in her stool. Laboratory studies: Hemoglobin 11 .5 g/dL Hematocrit 36% Leukocyte count 55,000/μL Platelet count 370,000/μL Differential Mostly segmented and band neutrophils and circulating metamyelocytes Bone marrow aspirate smear shows a hypercellular specimen with a very high myeloid:erythroid ratio, a significant left shift, and 2% blasts. Karyotypic analysis shows a translocation between chromosomes 9 and 22 in all 20 metaphases. What is the best next step in the treatment of this patient? A. Hospitalize her to start induction chemotherapy B. Refer her for hospice care C. Administer interferon-alfa therapy D. Refer her to a bone marrow transplant center E. Initiate therapy with imatinib mesylate (Gleevec®)

Hematology and Oncology:Question 59 A 35-year-old woman is evaluated in her 36th week of pregnancy because of the gradual

Page 57: MKSAP13-Hematology and Oncology

onset of severe right upper quadrant abdominal pain. On physical examination, her blood pressure is 110/70 mm Hg. She has decreased bowel sounds and mild rebound tenderness in the right upper quadrant. Laboratory studies: Hemoglobin 9.6 g/dL Leukocyte count 4600/μL with 80% neutrophils Platelet count 35,000/μL Serum aspartate aminotransferase 120 U/L Serum total bilirubin 2.1 g/dL Urinalysis No protein The peripheral blood smear shows abundant schistocytes and decreased platelets. What is the most appropriate therapy for this patient? A. Administration of intravenous immunoglobulin B. Urgent delivery of the fetus C. Plasma exchange D. Aggressive diuresis E. Administration of antiplatelet agents

Hematology and Oncology:Question 60 A 74-year-old woman presents for an urgent office visit. She has been experiencing epigastric discomfort over the last 3 months. Discomfort is worse during meals and prevents her from eating full portions. She has lost 5.4

Page 58: MKSAP13-Hematology and Oncology

kg (12 Ib). She also reports dyspnea on exertion. She has been taking ramipril, aspirin, and simvastatin. On physical examination, her pulse rate is 88/min, and her blood pressure is 155/85 mm Hg. There is mild epigastric tenderness with palpation. Her spleen is massively enlarged and palpable at the umbilicus. The edge of her liver is also palpable 6 cm below the costal margin. Her hemoglobin is 7.5 g/dL, mean corpuscular volume 89 fL, leukocyte count 11,200/μL, and platelet count 114,000/μL. Peripheral blood smear shows numerous erythroblasts, myeloid precursors, and teardrop cells. Bone marrow cannot be aspirated, and biopsy specimen shows marked fibrosis. Analysis of blood is negative for t(9;22) by fluorescent in situ hybridization (FISH). What is the most likely diagnosis? A. Chronic myeloid leukemia B. Hairy cell leukemia C. Myelodysplastic syndrome D. Disseminated tuberculosis E. Myelofibrosis

Hematology and Oncology:Question 61 A 50-year-old woman has factor XI deficiency

Page 59: MKSAP13-Hematology and Oncology

(2% activity) and a history of menometrorrhagia and of excessive bleeding during previous foot surgery. She is group A, Rh-positive. Her only medication is ramipril for hypertension. The patient has a tooth abscess and requires oral surgery. Her hematologist orders four units of fresh frozen plasma to be given just before surgery. The units are group AB, Rh-positive. Thirty minutes after starting infusion of the first unit, she develops an erythematous, pruritic rash over her arms, trunk, and face with generalized urticaria and some lymphedema of the oral mucosa. Wheezing and stridor develop. Her systolic blood pressure is 90 mm Hg, and she complains of crampy abdominal pain and nausea. She is afebrile and has no back pain. Results of urinalysis are normal. What is the most likely cause of this transfusion reaction? A. Bacterial contamination B. Anaphylaxis C. ABC incompatibility D. Transfusion-related acute lung injury E. Use of an angiotensin-converting enzyme inhibitor

Hematology and Oncology:Question 62 A 40-year-old woman has an unprovoked first episode of deep venous thrombosis. She has no family history of thrombosis. Testing shows

Page 60: MKSAP13-Hematology and Oncology

that she is heterozygous for the prothrombin G2021 0A gene mutation, but she has no other biologic defects predisposing her to thrombosis. Which of the following is the most important consideration to relay to this patient? A. Her risk of recurrence is less than 5% per year B. The prothrombin gene mutation is a risk factor for an initial episode of deep venous thrombosis, and her first-degree relatives should be tested for the mutation C. The presence of the prothrombin gene mutation clearly identifies her as being at higher risk for recurrent deep venous thrombosis than patients without a prothrombotic defect D. She should take warfarin at a target INR of 2 to 3 for the next 2 years

Hematology and Oncology:Question 63 A 44-year-old woman with idiopathic thrombocytopenic purpura is evaluated because of worsening thrombocytopenia. She presented 3 years ago with menorrhagia and a platelet count of 10,000/μL. She responded for several months to oral corticosteroid therapy but ultimately required more than 60 mg/d of prednisone to maintain her platelet count above 10,000/μL. She underwent splenectomy 18 months ago, and her platelet count initially

Page 61: MKSAP13-Hematology and Oncology

improved. Approximately 6 months ago, however, her platelet count began to fall and currently is 12,000/μL. Her hemoglobin is 11.8 mg/dL, and leukocyte count is 8200/μL. Prednisone, 30 mg/d, had been reinitiated without significant improvement in her platelet count. Her blood type is group B and she is Rh-positive. A trial of which of the following therapies would NOT be appropriate for this patient? A. Intravenous immunoglobulin B. Cyclophosphamide C. Rituximab D. Intravenous anti-Rh(D) E. Danazol

Hematology and Oncology:Question 64 A 68-year-old woman is evaluated because of a lump on her chest wall. Seventeen years ago she developed stage I ductal adenocarcinoma (estrogen receptor-positive) of the left breast; her disease was managed with lumpectomy, breast radiation therapy, and 5 years of tamoxifen therapy. The lump she now has is separate from the breast, subcutaneous in location, fixed to the underlying 4th rib, and nontender. The mass is excised and found to be a fibrosarcoma. What is the relationship of this fibroscarcoma

Page 62: MKSAP13-Hematology and Oncology

to her original cancer and its treatment? A. It is not related to the original cancer or its treatment B. It is related to the radiation therapy given to control the breast cancer C. It is related to the tamoxifen given to control the breast cancer D. It is related to the primary estrogen receptor-positive breast cancer

Hematology and Oncology:Question 65 A 70-year-old man with an 80-pack-year smoking history is evaluated because of a chronic cough of 6 months duration. Chest radiograph shows a 3-cm mass in the left perihilar region, and bronchoscopic biopsy confirms poorly differentiated adenocarcinoma. There are no significant findings on physical examination, and all blood studies are normal. Bone scan and CT scan of the head are normal, but CT scans of the chest and abdomen show the mass and two 1-cm nodules in the right lobe of the liver. Intravenous contrast perfusion of the two nodules during the CT scan is not suspicious for hemangioma. Positron emission tomography scan shows distinct uptake in the left perihilar mass but only faint focal activity in the right lobe of the liver. Pulmonary function tests show mild obstructive disease. What is the best next step in this patient’s

Page 63: MKSAP13-Hematology and Oncology

management? A. Radiation therapy B. Chemotherapy C. CT-directed liver biopsy D. Serum carcinoembryonic antigen test E. Surgery

Hematology and OnCology:Question 66 A 25-year-old woman has her first routine check-up. Her mother died of lung cancer at the age of 60 years, and her father has had a head and neck cancer. She is very worried about getting cancer and wants to know what she can do to reduce her risk, as much as possible, of getting either of these cancers. In addition to avoidance of tobacco, which of the following approaches has been demonstrated to decrease risk for one or both of these cancers? A. Avoidance of alcohol abuse B. Daily intake of antioxidant vitamins C. Avoidance of exposure to benzene D. Daily intake of β-carotene

Hematology and Oncology:Question 67 A 66-year-old man has newly diagnosed adenocarcinoma of the prostate gland. His clinical stage is Tic (that is, clinically organ-confined and detected by screening serum prostate-specific antigen [PSA] blood test only). His PSA is 7 ng/mL and his Gleason score is 6.

Page 64: MKSAP13-Hematology and Oncology

He has decided to undergo local therapy for his prostate cancer. He has met with a radiation oncologist and a urologist and has decided to undergo external-beam radiation therapy. Which of the following side effects is he most likely to have with radiation therapy? A. Urinary incontinence B. Impotence C. Thrombosis D. Difficulties with bowel function

Hematology and Oncology:Question 68 A 69-year old black man is evaluated because of a history of steadily increasing upper abdominal pain, loss of appetite, and a 4.6-kg (1 0-Ib) weight loss over the past several months. He recently noticed that the color of his urine was darker than usual. The patient had a 40-pack-year history of cigarette smoking, but quit smoking 4 years ago. He has hypertension that is controlled by medical management. Laboratory studies: Hemoglobin 11.5 g/dL Serum total bilirubin 3.3 g/dL Serum albumin 3.2 g/dL Serum aspartate aminotransferase 105 U/L Serum alanine aminotransferase 95 U/L Urinalysis shows elevated bilirubin. CT scan of the abdomen shows dilation of the common

Page 65: MKSAP13-Hematology and Oncology

bile and pancreatic ducts, a 5.2-cm mass in the head of the pancreas, and compression of the superior mesenteric vein. A fine-needle aspirate of the mass reveals atypical cells that are suspicious for malignancy. What is the most appropriate course of action? A. Refer the patient for exploratory laparotomy, biopsy, and biliary bypass B. Request a CT-guided biopsy of the mass in the head of the pancreas to establish a diagnosis C. Obtain a consultation for endoscopic retrograde cholangiopancreatography and possible biopsy and biliary stent placement D. Refer the patient to a tertiary-care center with surgical expertise in the management of patients with pancreatic and hepatobiliary disease

Hematology and OnCology:Question 69 Which of the following cancer screening tests has been shown in randomized trials to decrease the risk of death from the target cancer? A. Fecal occult blood testing every 2 years for colorectal cancer B. Human papillomavirus DNA test for cervical cancer every year C. Pap smear of the cervix every year in

Page 66: MKSAP13-Hematology and Oncology

sexually active women for uterine cancer D. Chest radiography every year in cigarette smokers for lung cancer

Hematology and OnCology:Question 70 A 45-year-old woman is evaluated because of a palpable 2-cm right axillary lymph node. The lymph node is completely resected, and histologic study shows adenocarcinoma; hormone receptors are negative. Bilateral mammography is negative, as is MRI of the right breast. CT scans of the chest and abdomen are negative for tumor, and no further lymphadenopathy is detected. Results of bone scan and MRI of the head are also unremarkable. The patient had been previously healthy and has never smoked.

Which of the following statements about this patients condition is correct? A. Breast cancer is the most likely diagnosis, and optimal therapy for breast cancer paradigm should be initiated. B. Lung cancer is the most likely diagnosis, and optimal therapy for lung cancer paradigm should be initiated. C. All known disease has been resected, and the patient requires careful monitoring for possible future recurrence. D. Radiation therapy to the right axilla is required with fields encompassing the right

Page 67: MKSAP13-Hematology and Oncology

breast.

Hematology and Oncology:Question 71 A 42-year-old woman is evaluated because she has noticed a thickening in her left breast over the past few weeks. She noticed it a few months ago, but because it seems to come and go, she put off seeking medical attention. She is premenopausal. She had menarche at age 13 years. When she was 35 years, she had one child (whom she breast-fed) after a normal first full-term pregnancy. She took oral contraceptives for 10 years before her pregnancy. She has no known radiation exposure and no family history of breast cancer. On physical examination, there appears to be some asymmetry in breast tissue density in the upper outer quadrant of the left breast compared with the right one, but no discrete mass. There are no palpable lymph nodes. Results of a mammogram are negative. What would be the most appropriate next step in this patients management? A. Breast ultrasound; consultation with a surgeon experienced in breast diagnosis B. Reassurance that because her family history is negative and the mammogram is negative, no additional studies or treatment is required C. MRI of the breast

Page 68: MKSAP13-Hematology and Oncology

D. Measurement of circulating tumor markers, including CA15-3 and carcinoembryonic antigen

Hematology and Oncology:Question 72 A 26-year-old man with testicular cancer who is receiving chemotherapy is evaluated in the emergency department. His temperature is 38.9 °C (102 °F), and he complains of feeling flushed and tired. He has an indwelling central venous port, but there are no localizing symptoms to suggest a source of infection. Chest radiograph shows no abnormalities. The patients hemoglobin is 8.9 gIdL; the absolute neutrophil count is 165/μL and the platelet count is 56,000/μL. Results of other laboratory studies and urinalysis are within normal limits. Blood samples are obtained from a peripheral vein and through the port and sent for culture; a urine culture is also ordered. The patient is hospitalized and intravenous ceftazidime is initiated. Three days later, the patient’s temperature is 37.8 °C (100 °F) and absolute neutrophil count is 4504/μL; his clinical condition is otherwise stable. Results of blood and urine cultures are negative. A repeat chest radiograph is normal, and blood and urine specimens are again sent for culture. What is the most appropriate treatment strategy for this patient?

Page 69: MKSAP13-Hematology and Oncology

A. Continue the current antibiotic regimen B. Add vancomycin C. Switch to oral ciprofloxacin and amoxicillin-clavulanic acid D. Add granulocyte colony-stimulating factor E. Remove the central venous port

Hematology and Oncology:Question 73 A 43-year-old man is evaluated because of crushing substernal chest pain that developed during a pickup basketball game with colleagues at work. He was treated 15 years ago for stage IIB massive mediastinal Hodgkin’s disease. Treatment included doxorubicin, bleomycin, vinbiastine, and dacarbazine (ABVD) followed by mantle-field radiation therapy to a total dose of 4400 cGy. He has had no recurrence of his Hodgkin’s disease. He takes thyroid medication because he developed hypothyroidism 2 years after completing his therapy for Hodgkins disease. What is the most likely diagnosis? A. Recurrent Hodgkins disease B. Myocardial infarction C. Pulmonary fibrosis D. Anemia associated with secondary myelodysplasia evolving to acute leukemia E. Constrictive pericarditis

Page 70: MKSAP13-Hematology and Oncology

Hematology and Oncology:Question 74 Four years ago, a 67-year-old man had a serum prostate-specific antigen (PSA) level of 16 ng/mL. Biopsy specimen showed adenocarcinoma of the prostate gland. His Gleason score was 7. He was treated with external-beam radiation therapy. One month ago, the patient noted fatigue and rib pain. His PSA was found to be 87 ng/mL, and bone scan revealed diffuse metastatic disease. What is the best treatment for this patient? A. Chemotherapy with docetaxel B. Chemotherapy with mitoxantrone and prednisone C. Androgen ablation (medical or surgical) D. Radiation therapy with strontium-89

Hematology and Oncology:Question 75 A 68-year-old woman is evaluated because of rectal bleeding that began recently and a sense of fullness in the rectum. Flexible sigmoidoscopy shows a mass 11 cm from the anus, and biopsy reveals adenocarcinoma. The patient is referred to a colorectal surgeon, and a low anterior resection is performed. The primary tumor, a 4-cm moderately differentiated adenocarcinoma, penetrates the bowel wall. No lymph nodes are involved. What is the most appropriate next step in this

Page 71: MKSAP13-Hematology and Oncology

patient’s management? A. No chemotherapy or radiation therapy; annual flexible sigmoidoscopy B. Postoperative adjuvant chemotherapy and pelvic radiation therapy C. Postoperative pelvic radiation therapy D. Complete colonoscopy within the first year, repeated every 3 to 5 years

Hematology and Oncology:Question 76 A 58-year-old postmenopausal woman has been taking hormone replacement therapy with combined estrogen and progestin for the past 4 years because she was told it would decrease her risk for heart disease. Her father died at age 65 years of heart disease, and she is concerned that she is also at risk. Recently, her sister has been diagnosed with breast cancer. Her mother died of breast cancer. Three years ago, the patient had a breast biopsy that showed atypical hyperplasia. She has not had a hysterectomy. The patient is considering chemoprevention for breast cancer, but is nonetheless concerned about heart disease. Her calculated risk of breast cancer is 10.4% over the next 5 years. Which of the following options is reasonable to consider for this patient? A. Continue hormone replacement therapy alone for primary prevention of heart disease B. Continue hormone replacement therapy and

Page 72: MKSAP13-Hematology and Oncology

add tamoxifen C. Discontinue hormone replacement therapy and start tamoxifen D. Change her hormone replacement therapy to estrogen alone and add tamoxifen

Hematology and Oncology:Question 77 A 68-year-old man with locally advanced non-small-cell lung cancer is evaluated because of the new onset of low back pain over the past 2 weeks. It is relieved with ibuprofen, and his only other symptom is mild fatigue. The patient completed combined chemotherapy and radiation therapy 6 months ago, and restaging scans afterward showed marked shrinkage of the right perihilar mass. He has no muscle weakness. Plain radiograph of the thoracic and lumbar spine shows no abnormalities other than signs of mild osteoarthritis. Neurologic examination is unremarkable. What is the most appropriate next step in the management of this patient? A. Gallium scan B. Re-evaluation if the symptoms get worse C. CT scan of the chest and abdomen with bone windows of the spine D. MRI of the spine E. Intravenous dexamethasone and MRI of the spine

Page 73: MKSAP13-Hematology and Oncology

Hematology and Oncology:Question 78 A 72-year-old man is evaluated because of constipation, abdominal pain, and distention that have worsened over the past week. Two years ago, he was diagnosed with stage III rectal cancer (primary tumor and renal involvement) and underwent low anterior resection. Chemotherapy with 5-fluorouracil and leucovorin followed, and pelvic radiation was given with concurrent infusion of 5-fluorouracil. He has been having regular bowel movements, and results of his most recent colonoscopy (1 year ago) were unremarkable. On physical examination, his pulse rate is 100/min. He has orthostatic hypotension, a slightly distended abdomen with hyperactive bowel sounds, and some guarding to deep palpation in the left lower quadrant. Plain radiograph of the abdomen shows distended loops of small bowel, with no stool in the distal colon or rectum. The patient is hospitalized for bowel rest and intravenous hydration. What is the next step in this patient’s management? A. Complete colonoscopy B. Upper gastrointestinal series with small-bowel follow-through C. Positron emission tomography scan of the abdomen D. Measurement of serum carcinoembryonic

Page 74: MKSAP13-Hematology and Oncology

antigen E. CT scan of the abdomen with oral and intravenous contrast

Hematology and Oncology:Question 79 A 60-year-old postmenopausal woman at elevated risk for breast cancer is taking tamoxifen to reduce her risk. She has not had a hysterectomy. Which of the following surveillance strategies for the detection of endometrial cancer is most important to incorporate into this patient’s care? A. Annual transvaginal ultrasound B. Annual transabdominal pelvic ultrasound C. Annual endometrial aspiration sampling D. Biennial dilatation and curettage E. Annual routine bimanual pelvic examination

Hematology and Oncology:Question 80 An 82-year-old woman who has never smoked is evaluated because of a persistent cough. Chest radiograph shows several lung nodules and infiltrates. There are no significant findings on physical examination. The patient reports no weight loss. Blood studies show no abnormalities. CT scans confirm pulmonary involvement only, and results of bone scan and CT scan of the head are normal. The patient most likely has which of the following histologic types of lung cancer?

Page 75: MKSAP13-Hematology and Oncology

A. Bronchoalveolar cell carcinoma B. Small-cell lung cancer C. Large-cell carcinoma D. Squamous cell carcinoma E. Carcinoid tumor

Hematology and OnCology:Question 81 A 45-year-old woman has recently undergone lumpectomy and radiation therapy for a stage II breast cancer. She received chemotherapy for 6 months. Her periods ceased while she was receiving chemotherapy, and she remains amenorrheic. Her tumor was rich in estrogen receptor, and she is taking tamoxifen. Her hair is growing back, her energy is returning, and she has no specific complaints, but she is worried about recurrence. In addition to routine follow-up, what is the most appropriate management of this patient? A. Positron emission tomography now and annually B. No further management C. Routine tumor marker evaluation every 3 to 4 months (serum CAl 5-3, carcinoembryonic antigen) D. Bone scan and annual CT of the chest, abdomen, and pelvis E. Estrogen replacement therapy

Page 76: MKSAP13-Hematology and Oncology

Hematology and Oncology:Question 82 A 66-year-old black woman diagnosed with stage III colon cancer underwent definitive resection of the primary tumor 3 years ago. After surgery, she received adjuvant chemotherapy with 5-fluorouracil and leucovorin. Approximately 20 months later, she developed metastatic disease in the liver (five lesions in right and left lobe) and lungs (one lesion in the right and left lobe). She was treated with irinotecan, 5-fluorouracil, and leucovorin, and had a partial response to therapy. One month ago, disease progression was documented on restaging CT scans. The patient’s laboratory studies show relatively normal organ function. She has fatigue, decreased appetite, and has noticed a 2.3-kg (5-Ib) weight loss over the past 3 months. She has stopped participating in weekend bike trips with a cycling club, but remains involved in church and family activities and states that she wants to be as aggressive as possible in fighting the cancer. What is the most reasonable recommendation for this patient? A. Metastatectomy B. Hepatic arterial infusion C. Second-line chemotherapy regimen with leucovorin and oxaliplatin

Page 77: MKSAP13-Hematology and Oncology

D. High-dose chemotherapy with autologous peripheral stem cell transplantation

Hematology and Oncology:Question 83 A 68-year-old man is evaluated because of worsening chronic epigastric pain. He now has fatigue and early satiety. He has iron deficiency anemia. Results of upper gastrointestinal endoscopy reveal diffuse gastritis, along with mucosal thickening in the gastric antrum associated with a mass lesion. Abundant Helicobacter pylon organisms are noted on biopsy, and histologic evaluation of the mass lesion shows it to be a gastric lymphoma of mucosa-associated lymphoid tissue (MALT) type. What is the most appropriate next step in the management of this patients illness? A. Combination chemotherapy with 5-fluorouracil, doxorubicin, and mitomycin C (FAM) B. Combination chemotherapy with cyclophosphamide, doxorubicin, vincristine, and prednisone (CHOP) C. Four cycles of CHOP followed by involved-field radiation therapy D. Eradication of Helicobacter pylon E. Total gastrectomy followed by radiation

Page 78: MKSAP13-Hematology and Oncology

therapy

Hematology and Oncology:Question 84 A 65-year-old woman has a modified radical mastectomy for a 1.0-cm, well-differentiated breast cancer. The tumor is positive for estrogen and progesterone receptors and negative for HER2. Sentinel node mapping and excision show that none of the three lymph nodes removed is positive for metastasis. She is otherwise healthy. What is the best treatment for this patient at this time? A. Chest wall radiation therapy and tamoxifen for 5 years B. Chest wall radiation therapy and anastrozole for 5 years C. Tamoxifen for 5 years D. Tamoxifen and anastrozole for 5 years

Hematology and Oncology:Question 85 A 59-year-old woman with an 80-pack-year smoking history is evaluated because of weight loss and severe pain in the upper part of her left leg. She has lost 11.3 kg (25 Ib) from baseline weight of 59 kg (130 Ib). A large lytic lesion is noted on the left femur, with erosion into the cortex. Bone scan shows multiple lesions, and CT scan of the chest shows a large left hilar mass and med iastinal lymphadenopathy. Bronchoscopic biopsy

Page 79: MKSAP13-Hematology and Oncology

specimen shows poorly differentiated adenocarcinoma. Radiation therapy to the left femur is initiated. Which of the following findings would preclude use of palliative chemotherapy for this patient?

A. A solitary liver metastasis with normal serum bilirubin level B. Multiple liver metastases with a serum bilirubin level of 3.0 mg/dL C. A cytologically positive pleural effusion D. Hypercalcemia E. Poor performance status

Hematology and Oncology:Question 86 A 57-year-old man has tried to stop smoking unsuccessfully for the past year. He has tried smoking cessation counseling and use of nicotine gum. Although he has no new symptoms, the ‘smokers cough” that he has had for years is a constant reminder that he is at risk of dying of lung cancer. He wants to know what measures he can take that have been shown to reduce that risk. In addition to a smoking cessation program, what is the best recommendation for this patient? A. Daily dietary supplementation with the antioxidantl3-carotene B. Daily isotretinoin, titrated to limit skin toxicity

Page 80: MKSAP13-Hematology and Oncology

C. Daily bupropion D. An annual low-dose spiral CT scan of the chest E. Sputum cytology every4 months

Hematology and Oncology:Question 87 A 71-year-old man with mild chronic obstructive pulmonary disease and mild hypertension is evaluated during an annual routine visit. Review of systems is notable for intermittent cough, increasing dyspnea on exertion, a 2.3-kg (5 Ib) weight loss, and fatigue. The patient takes aspirin, 81 mg/d, and hydrochlorothiazide, 50 mg/d orally. He smoked one pack of cigarettes per day for 49 years but quit smoking 2 years ago. On physical examination, distant breath sounds are audible in both lungs and there are scattered rhonchi. Chest radiograph shows a perihilar mass. Abnormal laboratory results include hemoglobin of 12.5 g/dL and a serum sodium of 127 meq/L. Endobronchial biopsy reveals small-cell lung cancer. Further staging studies suggest that the disease is limited-stage. What is the most appropriate treatment for this patient’s hyponatremia? A. Fluid restriction to 1 L/d B. Fluid restriction to 1 L/d and demeclocycline therapy C. Discontinuation of hydrochiorothiazide

Page 81: MKSAP13-Hematology and Oncology

D. Combination chemotherapy for the small-cell lung cancer

Hematology and Oncology:Question 88 A 23-year-old man is evaluated because of a painless right-sided scrotal mass. Ciprofloxacin, 500 mg every 12 hours, is administered for 10 days, but he notes little improvement in the swelling. His serum α-fetoprotein level is elevated at 100 ng/mL, and his 13-human chorionic gonadotropin level is 64 m/UImL. Testicular ultrasound examination reveals a hypoechoic mass. Which of the following would be the most appropriate next step in his treatment? A. Retroperitoneal lymph node dissection B. Combination chemotherapy with bleomycin, etoposide, and cisplatin C. Radiation to the pelvis D. Inguinal orchiectomy

Hematology and Oncology:Question 89 A 40-year-old woman has a routine gynecologic examination. At the age of 32 years, she gave birth to twins after receiving fertility drugs. She has just seen a television program on ovarian cancer, and is concerned because previous use of fertility drugs and a family history of ovarian cancer were mentioned as possible risk factors. Her mother developed endometrial cancer at

Page 82: MKSAP13-Hematology and Oncology

56 years, and a paternal uncle was diagnosed with a type of lymphoma at 60 years. She has one older sister with hypertension and one younger brother who is healthy. Results of her pelvic examination are unremarkable, but she asks what additional tests she should have to look for ovarian cancer in the early stages that cannot be detected by physical examination. What is the most appropriate management for this patient? A. Blood test for CA-125 B. Rectovaginal pelvic examination in 1 year C. Transvaginal ultrasound D. Screening test for BRCA-1 and BRCA-2

Hematology and Oncology:Question 90 A 44-year-old woman has a lumpectomy and radiation therapy for stage II breast cancer. She receives 6 months of chemotherapy. Her periods cease while she is receiving chemotherapy, and she remains amenorrheic. Her tumor was found to be rich in estrogen receptor, and she is taking tamoxifen.

Her hair is growing back and her energy is returning, but she is having severe hot flushes that keep her awake at night. She is so tired during the day that she is unable to perform her job in a satisfactory manner. She also relates that she and her husband have been unable to have satisfying sexual relations

Page 83: MKSAP13-Hematology and Oncology

because intercourse is painful for her. What would be the most appropriate treatment for this patient? A. A selective serotonin reuptake inhibitor and use of nonhormonal vaginal lubricating preparations B. Reassurance that these normal physiologic responses to menopause will resolve in time C. Estrogen replacement therapy with a progesterone supplement D. Work-up for endometrial cancer because she is taking tamoxifen

Hematology and Oncology:Question 91 A 64-year-old white man presents for serum prostate-specific antigen (PSA) screening at his wife’s urging. He is in generally good health, except for mild hypertension. His digital rectal examination reveals a mildly enlarged prostate gland with no discrete nodules. He has no family history of prostate cancer. What is the best advice to give him? A. There is no evidence that PSA screening leads to the earlier detection of prostate cancer B. There is insufficient evidence to establish whether PSA screening affects overall mortality rates C. On the basis of his lack of symptoms it is unlikely that he has an elevated PSA D. Because a digital rectal examination is more

Page 84: MKSAP13-Hematology and Oncology

sensitive than a serum PSA test, his normal examination makes it unlikely that he has cancer

Hematology and Oncology:Question 92 A 58-year-old woman with a 60-pack-year smoking history is evaluated because of hemoptysis and weight loss. Chest radiograph and CT scan show a right perihilar mass with mediastinal adenopathy. The results of CT scans of the abdomen, bone scan, and MRI of the head are otherwise negative. Examination of a specimen by bronchoscopic biopsy confirms small-cell lung cancer, and the findings suggest limited-stage disease. After the patient completes mediastinal radiation therapy and four cycles of cisplatin and etoposide, repeat CT scans indicate that her disease is in complete remission. What is the best recommendation concerning further therapy? A. Prophylactic cranial irradiation B. Four more cycles of chemotherapy, including a taxane C. Resection of the localized disease D. No further therapy; follow-up only

Hematology and Oncology:Question 93 A 63-year-old man is evaluated because of a several-month history of increasing fatigue and some vague upper abdominal discomfort. His

Page 85: MKSAP13-Hematology and Oncology

medical history includes gastroesophageal reflux disease, coronary artery disease, and clinical depression, all of which are well controlled with medications. The patient has been working full time. On physical examination, he has mild hepatomegaly. His hemoglobin is 12.2 g/dL, serum alkaline phosphatase level 280 U/L, and serum aspartate aminotransferase level 65 U/L. CT scan of the abdomen and pelvis shows multiple hepatic lesions ranging in size from 1 cm to 4 cm; mesenteric, para-aortic, and paracaval lymphadenopathy, and a colonic mass at the splenic flexure. Colonoscopy reveals a nonobstructing, non bleeding lesion; biopsy shows it to be poorly differentiated adenocarcinoma. Fine-needle aspiration of one of the liver lesions confirms the presence of malignant cells consistent with a primary colon cancer. What is the most appropriate next step in the management of this patient? A. Resection of the primary tumor followed by systemic chemotherapy B. Combined regional chemotherapy to the liver and systemic chemotherapy C. Exploratory laparotomy with resection of the primary tumor and placement of a hepatic arterial infusion pump D. Systemic chemotherapy

Page 86: MKSAP13-Hematology and Oncology

E. Best supportive care

Hematology and Oncology:Question 94 A 69-year-old man underwent a radical prostatectomy ii years ago. His serum prostate-specific antigen (PSA) level was 7.1 ng/mL, clinical stage was Tic (clinically organ-confined and detected by screening PSA blood test only), and his Gleason score was 5. His PSA level became detectable 4 years ago. Over the past 3 years, his PSA values have been 1.4 ng/mL, 1 .8 ng/mL, and 2.2 ng/mL. He remains asymptomatic, and a recent bone scan revealed only degenerative disease. The patients risk for rapid metastasis is low for which of the following reasons? A. He was initially treated by radical prostatectomy. B. His PSA level became undetectable. C. He has had a slow, prolonged rise in his PSA.

D. He is almost 70 years old.

Hematology and Oncology:Question 95 A 61-year-old man with a 120-pack-year smoking history is evaluated for a persistent and worsening cough and found to have a 3-cm mass in the right upper lobe on chest radiography. CT-directed needle biopsy is positive for squamous cell carcinoma. CT scans of the abdomen and pelvis, bone scan, and MRI

Page 87: MKSAP13-Hematology and Oncology

of the head are negative for metastatic disease. Positron emission tomography scan shows uptake only in the right upper lobe mass, and pulmonary function tests indicate that the patient has adequate pulmonary reserve to undergo resection. The patient undergoes a right upper lobectomy. All margins are clear, and all peribronchial lymph nodes are negative for tumor within the resected specimen. What is the most reasonable adjuvant therapy for this patient? A. Four cycles of combination chemotherapy including a taxane B. Four cycles of combination chemotherapy including a taxane, followed by radiation therapy to the draining lymph node sites in the mediastinum C. Radiation therapy to the draining lymph node sites in the mediastinum D. Six cycles of combination chemotherapy including a platinum agent E. No radiation therapy or chemotherapy is required at this time

Hematology and Oncology:Question 96 A 47-year-old woman is recovering from surgery for stage III ovarian cancer (spread to the peritoneal cavity but without parenchymal liver involvement). Not all of the visible cancer could be resected; the largest residual tumor

Page 88: MKSAP13-Hematology and Oncology

was 3 cm in diameter. Her performance status is excellent. She is interested in pursuing aggressive therapy and wishes to be treated as soon as possible in her local community. The proposed systemic therapy will involve cisplatin or carboplatin and paclitaxel. What further treatment is appropriate for this patient? A. No further treatment beyond the proposed standard therapy B. Another drug that has a different mechanism of action C. Regional peritoneal therapy D. High-dose therapy with peripheral or bone marrow stem cell rescue E. A second-look laparotomy

Hematology and Oncology:Question 97 A 62-year-old postmenopausal woman with a family history of breast cancer in two first-degree relatives wishes to consider taking tamoxifen to reduce her risk of breast cancer. Her baseline risk of breast cancer qualifies her for consideration of tamoxifen (i.e., an absolute risk of at least 1.66% over the next 5 years). She has had a prior hysterectomy, but her ovaries are intact. Which of the following effects of tamoxifen are relevant to the decision? A. Decreased risk of myocardial infarction or

Page 89: MKSAP13-Hematology and Oncology

ischemic heart disease B. Increased risk of deep venous thrombosis C. Increased risk of ovarian cancer D. Increased risk of major depression E. Increased risk of colorectal cancer

Hematology and Oncology:Question 98 A 50-year-old perimenopausal woman is evaluated because of abdominal swelling. CT scan of the chest, abdomen, and pelvis shows mesenteric lymphadenopathy and a small amount of ascites but no ovarian masses. Needle biopsy specimen shows adenocarcinoma; the tumor is found to be negative for hormone receptors. The serum carcinoembryonic antigen level is normal at 2.0 ng/mL and the serum CA-125 is 1200 ng/mL (elevated). What is the best next step in her treatment? A. Combination chemotherapy B. Debulking surgery C. Radiation therapy to a wide port D. Positron emission tomography scan to assess other sites of disease E. Bone scan to rule out skeletal metastases

Hematology and Oncology:Question 99 A 50-year-old menstruating woman has a 1 .5-cm moderately differentiated breast cancer.

Page 90: MKSAP13-Hematology and Oncology

The lesion is completely excised, and the margins of the excision are negative. Axillary node sampling shows that she has three positive nodes. The tumor is negative for estrogen and progesterone receptors and is highly positive for HER2. She is otherwise healthy. Her mother had breast cancer at 62 years and was treated by mastectomy; she is alive and healthy at the age of 80 years. The patients sister had breast cancer at 54 years and was treated by breast conservation therapy; 4 years later she died of a recurrence of breast cancer. The patient has seen two different surgeons with opposing viewpoints regarding the best treatment, and she has been reading extensively on the Internet and has become confused about her options.

Which of the following represents the best treatment for this patient? A. Modified radical mastectomy, followed by tamoxifen and chemotherapy B. Chemotherapy with no further treatment to the breast C. Chemotherapy and tamoxifen, with no further treatment to the breast D. Chemotherapy and radiation therapy to the breast and axillary regions E. Chemotherapy, tamoxifen, and radiation

Page 91: MKSAP13-Hematology and Oncology

therapy to the breast and axillary regions

Hematology and Oncology:Question 100 A 32-year-old woman is evaluated because of a painless swelling in her lower neck. She noted the swelling about 3 weeks ago and now thinks it is getting larger. She has no history of foreign travel. She smokes but does not drink alcohol. She owns a cat. She has not had fever or unexplained weight loss, but has had night sweats twice in the past week. She has been feeling fatigued lately and has noted dyspnea on exertion. On physical examination, a 3 X 5-cm left supraclavicular lymph node is palpable. It is firm but not rock-hard and is nontender. The remainder of her physical examination and results of a complete blood count are normal. What is the best next step in this patient’s management? A. Observe for 2 weeks B. Refer her to an otolaryngologist for a thorough search for a primary lesion of the oropharyngeal mucosa C. Administer oral antibiotics and observe the course of the node D. Perform a needle aspiration of the node E. Perform a chest radiograph

Hematology and Oncology:Question 101 A 64-year-old man is evaluated because of

Page 92: MKSAP13-Hematology and Oncology

fatigue and shortness of breath on exertion. He has had three episodes of urinary tract infection in the past 7 months. On physical examination, he has pallor but is otherwise normal. He denies blood loss, and his stool is negative for occult blood on three measurements. He is anemic (hemoglobin 8.4 g/dL) with normochromic, normocytic indices, and his serum creatinine level is 2.9 mg/dL. The total serum protein level is elevated, and the serum albumin level is low normal. His serum calcium level is 11.8 mg/dL. What is the most likely cause of his symptoms?

A. Multiple myeloma B. Chronic pyelonephritis C. Iron deficiency anemia D. Colon cancer E. Small-cell lung cancer

Hematology and Oncology:Question 102 A 63-year-old man is evaluated because of new-onset hemoptysis and a 100-pack-year smoking history. Chest radiograph shows a 4-cm right perihilar mass, and bronchoscopic biopsy and cytologic evaluation of sputum confirm squamous cell carcinoma. There are no significant findings on physical examination, and the patient reports no weight

Page 93: MKSAP13-Hematology and Oncology

loss. Complete blood count is normal except for a serum alkaline phosphatase level at 110 U/L. CT scans of the chest and abdomen show no mediastinal adenopathy. Results of a positron emission tomography scan are negative, except for the right perihilar mass.

In which of the following situations would surgical therapy be offered? A. A solitary bone metastasis is noted on bone scan, and a lytic lesion is confirmed radiographically. B. A solitary liver metastasis is noted on CT scan and confirmed by needle biopsy and positron emission tomography scan. C. A solitary brain metastasis is noted on CT scan of the head. D. A small effusion is noted in the right hemithorax, and cytopathologic examination of the pleural fluid is positive for tumor.

Hematology and Oncology:Question 103 A 40-year-old woman was treated for breast cancer 5 years ago by lumpectomy, breast irradiation, and 3 months of chemotherapy. Her original cancer was negative for estrogen and progesterone receptors and unequivocally positive for HER2. Her periods stopped during chemotherapy but

Page 94: MKSAP13-Hematology and Oncology

resumed 4 months later, and she remains premenopausal now. She is evaluated now because of pain in her back, a nagging cough, and fatigue. Physical examination shows palpable skin nodules over the affected breast, axillary adenopathy, and dullness to percussion, and decreased breath sounds in the base of the left lung; her liver edge is palpable below the right costochondral border. Liver function values are approximately twice normal; serum bilirubin is normal. Radiograph of the chest shows multiple pulmonary nodules and a left pleural effusion confirmed by CT, which also shows the liver nodules. Bone scan and MRI are consistent with bone metastases. Biopsy of one of the skin lesions is consistent with metastatic breast cancer; it is negative for estrogen and progesterone receptors, and positive for HER2. What is the most appropriate treatment for this patient? A. Hospice care and comfort measures B. Combination endocrine therapy with tamoxifen and an aromatase inhibitor C. High-dose chemotherapy with bone marrow stem cell support D. Trastuzumab and taxane-based chemotherapy E. Combination endocrine therapy with ovarian ablation and an aromatase inhibitor

Page 95: MKSAP13-Hematology and Oncology

Hematology and Oncology:Question 104 A 65-year-old man has a routine screening examination, and his serum prostate-specific antigen (PSA) level is found to be 6.7 ng/mL. Digital rectal examination reveals a mildly enlarged prostate gland, but no discrete nodules. One year ago, his PSA was 2.1 ng/mL. The patient is referred to a urologist who performs a biopsy of the gland. Biopsy specimen shows adenocarcinoma of the prostate with a Gleason score of 9. Which of the following best describes his prognosis on the basis of his Gleason score? A. The Gleason score is not reproducible and should not be used as a prognostic variable. B. Because the patient is asymptomatic, the Gleason score will not be predictive of outcome. C. The Gleason score indicates that the patient would have a high risk of recurrence after a radical prostatectomy. D. The Gleason score indicates that the patient would have a very low risk of recurrence after radiation therapy.

Hematology and OnCology:Question 105 A 78-year-old man is evaluated because of shortness of breath of acute onset. The patient reports a 13.6-kg (30-Ib) weight loss, vague upper abdominal discomfort, nausea, loss of

Page 96: MKSAP13-Hematology and Oncology

appetite, and fatigue. He has hypertension and symptomatic atherosclerotic coronary artery disease despite optimal medical management. Even before the recent episode of dyspnea, his physical activity has been limited; he spends most of the day resting either in bed or in a chair. Laboratory studies: Hemoglobin 10.5 g/dL Serum albumin 2.6 g/dL Serum alanine aminotransferase 65 U/L Serum aspartate aminotransferase 78 U/L CA 19-94500 U/L Arterial oxygen saturation 85% by pulse oximetry Spiral CT scan of the chest shows a pulmonary embolism. The patient is hospitalized for anticoagulation and oxygen therapy and gradually improves. Subsequent CT of the abdomen shows a 4-cm mass in the tail of the pancreas and numerous low-attenuation lesions in the liver. CT-guided biopsy of the hepatic lesions and the pancreatic mass is deferred during this hospitalization. What is the most appropriate next step in the management of this patient? A. Best supportive care and referral to a hospice B. Diagnostic CT-guided biopsy under a heparin

Page 97: MKSAP13-Hematology and Oncology

window” when the patients clinical status improves C. Neurolytic celiac axis block D. Palliative chemotherapy E. Palliative radiation therapy

Hematology and Oncology:Question 106 A 76-year-old man was diagnosed with prostate cancer 8 years ago. At that time he had a serum prostate-specific antigen (PSA) level of 12 ng/mL, a Gleason score of 6, and a clinical stage of T2b (organ-confined on digital rectal examination but involving both lobes of the prostate gland). He was treated with external-beam radiation therapy, and his PSA level dropped to a low of 1.4 ng/mL. Four years later, his PSA level began to rise and now is 14.1 ng/mL. A recent bone scan and CT scans revealed no metastatic disease. His urologist has suggested initiating a course of leuprolide. Which of the following is a potential side effect of leuprolide that the patient should be informed about before commencing therapy? A. Impaired urinary flow B. Bone thinning C. Weight loss D. Increased hair growth

Hematology and Oncology:Question 107 A 68-year-old man with a 60-pack-year

Page 98: MKSAP13-Hematology and Oncology

smoking history is evaluated because of hemoptysis. Radiograph of the chest shows a right hilar mass and mediastinal widening. CT scans of the chest and abdomen confirm the hilar mass and bulky lymphadenopathy in the mediastinum with no other overt metastases. Bronchoscopic biopsy specimen shows small-cell lung cancer. Bone scan and CT scan of the head are negative for tumor. A low serum sodium level and inappropriately high urine osmolality suggest that he has the syndrome of inappropriate antidiuretic hormone secretion. Which of the following is the most appropriate treatment recommendation? A. Surgery followed by chemotherapy B. Radiation therapy followed by chemotherapy

C. Chemotherapy alone D. Radiation therapy and concomitant chemotherapy E. Chemotherapy followed by radiation therapy

Hematology and Oncology:Question 108 A 78-year-old woman with metastatic breast cancer involving the bones and soft tissues who has been taking hormone replacement therapy is evaluated in the emergency department because of lethargy and weakness, nausea, thirst, and dizziness. She is orthostatic and clinically dehydrated.

Page 99: MKSAP13-Hematology and Oncology

She has a history of congestive heart failure that has been controlled with medications. Laboratory studies: Blood urea nitrogen 42 mg/dL Total serum calcium 11 .4 mg/dL Serum creatinine 1.6 mg/dL Serum albumin 3.0 g/dL What is the most appropriate initial treatment? A. Slow rehydration with half-normal saline B. Intravenous administration of a bisphosphonate C. Vigorous rehydration with normal saline D. Intravenous administration of furosemide along with saline rehydration E. Intravenous administration of corticosteroids

Hematology and Oncology:Question 109 A 52-year-old woman who is a nonsmoker has a 3-cm right inguinal lymph node. Biopsy specimen shows a poorly differentiated malignancy that is difficult to characterize by light microscopy. The biopsy specimen is negative for leukocyte common antigen, cytokeratin, and estrogen receptors. Which of the following additional tests would best establish the source of this tumor? A. Bone scan B. Measurement of serum carcinoembryonic

Page 100: MKSAP13-Hematology and Oncology

antigen C. Stain for S-100 D. Measurement of serum CA 19-9

Hematology and Oncology:Question 110 A 59-year-old man who is a heavy smoker presents with cough, shortness of breath, and a sensation of head fullness. Physical examination is notable for cervical venous distention and facial edema. A chest radiograph shows a widened superior mediastinum. What is the most appropriate initial intervention for this patient? A. Initiation of radiation therapy while a diagnostic work-up is pursued B. CT of the chest and consultation with a pulmonologist and a thoracic surgeon C. CT of the chest and a venous dye study to rule out thrombosis of the superior vena cava D. Mediastinoscopy with biopsy E. Ventilation/perfusion scan

Hematology and Oncology:Question 111 A 29-year-old man was diagnosed with stage II (that is, spread from the primary tumor to retroperitoneal lymph nodes) nonseminomatous germ cell tumor 1 year ago. Treatment included orchiectomy and retroperitoneal lymph node dissection.

Page 101: MKSAP13-Hematology and Oncology

He now presents with an elevated level of β-human chorionic gonadotropin hormone and multiple pulmonary lesions ranging in size from 0.61 to 3.22 cm. Needle biopsy specimen of the largest lesion reveals a germ cell tumor consistent with the initial diagnosis. He is asymptomatic. What is the best treatment for this patient? A. Surgical resection of the lung lesions B. Combination chemotherapy with bleomycin, etoposide, and cisplatin C. Radiation therapy to the chest D. Watchful waiting

Hematology and Oncology:Question 112 A 64-year-old woman has the following family history: One sister, who was a smoker, had lung cancer at the age of 63 years; another sister had breast cancer at the age of 70 years. Her father had prostate cancer at 82 years of age, and her mother had breast cancer at age 71 years. She has three other sisters, now aged 58, 60, and 65 years who have no history of cancer. She has two daughters and two sons between the ages of 25 and 39 years, who have no history of cancer. Her ancestry is European in general, and she is not of known Ashkenazi Jewish descent. She is concerned that “cancer runs in the family” and would like to have one of those “gene tests” she has read

Page 102: MKSAP13-Hematology and Oncology

about. What is the best advice to give her and her family? A. Have blood testing for abnormalities in known germ-line tumor suppressor genes (such as BRCA-1 and BRCA-2). B. Encourage her children to have genetic counseling and testing for abnormalities in known germ-line tumor suppressor genes. C. Advise her children to start routine cancer screening tests immediately, including mammography, colonoscopy or sigmoidoscopy, and prostate-specific antigen testing. D. Advise her that genetic testing is unnecessary, and recommend that she and her family continue screening and risk-reduction strategies as recommended for the general population. E. Recommend bilateral prophylactic mastectomies and oophorectomies for her and her daughters.

Hematology and Oncology:Question 113 A 72-year-old man is evaluated because of back pain and fatigue. Laboratory values indicate he is anemic, with normal leukocyte count and platelet count. He has mild hypercalcemia and a normal serum creatinine level. His gamma globulins are elevated, and serum protein electrophoresis shows the presence of a monoclonal protein, an IgG-K

Page 103: MKSAP13-Hematology and Oncology

light chain containing immunoglobulin at 4.4 g/dL. His bone marrow contains 20% plasma cells. A skeletal survey reveals multiple osteolytic lesions in the spine, ribs, and skull. Therapy with melphalan and prednisone is initiated. Which of the following agents would NOT be routinely used in his management? A. Erythropoietin B. Radiation therapy C. Intravenous gamma globulin D. Adequate analgesia (including narcotics, if necessary) E. A bisphosphonate

ANSWERSHematology and Oncology:Question 1The correct answer is D Educational Objectives Diagnose thrombocytopenia in a critically ill patient. Critique The acute onset of a febrile illness accompanied by leukocytosis, hypotension,

Page 104: MKSAP13-Hematology and Oncology

and thrombocytopenia is consistent with sepsis, although the cause may not be immediately evident. Routine studies not mentioned should involve blood and urine cultures, and chest radiography with sputum examination and culture if indicated. As always, the peripheral blood smear should be examined closely in any patient with hematologic abnormalities. The levels of fibrin D-dimer and total fibrinogen should be assessed in this patient, in conjunction with routine coagulation studies. Increased levels of D-dimer and low-normal or decreased levels of fibrinogen would be consistent with disseminated intravascular coagulation. Since the patient was known to be in previously good health without underlying hematologic disease, it is likely that the thrombocytopenia is acute and results from accelerated platelet clearance. Therefore, a bone marrow aspiration and biopsy are not likely to be helpful. Measuring factor VIII will not provide useful information in this case—the level may be normal, decreased due to consumption, or increased due to acute inflammation. Platelet-associated IgG levels are elevated in many disorders, including sepsis, and therefore will not provide useful information concerning the likelihood of a

Page 105: MKSAP13-Hematology and Oncology

primary immune-mediated thrombocytopenia. The bleeding time has poor predictive value for bleeding and is usually prolonged in thrombocytopenic patients. Diagnosis of disseminated intravascular coagulation would provide a likely explanation for the patients thrombocytopenia, although sepsis alone may lead to decreased platelet counts. Moreover, the coagulation status in a thrombocytopenic patient should be assessed, and factor replacement initiated if indicated.

Hematology and Oncology:Question 2 The correct answer is E Educational Objectives Treat a first unprovoked episode of pulmonary emboli provoked by a transient risk factor. Critique The duration of the anticoagulation therapy this patient has received is appropriate (3 to 6 months) for a first episode of venous thromboembolism. An episode of pulmonary embolism that is associated with a transient risk factor (such as oral contraceptive use) carries a substantially lower annual risk of recurrence than one that is “unprovoked.’ Although heterozygosity for the prothrombin G2021 0A mutation is a risk factor for a first episode of unprovoked embolism (and embolism in association with oral contraceptive use), its presence does not confer an increased

Page 106: MKSAP13-Hematology and Oncology

risk of recurrence. She should not resume taking oral contraceptives and be made aware of the symptoms of recurrent thromboembolism. The recurrence-prevention benefits of anticoagulation at an INR of 2 to 3 for more than 3 to 6 months have not been demonstrated to outweigh the bleeding risk associated with extended treatment. Although low-intensity warfarin (target INR 1 .5 to 2) was recently shown to protect against recurrent venous thrombosis without a significant increase in bleeding risk, this patient’s relatively low annual recurrence risk in the absence of provocative risk factors would argue against the continued use of even low-intensity warfarin. The extent to which oral anticoagulation at a target INR of 1.5 to 2 provides antithrombotic protection to women with a history of pulmonary embolism and continued use of oral contraceptives is unknown; it is not a preferred recommendation for this patient. Aspirin or a combination of aspirin and dipyridamole has not been shown to decrease the risk of recurrent venous thromboembolism, although aspirin is effective in the primary and secondary prevention of myocardial infarction.

Hematology and Oncology:Question 3 The correct answer is B

Page 107: MKSAP13-Hematology and Oncology

Educational Objectives Recognize therapeutic options in patients with sickle cell disease. Critique Hydroxyurea therapy reduces the incidence of sickle-cell-related events through its capacity to re-activate expression of the developmentally silenced fetal gamma globin. The potential toxicities of hydroxyurea are well described. The patients levels can be easily monitored through periodic complete blood counts. The patient’s current transfusion-related decrease in hemoglobin S should not affect the decision to initiate therapy, because the benefits of hydroxyurea are generally delayed. Although bone marrow transplantation might be a consideration for this patient, it is attended by substantially higher toxicity than hydroxyurea therapy. Gene therapy approaches for the treatment of sickle cell disease remain investigational. There is no role for either long-term transfusion therapy or oxygen therapy for this patient.

Hematology and Oncology:Question 4 The correct answer is B Educational Objectives Diagnose and treat drug-induced neutropenia. Critique The patient should be treated with granulocyte

Page 108: MKSAP13-Hematology and Oncology

colony-stimulating factor. Two types of neutropenia are reported with clozapine. One is mild-to-moderate neutropenia (absolute neutrophil count, 500 to 1500 μ/L) that occurs in 1.5% to 2.0% of patients receiving clozapine. When clozapine is discontinued, recovery occurs within 3 to 7 days, and patients remain asymptomatic. The second type of neutropenia is more severe. It consists of agranulocytosis (absolute neutrophil count less than 500/μL) and has an incidence of 0.8% in patients treated for 1 year. Most cases of agranulocytosis occur within 3 months after the start of treatment. All affected patients are at risk for neutropenic sepsis. There is usually a complete, selective cessation in the bone marrow of neutrophil-precursor production beyond the myeloblast stage. Recovery requires 14 to 22 days; G-CSF therapy has been shown to shorten this duration. Given the patient’s history of clozapine use and the absence of anemia and thrombocytopenia, it is not necessary to perform bone marrow aspiration and biopsy unless her neutropenia fails to improve over the next 2 weeks. Granulocyte transfusions are reserved for patients with uncontrolled infection despite antibiotic therapy and persistent severe neutropenia. Drug-induced neutropenia

Page 109: MKSAP13-Hematology and Oncology

commonly resolves after discontinuation of the drug, and bone marrow transplantation is not warranted. This patient reported no HIV risk factors, and her presentation does not suggest HIV infection.

Hematology and Oncology:Question 5 The correct answer is A Educational Objectives Understand the differential diagnosis of anemia in severe hepatic failure. Critique The patient has spur cell anemia. In a small percentage of patients with alcoholic cirrhosis, erythrocytes accumulate large amounts of free cholesterol with significant erythrocyte membrane distortion. A progressive severe hemolytic anemia ensues, with cells that partly resemble acanthocytes. Unlike true acanthocytes, the cells in spur cell anemia are small and spheroidal with short, spiny projections (hence the term spur cell). Increased amounts of membrane cholesterol lead to increased membrane surface area. Membrane distortion forms the basis of the morphologic presentation secondary to abetalipoproteinemia. The spur cells are

Page 110: MKSAP13-Hematology and Oncology

trapped and destroyed in the splenic cords leading to progressive splenic enlargement. The prognosis of patients with spur cell anemia is extremely poor. Splenectomy is occasionally effective in moderating the hemolysis, but patients usually have underlying portal hypertension and coagulopathies, which complicate the procedure. More often, patients with alcoholic liver disease may have chronic, mild to moderate anemia that is multifactorial (liver disease, hypersplenism, alcohol as a direct toxin to erythroid progenitor cells). Liver disease, particularly extrahepatic biliary obstruction, may be associated with membrane changes such as target cell formation. With an increase in cholesterol, the cholesterol : phospholipid ratio changes, and the cell membrane surface area increases. Thus, the cells become broad and flat. As the cells dry, hemoglobin collects in the thicker areas of the cells (the center and edges) producing a “target’ appearance. Folic acid deficiency should be considered in any patient with alcoholism who has a macrocytic anemia. In this patient, however, the high reticulocyte count makes this diagnosis unlikely. The patient probably has preexisting hypersplenism, based on her diagnosis of cirrhosis, but it would not account for her severe anemia and elevated

Page 111: MKSAP13-Hematology and Oncology

reticulocyte count. Patients who have chronic alcoholism should be maintained prophylactically on folic acid supplementation.

Hematology and Oncology:Question 6 The correct answer is D Educational Objectives Treat symptomatic anemia of chronic disease. Critique The patient has anemia of chronic disease, for which erythropoietin (usually given concurrently with iron) is an effective therapy. His serum iron and total iron-binding capacity are low, but his reticuloendothelial iron stores are adequate, as demonstrated by his elevated serum ferritin level. The clinical picture is characteristic of anemia of chronic disease, which often includes mild microcytosis. If the patient were asymptomatic, it would be appropriate to focus treatment on the osteomyelitis alone. Because the anemia is associated with symptoms, specific therapy for anemia is indicated. Iron therapy by itself is not effective in anemia of chronic disease. Cyanocobalamin is a treatment for vitamin B12 deficiency but not for anemia of chronic disease.

Page 112: MKSAP13-Hematology and Oncology

Hematology and Oncology:Question 7 The correct answer is E Educational Objectives Recognize the presentation of delayed hemolytic transfusion reaction. Critique The patient is having a delayed hemolytic transfusion reaction. Multitransfused and multiparous recipients are at risk for prior erythrocyte alloimmunization. These antibodies are usually detected in the pretransfusion antibody screen by means of reagent cells that carry most clinically significant antigens. The time course and presentation are typical for a delayed hemolytic transfusion reaction arising from an anamnestic response to rechallenge by erythrocyte antigens after previous sensitization. If the antibody titer decays to a subdetectable level, as can happen, the antibody screen or even full crossmatching will be negative using a pretransfusion sample. Conversely, if the alloantibody is specific for a low-frequency antigen not represented on the reagent cells in the antibody screen, pretransfusion testing will not detect the incompatibility unless a full crossmatch is performed. There is usually no problem because most donors are negative for low-frequency antigens. Occasionally, however,

Page 113: MKSAP13-Hematology and Oncology

a patient might receive a low-frequency antigen-positive unit. This situation causes a more acute presentation of the hemolysis, because preformed, but not detected, antibody is present. Failure to keep a unit fully refrigerated can cause erythrocytes to become nonviable; hemolysis occurs during storage from excessive red blood cell metabolic activity. The effects of transfusion with such a unit are seen within 24 hours as the cell stroma and supernatant hemoglobin are cleared and the nonviable cells are destroyed. Transfusion can trigger autoimmune hemolysis, but the fact that this patients direct antiglobulin test was negative makes it unlikely that her symptoms are attributable to autoimmune hemolysis. Glucose-6-phosphate dehydrogenase deficiency is X-linked and rare in females. It causes immediate rather than delayed hemolysis in response to oxidant stress, and does not cause a positive result on a direct antiglobulin test. Errors in ABC compatibility show up immediately rather than after 7 days.

Hematology and Oncology:Question 8 The correct answer is D Educational Objectives Diagnose hereditary hemochromatosis. Critique

Page 114: MKSAP13-Hematology and Oncology

Diagnostic liver biopsy is the procedure of choice, along with a therapeutic phlebotomy program. Although the diagnosis of hereditary hemochromatosis can be made on the basis of homozygosity for the C282Y mutation in the HFE gene, important prognostic information can be obtained from a diagnostic liver biopsy. If the diagnosis of HFE hemochromatosis is made before the development of cirrhosis, then therapeutic phlebotomy to remove excess iron stores and to prevent reaccumulation should enable the patient to have a normal life expectancy. If the diagnosis of hemochromatosis is made after the development of cirrhosis, there is an increased risk for the development of hepatocellular carcinoma even after removal of excess body iron. In general, in a C282Y homozygote, a diagnostic liver biopsy should be recommended if the serum ferritin concentration is greater than 1000 ng/mL or if liver function test results are elevated at the time of diagnosis. Swelling of the first and second metacarpophalangeal joints is a characteristic but not necessary finding in the diagnosis of HFE hemochromatosis. A finding of low amounts of iron in the usual storage sites of bone marrow macrophages is a characteristic of HFE hemochromatosis. It is

Page 115: MKSAP13-Hematology and Oncology

important to proceed with appropriate diagnostic and therapeutic measures; treatment of hemochromatosis with iron removal can arrest organ damage and improve survival.

Hematology and Oncology:Question 9 The correct answer is D Educational Objectives Diagnose myelodysplastic syndrome secondary to chemotherapy and radiation therapy. Critique This patient presents with pancytopenia and infection, most likely pneumonia. Given his history of chemotherapy, radiation therapy, and anemia, the current finding of pancytopenia strongly suggests that he has secondary myelodysplastic syndrome or leukemia. A bone marrow aspiration and biopsy should be performed to confirm the diagnosis. This lethal late complication is seen in 5% of Hodgkins disease survivors treated with multimodality therapy. Most of the myelodysplastic syndromes and acute leukemias occur 3 to 11 years after therapy. Deletions or loss of chromosomes 5 and 7 is commonly associated with alkylating agent therapy. Relapse of Hodgkins disease with bone marrow infiltration and pancytopenia is uncommon and very unlikely 8 years after successful therapy.

Page 116: MKSAP13-Hematology and Oncology

This patients clinical presentation is not consistent with severe sepsis and associated pancytopenia. Hypothyroidism can result in anemia but rarely pancytopenia. There is no history of new drug intake to suggest drug-associated bone marrow suppression.

Hematology and Oncology:Question 10 The correct answer is C Educational Objectives Recall the purpose of blood irradiation prior to transfusion. Critique The sole reason for blood irradiation is to prevent transfusion-related graft-versus-host disease (GVHD) in patients at risk for it. Transfusion-related GVHD has a high mortality rate. Not only are the liver, skin, and gastrointestinal system under immunologic attack, but the recipients bone marrow is also destroyed by the donor T cells. Gamma irradiation of cellular blood products prevents transfusion-related GVHD by rendering donor T cells nonviable and incapable of mounting a GVHD response. The dose used is between 25 and 50 Gy. Higher doses may interfere with the function or survival of red cells and platelets. Leukocyte reduction is not sufficient for this purpose because it leaves behind too many cells.

Page 117: MKSAP13-Hematology and Oncology

Irradiation at this dose has no effect on the transmission of cytomegalovirus (or any pathogen), alloimmune sensitization, or leukocyte antigenicity. Although the cells are nonviable and will not cause a sustained attack against the host, they can still elaborate cytokine. Therefore, irradiation will not prevent febrile reactions. Irradiation has no effect on any of the mechanisms of transfusion-induced hemolysis.

Hematology and Oncology:Question 11 The correct answer is E Educational Objectives Select the appropriate timing for a work-up for a hereditary hypercoagulable state. Critique No testing is necessary; the patient should be started on anticoagulation therapy. Although her family history suggests that she has an underlying hereditary defect predisposing to venous thromboembolism, it is unlikely that she has the factor V Leiden or prothrombin G2021 0A mutation because these abnormalities are not found in native Asian

Page 118: MKSAP13-Hematology and Oncology

populations. It is possible that she has a deficiency of protein C, protein 5, or antithrombin III, but it is not advisable to test for these abnormalities upon presentation with an acute thrombotic event or after the initiation of anticoagulant therapy. It is best to wait at least 2 weeks after anticoagulants are discontinued (upon completion of 3 to 6 months of therapy) to test for these deficiency states. Testing for a lupus anticoagulant is reasonable at presentation, but it should not be done in the presence of heparin; furthermore, her clinical history is more suggestive of a hereditary than an acquired thrombotic disorder (that is, antiphospholipid antibody syndrome). The yield of a search for an occult malignancy in a young woman without any suggestive symptoms or signs would be extremely low.

Hematology and Oncology:Question 12 The correct answer is D Educational Objectives Recognize the clinical presentation and laboratory findings of paroxysmal nocturnal hemoglobinuria Critique Paroxysmal nocturnal hemoglobinuria (PNH) is an uncommon acquired chronic disorder of hematopoietic stem cells. It is characterized by attacks of intravascular hemolysis and

Page 119: MKSAP13-Hematology and Oncology

hemoglobinuria, pancytopenia, and recurrent venous thromboses. An acquired somatic mutation in the PIGA gene results in failure to express phosphatidylinositol-glycan anchor. This defect leads to absence of certain membrane proteins in affected hematopoietic stem cells, including proteins that regulate complement activation, and targets erythrocytes for complement-mediated destruction. A history of dark-colored urine indicative of hemoglobinuria is noted in only 25% of patients. Recurrent abdominal pain is common in PNH and is probably related to portal or mesenteric vein thrombosis. Moderate splenomegaly is common, and the finding helps differentiate this disorder from aplastic anemia. In PNH, laboratory evaluation typically shows severe anemia, leukopenia, and moderate to severe thrombocytopenia. There is evidence of intravascular hemolysis with low or absent haptoglobin, and high serum lactate dehydrogenase levels. There is marked reticulocytosis, although the absolute reticulocyte count is often relatively low for the given degree of anemia. Hemoglobin in the urine is reabsorbed by the proximal convoluted tubules and excreted as hemosiderin. Diagnosis can be made by showing erythrocyte sensitivity to acid-induced hemolysis (Hams

Page 120: MKSAP13-Hematology and Oncology

test) or lack of CD55 and CD59 proteins on erythrocytes demonstrated by flow cytometry. A diagnosis of autoimmune hemolytic anemia would not explain the leukopenia and thrombocytopenia. Aplastic anemia is not associated with hemolysis or splenomegaly. Myelodysplastic syndrome and acute myeloid leukemia are not associated with severe hemolysis as seen in this case and are associated with characteristic peripheral blood smear findings.

Hematology and Oncology:Question 13 The correct answer is D Educational Objectives Recall the principles of ABC compatibility Critique This patients liver disease makes it very likely that his ability to synthesize clotting factor is decreased; therefore, replacement therapy is warranted. Even if clotting factor levels were adequate for hemostasis before transfusion, the levels would have been diluted through replacement of his blood volume. The best choice for clotting factor replacement in this circumstance is group AB fresh frozen plasma. Use of fresh frozen plasma avoids the risk of causing polycythemia. ABC hemagglutinins arise naturally and are therfore expected to be present in plasma. In an emergency when a patient’s ABC type is

Page 121: MKSAP13-Hematology and Oncology

unknown, Group O packed cells have neither A nor B antigens and can be used since they are the “universal donor” and are compatible with all patients’ plasma. Even if a sample drawn for another purpose was found, the blood bank would be unable to use it to determine ABC type unless it were proven to be properly labled and the recipient’s identity had been positively confirmed at the time of draw (U.S. Food and Drug Administration requirements). It is considered safer to use unmatched Group O packed cells rather than risk making a mistake leading to ABC incompatibility. Because of the large amount of donor plasma in whole blood as opposed to packed red cells, it must be group specific, and so group O whole blood can only be given safely to group O recipients. Individuals who are group AB have both A and B antigens on their red cells, and so are incapable of making naturally occurring anti-A or anti-B. Just as group O is the universal donor for red cells, group AB is the universal donor for plasma. Whole blood is a poor choice to treat coagulopathies because it loses labile factors during storage.

Hematology and Oncology:Question 14 The correct answer is B Educational Objectives Diagnose progressive anemia in a patient with HIV infection.

Page 122: MKSAP13-Hematology and Oncology

Critique Human parvovirus B19 can cause severe chronic anemia in patients with HIV infection. Against a background of underlying immune deficiency, human parvovirus is able to infect patients with a particular tropism for erythroid precursors. The infection produces large, multinucleated dysplastic proerythroblasts. The erythrocyte P-antigen is the receptor for the parvovirus. Infection produces intracellular products that are directly toxic to erythroid precursors, leading to a complete cessation of erythropoiesis with associated reticulocytopenia. The diagnosis is confirmed by the demonstration of parvovirus in the serum, blood, or marrow with the use of polymerase chain reaction amplification of DNA. Intravenous immunoglobulin contains antiparvoviral IgG and neutralizes the infection with resolution of the anemia. HIV in general suppresses erythropoiesis and produces a mild anemia that sometimes responds to erythropoietin therapy. HIV infection can produce nonspecific changes sometimes associated with dysplasia, but not distinct from other viral infections. Direct antiglobulin tests are positive in 20% of patients with HIV, but this condition of autoimmunity rarely produces serious anemia, and most patients do not require treatment.

Page 123: MKSAP13-Hematology and Oncology

Drug-induced marrow suppression is common in patients with HIV and is often related to antiretroviral agents as well as drugs such as trimethoprim-sulfamethoxazole. This patient’s rapidly progressive anemia and reticulocytopenia would be inconsistent with marrow suppression from use of trimethoprim-sulfamethoxazole, which is usually mild. Drug-induced glucose-6-phosphate dehydrogenase deficiency and hemolysis represent a possible but unlikely diagnosis in a person of northern European extraction. The relatively unremarkable peripheral blood smear and the “classic” marrow findings make glucose-6-phosphate dehydrogenase hemolysis very improbable.

Hematology and Oncology:Question 15 The correct answer is B Educational Objectives Diagnose and treat severe aplastic anemia. Critique This patient has aplastic anemia, a rapidly fatal disorder in which myeloid progenitor and stem cells in the bone marrow are severely diminished or absent. Either an intrinsic defect of the stem cells or immune-mediated stem cell destruction leads to transfusion-dependent anemia, thrombocytopenia, and severe neutropenia. The bone marrow is typically aplastic or hypoplastic, without evidence of

Page 124: MKSAP13-Hematology and Oncology

infiltrating malignant cells. The differential diagnosis includes acute leukemia (particularly hypoplastic leukemia), myelodysplastic syndromes, and drug-induced or nutritional cytopenias. Allogeneic stem cell transplantation is the treatment of choice, particularly for patients younger than 50 years with HLA-identical sibling donors. Transplantation can result in long-term survival of 75% to 90%. Intravenous immunoglobulin is useful in pure red cell aplasia due to parvovirus B19 infection. Parvovirus infection does not cause severe anemia in healthy individuals who can mount an antibody response, and it does not affect leukocyte and platelet counts. Melphalan and prednisone are effective in multiple myeloma. This patient does not have a plasma cell disorder. It is common to see relative abundance of plasma cells in severely hypoplastic bone marrow of patients with aplastic anemia. Immunosuppression with antithymocyte globulin and cyclosporine can induce responses but is usually reserved for patients who do not have a suitable donor for allogeneic stem cell transplantation or who are too old for this procedure. Long-term follow-up of patients treated with immunosuppressive therapy shows an increased incidence of

Page 125: MKSAP13-Hematology and Oncology

myelodysplastic syndrome and leukemia, making this option less attractive for young patients. Long-term transfusion therapy will not correct the severe neutropenia and risk of life-threatening infections.

Hematology and Oncology:Question 16 The correct answer is C Educational Objectives Recognize dilutional coagulopathy as a consequence of massive transfusion. Critique This patient most likely has dilutional coagulopathy as a result of the massive transfusion with packed red blood cells and crystalloid. Clinical bleeding during massive transfusion is almost always the consequence of depletion of clotting factors through blood loss. It can also be a function of dilution by intravenous fluid and banked blood units that lack clotting factors. The best therapy is administration of fresh frozen plasma to aid in restoration of normal coagulation ability. Platelet concentrates would not be appropriate for this patient; low platelet counts are rarely a cause of bleeding, especially when a dilutional problem is involved, as in this case. After more than three blood volumes have been replaced, however, severe thrombocytopenia can result. When severe thrombocytopenia occurs earlier,

Page 126: MKSAP13-Hematology and Oncology

it is often a sign of disseminated intravascular coagulation or tissue ischemia causing a superimposed consumptive coagulopathy. Whole blood (which is almost never available because it is needed for component manufacture) would not help with hemostasis. The plasma in whole blood loses labile clotting factors during storage, and the platelets become nonviable rapidly during refrigerated storage. Cryoprecipitate is devoid of most coagulation factors and serves as a concentrated source of fibrinogen, factor VIII, and a few other cold-insoluble proteins. The development of more purified products to treat hemophilia and von Willebrand’s disease has obviated the use of cryoprecipitate as a stand-alone agent to treat bleeding. Its major use is as an adjunct to fresh frozen plasma when additional fibrinogen is required. Cryoprecipitate can also be used as a fibrinogen source in the making of fibrin glue used as a biologic sealant, but in this application it is used chiefly as a patch for serosal or other surface tears rather than to stop diffuse bleeding.

Hematology and Oncology:Question 17 The correct answer is C Educational Objectives Diagnose and treat chemotherapy-induced anemia and fatigue.

Page 127: MKSAP13-Hematology and Oncology

Critique Weekly administration of erythropoietin would be appropriate for this patient. Approximately 75% of patients being treated for cancer experience fatigue. In most of these patients, the fatigue is not treated, probably because it is attributed to the underlying malignancy or its treatment. Compared with placebo, therapy with erythopoietin and longer-acting darbepoetin has been shown to decrease transfusion requirements, increase hemoglobin levels, and improve quality of life. Response to erythropoietin therapy usually occurs within 3 to 5 weeks, and if there is no response by 6 weeks, it is unlikely that the patient will benefit from further therapy. Granulocyte colony-stimulating factor stimulates granulocytic precursors in the bone marrow and does not affect hemoglobin levels. Although cessation of chemotherapy would eventually allow recovery from anemia, it is not recommended because of the risk that the cancer would progress. Administration of androgens plays no proven role in the treatment of fatigue associated with chemotherapy-induced anemia. Vitamin E supplements have not been shown to improve anemia and fatigue related to cancer chemotherapy.

Page 128: MKSAP13-Hematology and Oncology

Hematology and Oncology:Question 18 The correct answer is D Educational Objectives Understand the use of platelet function tests as screening tests for surgical bleeding. Critique It is never possible to guarantee that any given patient will not experience a bleeding complication. In most cases, however, these episodes reflect surgical lesions rather than an underlying predisposition to bleeding. A thorough patient history, focusing particularly on previous surgery, dental procedures, menstrual bleeding, and bleeding with prior trauma, remains the best method of detecting a patient who may be at risk for increased bleeding during surgery. The bleeding time might be a useful test when screening for inherited or acquired deficiencies in platelet function in a patient with an appropriate history. It cannot, however, predict surgical bleeding when used to screen unselected surgical patients. The Platelet Function Analyzer-100 (PFA-100) may be more sensitive and perhaps more specific in the diagnosis of platelet disorders and von Willebrand’s disease than bleeding time; it has not, however, been validated as a screening test for surgical bleeding in unselected patients. Therefore, in this regard it should not

Page 129: MKSAP13-Hematology and Oncology

be considered superior to the bleeding time. Platelet aggregation studies should be done only after a screening test such as the bleeding time or the PFA-i 00 analysis suggests an underlying platelet disorder. These studies are expensive and have not been established as preoperative screening tests. The prothrombin time and partial thromboplastin time may be prolonged in hemophilia A or B or in deficiencies of other intrinsic pathway factors such as factor XI or factor XII. They can, however, be normal in von Willebrand’s disease or in moderate deficiencies of factors VIII or IX that are sufficient for normal hemostasis but may nonetheless predispose a patient to surgical bleeding. Therefore, the partial thromboplastin time is not a useful presurgical screening test (although it has been suggested that the partial thromboplastin time be used to screen patients of Ashkenazi Jewish descent because of the high incidence of factor Xl deficiency in this population).

Hematology and Oncology:Question 19 The correct answer is B Educational Objectives Diagnose anemia with low serum iron. Critique There are two main reasons patients do not respond to oral iron therapy: Either they are

Page 130: MKSAP13-Hematology and Oncology

noncompliant, or they are not iron-deficient. A low serum iron level by itself does not distinguish iron deficiency from anemia of chronic disease. At the very least, the serum ferritin level must also be determined. The history and examination findings largely rule out noncompliance with treatment. Therefore, the physician should order a serum ferritin test to correctly characterize the anemia syndrome. Administration of an oral iron polysaccharide or possibly parenteral iron or addition of ascorbic acid would be correct if the patient were found to be iron-deficient.

Hematology and Oncology:Question 20 The correct answer is B Educational Objectives Diagnose leukoerythroblastosis due to tuberculosis. Critique The patient has fever, anemia, and a leukoerythroblastic blood smear characterized by immature myeloid cells and the presence of nucleated erythrocytes and teardrop forms. The differential diagnosis includes metastatic cancer, myelofibrosis with myeloid metaplasia, severe stress (blood loss, hemolysis, infection), and miliary tuberculosis. Distortion of the marrow sinuses causes deformity of the erythrocytes and also allows transit of immature myeloid and erythroid marrow cells

Page 131: MKSAP13-Hematology and Oncology

into the peripheral blood. In this patient, the history of exposure to tuberculosis, the recent use of corticosteroids as an immunosuppressive agent, and the anergy noted by skin testing should raise a very high degree of suspicion of disseminated tuberculosis. Nearly 90% of patients with miliary tuberculosis have anemia. Although infiltrative myelopathies usually are caused by metastatic cancer, tuberculosis may be implicated in as many as 5% of cases. Tuberculosis is increasingly recognized in elderly patients who lack pulmonary symptoms, who have been exposed to corticosteroids or other immunosuppressive agents, and whose findings slowly evolve toward a picture consistent with an infiltrative myelopathy. In this patient the easiest, safest approach to confirm the diagnosis is bone marrow biopsy with culture, which yields a diagnosis in 80% of patients with miliary tuberculosis. The demonstration of caseating granulomas is strong evidence for tuberculosis or histoplasmosis. Cultures of the marrow would likely grow acid-fast bacilli. In the presence of abnormal liver function studies, liver biopsy probably would yield a diagnosis but has a higher procedure risk profile in an elderly patient compared with bone marrow biopsy.

Page 132: MKSAP13-Hematology and Oncology

Similarly, although open-lung biopsy would also yield a diagnosis, it is very invasive. Splenomegaly is consistent with tuberculosis, and splenectomy would not be indicated because preservation of splenic function is desirable. Aspiration of an abdominal lymph node likewise could yield a diagnosis of tuberculosis, but the node is small and could be difficult to access. Furthermore, the risk of complicating infection or bleeding would be greater than in bone marrow biopsy. Since bronchoscopy with bronchoalveolar lavage yields a diagnosis of tuberculosis in only 30% of patients with miliary disease, an open lung biopsy or a transbronchial biopsy would be appropriate next steps if the marrow biopsy did not reveal a diagnosis.

Hematology and Oncology:Question 21 The correct answer is A Educational Objectives Recognize the need for prompt diagnosis of red cell aplasia in sickle cell disease. Critique Infection with parvovirus B19 is a common cause of transient reticulocytopenia in humans. Patients with sickle cell anemia or thalassemia compensate for their chronic hemolytic state through elevated erythropoietic activity and can rapidly develop life-threatening anemias in the face of

Page 133: MKSAP13-Hematology and Oncology

parvovirus-mediated erythrocyte hypoproliferation. Although the formation of alloantibodies can accelerate hemolysis of transfused blood, they do not adversely affect marrow erythropoiesis. From a practical perspective, donor erythrocytes will have been cleared from the circulation long before the 6-month anniversary of the patients last transfusion. Rarely, profound nutritional disorders can cause marrow aplasia, but the presenting symptoms of severe folate deficiency would be megaloblastic, macrocytic anemia. Although splenic sequestration crisis can cause a rapid fall in hemoglobin levels, compensatory reticulocytosis is typically maintained.

Hematology and Oncology:Question 22 The correct answer is E Educational Objectives Diagnose and treat heparin-associated thrombocytopenia. Critique The best approach to management of this patient would involve discontinuation of heparin therapy followed immediately by the institution of alternative anticoagulation therapy such as lepirudin or argatroban. This patient with presumptive exposure to heparin during prior peripheral vascular surgery develops acute thrombocytopenia 2

Page 134: MKSAP13-Hematology and Oncology

days after being exposed to heparin. There are no other obvious causes for thrombocytopenia, and a clinical diagnosis of heparin-induced thrombocytopenia should be made. After the diagnosis is made, it is essential to discontinue all heparin exposure immediately. Even when the platelet count has not reached levels below 150,000/μL (or the lower limit of normal for the specific laboratory), an acute decrease in the platelet count in the setting of ongoing heparin exposure should signal the need to discontinue heparin therapy. Continuation of exposure to heparin can cause life-threatening cardiovascular thrombotic events. Therefore, continuing the heparin infusion regardless of close monitoring or in anticipation of diagnosing the disease on the basis of laboratory studies is not an acceptable alternative. Although antiplatelet therapy may have value in the treatment of heparin-induced thrombocytopenia, it is not acceptable first-line therapy and initiating it does not make it safe to continue the heparin therapy. Moreover, recent studies suggest that patients with heparin-induced thrombocytopenia have a hypercoagulable state associated with a significant risk of thrombosis that may persist for days to weeks after heparin is discontinued. To address this risk, particularly in this patient

Page 135: MKSAP13-Hematology and Oncology

in need of ongoing anticoagulation for cardiac ischemia, discontinuing heparin as the only intervention is not sufficient.

Hematology and Oncology:Question 23 The correct answer is A Educational Objectives Recognize the need for compatibility blood samples in patients requiring multiple transufsions.. Critique The blood bank wants a new sample because this patient’s recent transfusion may have stimulated new alloantibody specificities to emerge that would not be present in older samples. The patient’s history marks her as an antibody responder, and there is a higher likelihood that she will make further antibodies upon continued challenge. Although 4 days is not sufficient time for primary alloimmunization to proceed to detectable levels of antibody, studies have shown that anamnestic reactions can produce clinically detectable antibodies in that short a time; hence the ‘3-day rule’ for compatibility samples. There is no reason to suspect hemolysis, because of the appropriate rise in hemoglobin. Antibody screening does not include a direct antiglobulin test, which is the preferred test to identify an ongoing hemolytic reaction. It is common to use group O cells when antigen-

Page 136: MKSAP13-Hematology and Oncology

negative units are required, in view of the universal compatibility of group O blood. In the absence of a hemolytic reaction, transfused group O cells are expected to be detectable in circulation after only 3 days, but there is no reason to look for them. Autoantibodies may be present, but compatibility-testing methods were not designed to screen for them. Rather, they are used to identify homologous units that have a high likelihood of surviving as long as the autologous cells in circulation. Even though autoantibodies can be detected in the antibody screen, the purpose of this testing is to detect alloantibodies. If an alloantibody had been missed during compatibility testing, it would be important to look for it only if hemolysis were suspected.

Hematology and Oncology:Question 24 The correct answer is D Educational Objectives Differentiate between iron deficiency anemia and anemia of chronic disease. Critique In this context, staining a bone marrow aspirate for iron is the gold standard for diagnosing iron deficiency anemia. Anemia with a low serum iron concentration is characteristic of the anemia of chronic disease and of iron deficiency anemia. The anemia of

Page 137: MKSAP13-Hematology and Oncology

chronic disease is also typically associated with low serum transferrin concentration and normal to elevated serum ferritin concentration, whereas iron deficiency anemia is also typically associated with elevated transferrin concentration and low serum ferritin concentration. When iron deficiency occurs in the setting of inflammation, the serum ferritin concentration may be normal because ferritin is an acute- and chronic-phase reactant. Findings consistent with but not diagnostic of iron deficiency in this patient are low mean corpuscular volume, increased red cell distribution width, and high normal to elevated serum transferrin concentration. The finding of iron deficiency should prompt a search for blood loss, and endoscopy of the gastrointestinal tract should be considered in this regard. Treatment with iron without identifying the source of blood loss could delay the diagnosis of an otherwise curable gastrointestinal malignancy. Furthermore, iron therapy should not be given to a patient with HIV disease in the absence of iron deficiency, because iron therapy can promote certain infections and toxicities in HIV disease. Erythropoietin can be used to treat anemia in patients with HIV disease but should generally be considered only after other causes of

Page 138: MKSAP13-Hematology and Oncology

anemia have been ruled out and the serum erythropoietin level is found to be less than 500 IU/mL.

Hematology and Oncology:Question 25 The correct answer is B Educational Objectives Diagnose and treat relapsed lymphoma. Critique Relapse should be documented by a lymph node biopsy, and staging should be completed with bone marrow aspiration and biopsy. Relapse occurs in approximately 50% to 60% of patients with advanced-stage (II-IV) aggressive non-Hodgkin’s lymphoma treated with cyclophosphamide, hydroxydaunomycin, vincristine, and prednisone. Systemic symptoms, findings on physical examination and CT scan, and the elevated serum lactate dehydrogenase level are all consistent with relapsed lymphoma in this patient. Most patients with relapsed lymphoma respond to second-line salvage chemotherapy. In these chemotherapy-sensitive patients, autologous stem cell transplantation has been shown to be superior to salvage chemotherapy alone; 40% to 50% of patients who underwent transplantation had long-term disease-free survival. By contrast with the indolent lymphomas, relapsed aggressive lymphomas have a

Page 139: MKSAP13-Hematology and Oncology

progressive clinical course, and treatment should not be delayed. Because there are numerous effective second-line chemotherapy regimens for this patient, a trial of investigational agents is not appropriate at this time. Repeating a course of CHOP is not recommended because of the increased risk of doxorubicin-induced cardiotoxicity and the availability of more active salvage regimens.

Hematology and Oncology:Question 26 The correct answer is D Educational Objectives Recognize the increased risk of venous thromboembolism associated with hormone replacement therapy, particularly in an asymptomatic carrier with an inherited thrombotic defect. Critique Not only should this patient be advised against the use of hormone replacement therapy (HRT), because of her history, she should use anticoagulant prophylaxis at times of increased risk. HRT has been shown to increase the risk of venous thromboembolism, and the risk is compounded in patients with an underlying hereditary thrombotic defect. Data from the Women’s Health Initiative indicate that HRT is also not beneficial in reducing the risk of arterial thrombotic complications. Although warfarin therapy is likely to be highly

Page 140: MKSAP13-Hematology and Oncology

effective in reducing the risk of venous thromboembolism, it is associated with a risk of major bleeding and for that reason is contraindicated. Aspirin provides minimal if any protection against the development of venous thrombosis.

Hematology and Oncology:Question 27 The correct answer is A Educational Objectives Recognize and treat drug-induced thrombocytopenia. Critique Determining the cause of hematologic abnormalities may be difficult in transplant recipients. Trimethoprim-sulfamethoxazole is a leading cause of drug-induced thrombocytopenia, and the most likely cause of thrombocytopenia in this patient. The thrombocytopenia usually occurs acutely, soon after beginning the drug, but may also develop insidiously. Discontinuation of trimethoprim-sulfamethoxazole should lead to improvement in the platelet count within 7 to 14 days in most cases—if improvement does not occur, other diagnoses should be considered. Because the hemoglobin level and leukocyte count are normal, and the peripheral blood smear shows only decreased platelets, the likelihood that thrombocytopenia is the result

Page 141: MKSAP13-Hematology and Oncology

of a primary or secondary marrow process is low and bone marrow examination is not required. Platelet transfusion would be indicated if the platelet count were lower than 10,000/μL or if there were active bleeding. Epsilon-aminocaproic acid is an antifibrinolytic drug that is sometimes of use in patients with thrombocytopenic bleeding but is not indicated in this patient. Immunosuppressive therapy should not be discontinued because doing so would risk the viability of the allograft. If the peripheral blood smear revealed a microangiopathic process, as demonstrated by the presence of schistocytes, then changing the immunosuppressive regimen or decreasing the doses of cyclosporine and/or tacrolimus should be considered, as these agents are associated with the development of post-transplant microangiopathy.

Hematology and Oncology:Question 28 The correct answer is A Educational Objectives Recognize the increased risk posed by a prior episode of thrombosis and its implications for pregnancy. Critique The best management would be to observe her carefully throughout her pregnancy and administer prophylactic low-molecular-weight heparin for 6 weeks after she gives birth. The

Page 142: MKSAP13-Hematology and Oncology

use of oral contraceptives confers a fourfold increase in the risk of venous thromboembolism, but the risk of recurrence is quite low in women with prior thrombosis in association with just such a transient risk factor. The study from Brill-Edwards et al. reported that the antepartum risk of venous thrombosis was extremely low in pregnant women with a prior thrombotic event in association with transient risk factors. Therefore observation rather than antepartum administration of low-molecular-weight heparin is appropriate for this patient. The postpartum period is associated with a higher risk of thrombosis, and anticoagulation is generally recommended for a period of 6 weeks. The hereditary thrombophilias, especially factor V Leiden mutation, have been associated with a threefold increase of stillbirth in the second and third trimester, but it remains a matter of controversy whether other adverse obstetric outcomes such as preeclampsia and intrauterine growth retardation are associated findings. Termination of pregnancy would therefore not be appropriate. Hematology and Oncology:Question 29 The correct answer is E Educational Objectives

Page 143: MKSAP13-Hematology and Oncology

Recognize when to initiate therapy for idiopathic (immune) thrombocytopenic purpura. Critique The correct approach to management of this patient, who has asymptomatic thrombocytopenia, is to monitor her platelet count closely and intervene therapeutically if the platelet count drops further. The differential diagnosis includes hypoproliferative disorders such as myelodysplasia, congenital thrombocytopenias such as the May-Hegglin anomaly, or consumption disorders such as thrombotic microangiopathies, disseminated intravascular coagulation, or idiopathic thrombocytopenic purpura. The most likely disorder in a patient of this age who is otherwise healthy and in whom the other hematologic parameters are normal is idiopathic (immune) thrombocytopenic purpura. Most experts would not recommend instituting therapy for idiopathic thrombocytopenic purpura in patients with a platelet count above 30,000/μL in the absence of bleeding. A bone marrow aspiration with biopsy is not required for the diagnosis of idiopathic thrombocytopenic purpura and is unlikely to be of value in an otherwise healthy young woman with no hematologic abnormalities. Platelet

Page 144: MKSAP13-Hematology and Oncology

aggregation studies are likely to be abnormal simply because of her thrombocytopenia; they are used to investigate potential causes of platelet dysfunction rather than abnormalities in platelet number. Genetic causes of thrombocytopenia are usually diagnosed in childhood and would be uncommon as a first diagnosis in a 27-year-old with no suggestive family history. Moreover, most congenital thrombocytopenias are associated with platelet morphologic abnormalities that were not observed in this case. Therefore, there is no reason to study the patient’s relatives.

Hematology and Oncology:Question 30 The correct answer is B Educational Objectives Diagnose and treat polycythemia vera. Critique This patient has polycythemia vera; therefore, phlebotomy is the treatment of choice. Polycythemia vera is characterized by an increased red cell mass; mild elevation in leukocyte and platelet counts is also common. Clinical symptoms are the result of hyperviscosity, hypervolemia, and hypermetabolism. Headache, pruritus, dyspnea, blurred vision, and night sweats are common. Patients usually have facial plethora and splenomegaly. Bleeding and thromboembolic events (arterial

Page 145: MKSAP13-Hematology and Oncology

and venous) represent potentially life-threatening complications of this disease. Elevated red cell mass (or hematocrit values above 60% for men and 56% for women) in the absence of secondary causes of erythrocytosis, and the presence of splenomegaly or an abnormal marrow karyotype other than the Philadelphia chromosome establish the diagnosis of polycythemia vera. Regular phlebotomy to maintain the hematocrit at less than 45% promptly controls the symptoms of the disease, but the thrombotic risk remains elevated. Splenectomy is reserved for the late stages of the disease, for palliation of symptomatic splenomegaly with hypersplenism. Induction chemotherapy with cytarabine and idarubicin does not play a role in this disease unless there is progression to leukemia. Alcohol use is not associated with polycythemia and abstaining would not affect the course of the disease. Imatinib mesylate is specific therapy for chronic myeloid leukemia and is not indicated in polycythemia vera. Hematology and Oncology:Question 31 The correct answer is C Educational Objectives Recognize and manage gestational thrombocytopenia. Critique

Page 146: MKSAP13-Hematology and Oncology

The major differential diagnosis in this patient is between idiopathic (immune) thrombocytopenic purpura and gestational thrombocytopenia. Although idiopathic thrombocytopenic purpura cannot be excluded with absolute certainty, the most common cause of mild, asymptomatic thrombocytopenia in pregnancy, particularly when it develops in the third trimester, is incidental (gestational) thrombocytopenia. The pathogenesis of this disorder is not well understood, but it is not associated with adverse maternal or fetal outcomes and therefore does not require specific intervention. If the platelet count drops below 70,000/μL or other pregnancy-related complications occur, other diagnoses should be considered. This patient presents with mild thrombocytopenia in the third trimester of an otherwise uncomplicated pregnancy. There is no personal or family history of prior hematologic disease, nor is there an elevated blood pressure that would suggest preeclampsia. The peripheral blood smear does not reveal schistocytes; therefore there is no evidence of an underlying microangiopathic hemolytic anemia. The clinical history and physical examination are unremarkable; therefore there is also no evidence of an underlying disorder such as sepsis, which

Page 147: MKSAP13-Hematology and Oncology

would predispose the patient to the development of disseminated intravascular coagulation. The patient is taking no medications that are associated with drug-induced thrombocytopenia.

Hematology and Oncology:Question 32 The correct answer is B Educational Objectives Recall the purpose of leukocyte reduction. Critique Donor leukocytes can act as immunizing antigens and are the chief cause of human leukocyte antigen (HLA) alloimmunization. This patient’s history argues against existing HLA sensitization through pregnancy or transfusion. Prevention of primary alloimmunization to HLA antigens can be achieved by leukocyte reduction. Success in finding a compatible cadaveric organ that will not be rejected acutely through humoral mechanisms is directly related to the presence of HLA class I antibodies. Prevention of HLA alloimmunization is critical to her chance of obtaining an organ and stopping dialysis. Many adverse reactions to transfusions are related to the presence of donor leukocytes in blood products. These cells can harbor pathogens such as cytomegalovirus or HTLV-1, which are not transmitted through cell-free plasma. The patient is cytomegalovirus-

Page 148: MKSAP13-Hematology and Oncology

positive herself; therefore transmission of the virus is not a concern. She is at risk for reactivation of her own virus during immune suppression after transplant. Current technology for leukoreduction does not remove enough T cells to prevent graft-versus-host disease. Prestorage leukocyte reduction eliminates donor-derived cytokine elaboration that causes febrile non hemolytic reactions; the actual incidence of these reactions is less than 5% of transfusions. None of these effects is as critical to her health as the prevention of HLA alloimmunization. Leukocyte reduction has no effect on erythrocyte alloimmunization or hemolytic reactions.

Hematology and Oncology:Question 33 The correct answer is D Educational Objectives Understand the features of glucose-6-phosphate dehydrogenase deficiency. Critique The patient has glucose-6-phosphate dehydrogenase (G6PD) deficiency and acute, drug-induced hemolytic anemia. G6PD protects cells from membrane oxidation. In deficient persons, membrane oxidation causes cells to become rigid and susceptible to mechanical

Page 149: MKSAP13-Hematology and Oncology

hemolysis and trapping by the spleen and liver. In this patient the occurrence of acute intravascular and extravascular hemolysis following administration of primaquine and the presence of Heinz bodies and characteristic bite cells on peripheral blood smear make this diagnosis very likely. The presence of a leukoerythroblastic blood smear is consistent with the stress of acute hemolysis and not necessarily a marrow infiltrative process. The absence of hemoglobinuria does not necessarily exclude an acute intravascular hemolytic episode. Because hemoglobin will be present only in the immediate period of the hemolysis, the detection of urine hemosiderin may indicate a more remote prior episode. Hemoglobin is absorbed into the proximal renal tubules where it is degraded ultimately into hemosiderin, which spills slowly into the urine. Sensitive screening tests are available for G6PD deficiency but may be abnormal in other hexose monophosphate shunt deficiencies. The detection of low G6PD enzyme activity during an acute hemolytic crisis is often futile because the erythrocytes most susceptible to destruction are those with the lowest enzyme activity (the younger cells have greater enzyme activity and therefore survive). Primaquine is one of a number of drugs that

Page 150: MKSAP13-Hematology and Oncology

produce acute hemolysis in G6PD-deficient subjects.

Hematology and Oncology:Question 34 The correct answer is A Educational Objectives Avoid needless radiologic evaluation for an occult neoplasm in an older patient with a first venous thrombotic event. Critique No further evaluation is necessary. An unprovoked episode of deep venous thrombosis in older patients has been associated with an increased incidence of an occult malignancy. An extensive search for an occult neoplasm in otherwise healthy patients does not always detect a treatable malignancy or is not cost-effective. Although the factor V Leiden and prothrombin G2021 0A mutations are frequently present in older patients with a first deep venous thrombosis in the absence of a family history, deficiencies of antithrombin III, protein C, or protein S are very rarely encountered in this setting, and these tests are not recommended.

Hematology and Oncology:Question 35 The correct answer is C Educational Objectives Recognize the limitations of laboratory studies in the diagnosis of thrombotic

Page 151: MKSAP13-Hematology and Oncology

microangiopathy. Critique This patient presents with signs and symptoms consistent with thrombotic thrombocytopenic purpura. The diagnosis of thrombotic thrombocytopenic purpura is a clinical one, and microangiopathic hemolytic anemia and an elevated serum lactate dehydrogenase level without another apparent cause should be sufficient evidence of thrombotic thrombocytopenic purpura to institute plasma exchange therapy. Thrombotic thrombocytopenic purpura primarily affects the microvasculature, and brain ischemia, although diffuse, does not generally lead to the development of lesions of sufficient size to be detected by CT; therefore, CT of the head is often normal. Liver function studies may also be normal in these patients and are not helpful in reaching a diagnosis. Although functional deficiency of the vWF-cleaving protease, ADAMTS 13, is common in patients with thrombotic thrombocytopenic purpura, diagnosis and therapy of this disorder should be made on clinical criteria. The utility of functional ADAMTS 13 assays in diagnosing and monitoring the therapy of this disorder has not been established. Continued assay improvement and correlation with clinical disease in humans is required before assays of

Page 152: MKSAP13-Hematology and Oncology

ADAMTS 13 are accepted for use in clinical decision making.

Hematology and Oncology:Question 36 The correct answer is B Educational Objectives Diagnose chronic myeloid leukemia in a patient presenting with neutrophilia. Critique Typically, chronic myeloid leukemia (CML) is diagnosed on a routine blood count that demonstrates leukocytosis with circulating myeloid precursors. Patients may have fatigue, weight loss, splenomegaly, and thrombocytosis. A balanced translocation between chromosomes 9 and 22 (Philadelphia chromosome) creates a unique gene, bcr-abl, which codes a 210-kDa protein (p210) that functions as a tyrosine kinase. This kinase was shown to cause a CML-like disease in mice when expressed in hematopoietic cells. The Philadelphia chromosome can be detected in peripheral blood mononuclear cells by interphase fluorescence in situ hybridization (FISH) analysis and is diagnostic of CML. This patients abdominal symptoms can be explained by his splenomegaly. Although significant neutrophilia can be associated with occult tumors, the lack of constitutional symptoms and weight loss and absence of anemia and occult blood in the stool make an

Page 153: MKSAP13-Hematology and Oncology

occult gastrointestinal malignancy very unlikely. CT of the abdomen is not warranted in this case. Leukemoid reaction attributable to infection is unlikely in this patient because he does not have fever and lacks other manifestations of infection. Acute bacterial endocarditis can present with neutrophilia with a left shift but is commonly associated with fever and constitutional symptoms that are lacking in this case. Therefore blood cultures and echocardiography are not necessary. The leukocyte alkaline phosphatase score is often low in chronic myeloid leukemia, but a low score is not diagnostic of CML. It may also be low in other hematologic disorders, such as paroxysmal nocturnal hemoglobinuria, hypophosphatasia, myelofibrosis, and acute myeloid leukemia.

Hematology and Oncology:Question 37 The correct answer is D Educational Objectives Diagnose heparin-induced thrombocytopenia. Critique This most likely cause of this patient’s thrombocytopenia is heparin-induced thrombocytopenia. Extracorporeal circulation during coronary artery bypass grafting induces transient thrombocytopenia, but it is uncommon for the platelet count to drop by

Page 154: MKSAP13-Hematology and Oncology

more than 25% because of bypass alone. Furthermore, platelet counts generally normalize within 3 to 4 days after surgery. Therefore, surgery alone cannot account for this persistent and severe thrombocytopenia. Although thrombotic thrombocytopenic purpura may on occasion occur soon after bypass surgery, the fact that the peripheral blood smear does not reveal increased schistocytes excludes this diagnosis. Idiopathic thrombocytopenic purpura cannot be entirely excluded but does not usually develop so rapidly and would be less common than heparin-induced thrombocytopenia in an elderly patient in this setting. Disseminated intravascular coagulation is relatively unlikely, given the patient’s normal prothrombin time and partial thromboplastin time, and no obvious primary disorder that would stimulate its development. It is likely that the patient developed heparin-induced thrombocytopenia at or immediately after the time of her surgical procedure. The platelet count might actually have been improving at the time of her read mission. At such a time she would be expected to be particularly susceptible to recurrent exposure to heparin, as her antibody levels to heparin would be maximal. The role of heparin-induced thrombocytopenia in her persistent

Page 155: MKSAP13-Hematology and Oncology

thrombocytopenia is supported by the precipitous decline in the platelet count and development of lower-extremity ischemia after reexposure to heparin.

Hematology and Oncology:Question 38 The correct answer is B Educational Objectives Distinguish among different causes of hypochromic microcytic anemia. Critique Anemia can result from any of the listed conditions, although the anemias associated with uncomplicated glucose-6-phosphate dehydrogenase deficiency and sickle cell disease are typically normocytic and normochromic. By contrast, the anemias of iron deficiency, α-thalassemia, and β-thalassemia, are characteristically both microcytic and hypochromic (low mean corpuscular volume and mean corpuscular hemoglobin values). Iron deficiency anemia would be unusual in an otherwise well-appearing young man. Furthermore, it would be associated with a hypoproliferative state (low reticulocyte count) and is not a recognized cause of elevated hemoglobin A2 levels (in fact, in patients with β-thalassemia who are also iron deficient, elevated hemoglobin A2 can be reduced into the normal range). Iron deficiency could be ruled out by normal plasma iron studies or the

Page 156: MKSAP13-Hematology and Oncology

presence of stainable iron on bone marrow biopsy. Patients with mild α- or β-thalassemia appear well, and the spleen is often not palpable. They have mild elevations in reticulocyte count consistent with their chronic hemolytic disorder. An elevation in the hemoglobin A2 level is consistent with β-thalassemia and is one of the chief methods for distinguishing this disorder from mild ct-thalassemia. Glucose-6-phosphate dehydrogenase deficiency would not account for the patient’s signs and symptoms.

Hematology and Oncology:Question 39 The correct answer is C Educational Objectives Diagnose delayed-onset heparin-induced thrombocytopenia. Critique This patient developed severe thrombocytopenia and an arterial thrombotic event 2 weeks after a hospitalization in which she was given heparin. She has delayed-onset heparin-induced thrombocytopenia. This syndrome is associated with the development of anti-heparin-platelet factor 4 antibodies, as well as thrombocytopenia and thrombosis common to standard presentations of heparin-induced thrombocytopenia. In one study, the mean onset of thrombocytopenia and

Page 157: MKSAP13-Hematology and Oncology

thrombosis was 9.2 days after withdrawal of all heparin; in some patients, it did not occur until 19 days afterward. The presence of this disorder could be confirmed by measuring heparin-platelet factor 4 antibodies using the 14C-serotonin release assay. The pathogenesis of this disorder is not well understood; heparin is no longer present when the disease develops. It is assumed that the pathologic antibodies recognize conformationally altered platelet factor 4 bound to the platelet surface or to endogenous glycosaminoglycans expressed on the surface of endothelial cells. Although there are not many studies of the therapy of this disorder, patients have responded to alternative anticoagulation in a manner similar to those with more standard presentations of heparin-induced thrombocytopenia. Genetic assays for factor V Leiden or prothrombin G2021 0A mutation would not account for the patients thrombocytopenia. Moreover, these genetic defects are associated with venous rather than arterial thrombosis. Platelet aggregation studies would be unrevealing, as they would not test platelet function in the presence of heparin. Measurements of platelet-associated immunoglobulin G are nonspecific and would not help identify the patient’s disorder.

Page 158: MKSAP13-Hematology and Oncology

Hematology and Oncology:Question 40 The correct answer is D Educational Objectives Recall the risks of viral transmission through transfusion. Critique Hepatitis B poses the highest risk. With the combination of donor health screening, confidential unit exclusion, and highly sensitive testing, the risk of HIV transmission from asymptomatic donors is negligible. The risk of hepatitis C transmission is also extremely low, despite a relatively high incidence (0.5%) in the general first-time donor population. One of the reasons these two viruses have such a low risk is the use of nucleic acid testing, which relies on polymerase chain reaction technology to amplify viral material. Unfortunately, technical difficulties have delayed nucleic acid testing for hepatitis B. Although testing for hepatitis B surface antigen was initiated in the late 1970s and has been constantly refined and improved, there are still donors who are antigen-negative but capable of transmitting the virus. Screening for hepatitis B core antibody can identify some but not all of these donors.

Hematology and Oncology:Question 41 The correct answer is C Educational Objectives

Page 159: MKSAP13-Hematology and Oncology

Administer antithrombotic therapy to a patient with antiphospholipid antibody syndrome and transient cerebral ischemia. Critique Lupus anticoagulants are acquired defects that are associated with arterial as well as venous thrombotic events. The diagnosis of antiphospholipid antibody syndrome can be made in this patient, on the basis of clinical findings of transient cerebral ischemic episodes in association with the presence of a lupus anticoagulant and a high level of cardiolipin IgG antibodies. Retrospective studies, which comprise for the most part patients with antiphospholipid antibody syndrome and underlying collagen-vascular disease, indicate that warfarin to a target INR greater than 3 is required to prevent recurrent thrombotic events. Recent data however indicate that an INR of 2 to 3 is adequate for preventing recurrent venous thrombosis. Antiplatelet therapy alone does not adequately protect patients with antiphospholipid syndrome against arterial thrombosis.

Hematology and Oncology:Question 42 The correct answer is A Educational Objectives Diagnose and treat warm antibody autoimmune hemolytic anemia. Critique

Page 160: MKSAP13-Hematology and Oncology

This patient has warm antibody autoimmune hemolytic anemia. Her history is typical for the disease, and her physical findings of jaundice and mild splenomegaly are also consistent with this diagnosis. Laboratory studies reveal a decreased hemoglobin and elevated reticulocyte count, and the peripheral blood smear is notable for polychromatophilia and occasional spherocytes and rare nucleated erythrocytes. Taken together, these findings are consistent with increased erythrocyte destruction in the periphery, with a compensatory bone marrow response. The appropriate initial therapy for this patient is prednisone, 1 mg/kg daily, to which 65% to 80% of patients respond. Intravenous immunoglobulin and danazol may have activity in some patients with autoimmune hemolytic anemia but are not first-line agents. Likewise, splenectomy should be reserved for patients who are corticosteroid-resistant. Erythropoietin may be of value in certain hypoproliferative anemias or in patients with anemias secondary to exogenous influences such as chemotherapy. However, endogenous erythropoietin levels are elevated in patients with autoimmune hemolytic anemia, and the use of exogenous erythropoietin is of uncertain benefit.

Page 161: MKSAP13-Hematology and Oncology

Hematology and Oncology:Question 43 The correct answer is C Educational Objectives Diagnose and treat high-risk essential thrombocythemia. Critique Essential thrombocythemia is the most common myeloproliferative disorder affecting middle-aged and older adults and is characterized by thrombotic and hemorrhagic complications. Diagnosis is made in patients with platelet counts in excess of 600,000IjiL in the absence of known causes for reactive thrombocytosis. Essential thrombocythemia may be difficult to distinguish from other myeloproliferative syndromes. A hemoglobin level of 13 g/dL or lower excludes polycythemia vera; absence of t(9;22) excludes chronic myeloid leukemia, and absence of marked marrow fibrosis excludes myelofibrosis. Low-dose aspirin therapy is beneficial in the treatment of ischemic neurologic and vision symptoms and erythromelalgia in essential thrombocythemia. Patients older than 60 years or patients with a history of thrombotic events are considered to be at high risk for thromboembolic complications, and use of hydroxyurea to reduce the platelet count below 400,000/μL significantly reduces this risk.

Page 162: MKSAP13-Hematology and Oncology

Therefore aspirin therapy alone would not be sufficient for this patient. Treatment with a nonmutagenic agent such as anagrelide, which blocks megakaryocyte maturation, would also reduce the platelet count, but there are no randomized data regarding reduction of thrombotic events. Warfarin anticoagulation should be avoided in this patient because her essential thrombocythemia puts her at high risk for bleeding complications. Busulfan therapy is associated with significant risk of secondary leukemias and does not have a role in management of essential thrombocythemia. Imatinib mesylate is specific therapy for chronic myeloid leukemia and is not indicated in essential thrombocythemia.

Hematology and Oncology:Question 44 The correct answer is A Educational Objectives Recognize the indications for transfusion of fresh frozen plasma. Critique This patients liver disease gives evidence of poor synthetic function. Although the results of the coagulation tests are only slightly abnormal, it can be anticipated that excessive

Page 163: MKSAP13-Hematology and Oncology

hemorrhage due to impaired hemostasis will be a complication during major surgery. The cause of his recurrent venous thrombosis is unclear. The risks of donor exposures preclude prophylactic transfusion of fresh frozen plasma merely to treat elevated coagulation findings. In general, the hemostatic effect of fresh frozen plasma lasts only as long as the biologic life of the shortest factors being replaced. For example, factor VII has a steady-state half-life of approximately 6 hours. Also because of the risks of transfusion, hypovolemia and hypoproteinemia are best treated with sterilized plasma derivatives such as albumin or with plasma expanders. Albumin may transiently correct hypoproteinemia caused by malnutrition or decreased synthesis but is usually not effective if the protein deficiency is due to nephrosis or capillary leak. The anticoagulant effect of warfarin can be temporarily corrected by fresh frozen plasma by supplying factors that are synthesized with normal gamma carboxylation. Heparin, however, behaves as an inhibitor and its effect is not corrected through fresh frozen plasma administration. This patient’s low-molecular-weight heparin should be withheld the day before surgery to allow for the elimination of the anticoagulant effect; fresh frozen plasma is not, however, required because of the

Page 164: MKSAP13-Hematology and Oncology

low-molecular-weight heparin, nor would fresh frozen plasma be effective until the drug effect was diminished.

Hematology and Oncology:Question 45 The correct answer is E Educational Objectives Begin appropriate initial therapy for idiopathic thrombocytopenic purpura. Critique The presumptive diagnosis in this patient is idiopathic thrombocytopenic purpura; no other diagnostic studies are required to initiate therapy. Prednisone, 1 mg/kg daily, would be the most appropriate initial management. Approximately 10% to 15% of patients may have resolution of their disease or achieve stable platelet counts (greater than 30,000/ μL) on a low dose of prednisone. Splenectomy should not be routinely done if the patient responds initially to prednisone. Although the use of splenectomy in the treatment of idiopathic thrombocytopenic purpura remains in evolution, either splenectomy or the use of other agents such as intravenous immunoglobulin or intravenous anti-Rh(D) could be considered if the patient either does not respond to prednisone or requires unacceptably high doses to maintain a safe platelet count. Therapy is indicated because the platelet count

Page 165: MKSAP13-Hematology and Oncology

is very low, and the presence of petechiae suggests microvascular bleeding; therefore, simple observation would not be appropriate. Intravenous immunoglobulin and intravenous anti-Rh(D) are both highly effective therapies for idiopathic thrombocytopenic purpura, but because of cost are generally reserved for patients whose thrombocytopenia is severe or who do not achieve a stable platelet count in response to a low dose of prednisone (less than 5 to 10 mg/d).

Hematology and Oncology:Question 46 The correct answer is C Educational Objectives Administer appropriate therapy for a symptomatic distal deep venous thrombosis after arthroscopic surgery. Critique The incidence of symptomatic venous thrombosis after arthroscopic surgery is not nearly as high as that after total knee replacement. Arthroscopic surgery is, however, a risk factor, and venographic studies have shown an incidence of 4% to 17% after such procedures. Symptomatic calf vein thrombosis that has been objectively documented should be treated with therapeutic doses of low-molecular-weight heparin, if it is feasible to treat the patient entirely in the outpatient setting, followed by warfarin for a period of 3

Page 166: MKSAP13-Hematology and Oncology

months. Approximately 20% of patients develop clot extension if anticoagulation is not undertaken; therefore it is not recommended to use anti-inflammatory agents and to obtain a repeat ultrasound in 1 week. There is no indication for placement of an inferior vena cava filter, such as inability to undergo therapeutic anticoagulation or clot extension at a therapeutic level of anticoagulation. Enoxaparin, 30 mg subcutaneously every 12 hours, would be a prophylactic rather than a therapeutic regimen for venous thromboembolism in this setting.

Hematology and Oncology:Question 47 The correct answer is B Educational Objectives Select the proper plasmapheresis treatment for thrombotic thrombocytopenic purpura Critique This patient has thrombotic thrombocytopenic purpura and should be treated by plasmapheresis with fresh frozen plasma. This syndrome has been linked to disturbances in the macromolecular structure of factor VIII, caused by autoantibodies or enzyme defects. Daily plasma exchange with fresh frozen plasma allows an entire blood volume to be replaced with normal plasma while avoiding hypervolemia and is the standard of care for

Page 167: MKSAP13-Hematology and Oncology

this disorder. Therapy should be continued until well after the hemolysis has subsided and the lactate dehydrogenase level and platelet count have returned to normal; relapses are common if plasma exchange is discontinued prematurely. Plasmapheresis with the usual saline and albumin replacement is not used for thrombotic thrombocytopenic purpura. Thrombocytic thrombocytopenic purpura is one of the few disorders that requires fresh frozen plasma as pheresis replacement, because the beneficial effect is related to what is being given, in addition to what is being removed. Platelet transfusion as initial therapy in thrombotic thrombocytopenic purpura has been reported to worsen the microangiopathy, often with fatal outcome. Whether platelets can be safely used to treat bleeding once a patient’s thrombotic thrombocytopenic purpura has started to respond to plasma exchange remains controversial. Some hematologists believe that corticosteroids used early in thrombotic thrombocytopenic purpura can hasten resolution, but only as an adjunct to fresh frozen plasma infusion and plasmapheresis.

Page 168: MKSAP13-Hematology and Oncology

Hematology and Oncology:Question 48 The correct answer is B Educational Objectives Prevent venous thromboembolism after major orthopedic surgery in a patient at high risk for recurrent thrombosis. Critique Anticoagulation prophylaxis should be continued for 6 weeks. Because this patient had adequate therapy after a first deep venous thrombosis, his risk of recurrence would be quite low during the several weeks between cessation of warfarin and the orthopedic surgery, as long as he is reasonably active. Therefore, “bridging” therapy with low-molecular-weight heparin or intravenous unfractionated heparin would be unnecessary in the days immediately before surgery. Administration of low-molecular-weight heparin after knee replacement surgery for as long as 4 weeks has not been conclusively shown to reduce the rate of deep venous thrombosis, as compared with 7 to 10 days of therapy. The patients risk of postoperative venous thromboembolism is, however, greater than average because of his previous episode. Therefore it is recommended that anticoagulant prophylaxis be continued for 6

Page 169: MKSAP13-Hematology and Oncology

weeks after surgery rather than just 1 week. Aspirin is not a recommended agent for prophylaxis of deep venous thrombosis after major joint replacement.

Hematology and Oncology:Question 49 The correct answer is A Educational Objectives Recall the indications for red cell transfusion. Critique Except in the case of cardiovascular compromise, most patients can compensate for reductions in red cell mass through increased stroke volume, myocardial contractility, and other responses. Red blood cell transfusion is indicated in order to raise the hematocrit of this patient with cardiac disease. Many physicians now allow patient hemoglobins to drift to low levels, with an absolute threshold around 7 g/dL as the point where myocardial oxygen consumption is impaired strictly by low red cell mass. If this patient had not had cardiac disease, the red cells would not have been indicated. Increases in red cell mass will not improve lung function, and may contribute to some ventilation-perfusion mismatches. Most patients are on ventilators because of airway or alveolar dysfunction and not because of poor oxygen transport. Hypoxia from a pulmonary cause is not usually improved by

Page 170: MKSAP13-Hematology and Oncology

transfusion. Transfusion of red cells can improve hypotension and hypovolemia, but red cells are not as efficient at increasing oncotic pressure as colloids or plasma expanders. The risks of donor exposure are not warranted simply to increase circulating volume.

Hematology and Oncology:Question 50 The correct answer is C Educational Objectives Recognize the clinical manifestations of chronic graft-versus-host disease. Critique Approximately 50% to 70% of the recipients of allogeneic stem cell transplantation develop some manifestations of chronic graft-versus-host disease. Skin involvement is common, with scleroderma, lichen planus, vitiligo, scarring eczematoid rash, hyperpigmentation, and hair loss. Anorexia, malabsorption, weight loss, diarrhea, and abdominal pain are predominant gastrointestinal manifestations of the disease. Biopsy of the rectal or colonic mucosa may be helpful in confirming the diagnosis and eliminating other potential causes such as cytomegalovirus colitis. Negative biopsy results do not rule out underlying graft-versus-host disease, and symptomatic patients should be treated with corticosteroids empirically.

Page 171: MKSAP13-Hematology and Oncology

Pseudomembranous colitis is uncommon with trimethoprim-sulfamethoxazole therapy. Laxative abuse is unlikely and does not explain skin changes. Shigella, often found in stool culture, is associated with bloody diarrhea. Recurrence of chronic myeloid leukemia would not cause diarrhea.

Hematology and Oncology:Question 51 The correct answer is C Educational Objectives Treat venous thrombosis associated with transient risk factors. Critique Although this patient has protein S deficiency and a strong positive family history of venous thromboembolism, her first thrombotic event occurred in association with several transient risk factors; these include pregnancy and a caesarean section. Therefore, despite her protein S deficiency, it is appropriate to discontinue oral anticoagulation therapy after 3 to 6 months at a target INR of 2 to 3 and employ it prophylactically in high-risk situations only. Neither low fixed doses of warfarin nor the combination of aspirin and clopidogrel has been shown to be efficacious for the prevention of recurrent venous thromboembolism.

Page 172: MKSAP13-Hematology and Oncology

Hematology and Oncology:Question 52 The correct answer is C Educational Objectives Recognize and differentiate myelodysplastic syndrome from other causes of anemia. Critique Myelodysplastic syndromes are clonal hematopoietic stem cell disorders that occur predominantly in patients older than 50 years. They are characterized by peripheral blood cytopenias and dysplasia of erythroid, granulocytic, or megakaryocytic lineages. The mean corpuscular volume is commonly elevated. Although a history of diverticulosis and aspirin use suggests possible bleeding and iron deficiency, the low leukocyte and platelet counts, high mean corpuscular volume, and findings on peripheral blood smear do not support a diagnosis of iron deficiency anemia. Angiotensin-converting enzyme (ACE) inhibitors are rarely associated with blood dyscrasia, and the findings described do not occur with drug-induced cytopenias. Vitamin Bi 2 deficiency can cause all the laboratory abnormalities described except reduced segmentation of neutrophils. By contrast, there is hypersegmentation in vitamin Bi 2 deficiency. Myelofibrosis is commonly associated with splenomegaly.

Page 173: MKSAP13-Hematology and Oncology

Hematology and Oncology:Question 53 The correct answer is A Educational Objectives Recognize the presentation of a bacterial contamination reaction. Critique Bacterial contamination of platelets can occur when skin plugs enter the blood during needle insertion or because of asymptomatic donor bacteremia. It is most common in platelet concentrates that are 4 to 5 days old—long enough for log-phase growth to occur in the inoculum. The symptoms are due to the pyrogens associated with the bacteria, toxins elaborated during their growth, and the recipient’s inflammatory response. This patients fever, the fact that he has no respiratory distress or wheezing, and the time sequence make anaphylaxis unlikely as a cause of his symptoms. Group AB individuals have no ABC isoagglutinins and are therefore “universal” plasma/platelet donors. Febrile, nonhemolytic reactions from antibodies to donor leukocyte antigens are common but rarely cause hypotension and vascular collapse; leukocyte reduction prevents or alleviates these reactions. Passive acquisition of donor leukoagglutinins causes transfusion-related acute lung injury, but respiratory distress and hypoxemia are the

Page 174: MKSAP13-Hematology and Oncology

hallmarks of this reaction, not fever and hypotension.

Hematology and Oncology:Question 54 The correct answer is D Educational Objectives Determine a schedule by which to alter a patient’s anticoagulation therapy. Critique It would be best to initiate warfarin at 5 mgld, and discontinue argatroban when the INR is greater than 4.0. How to alter a patients anticoagulation therapy from argatroban to warfarin has been a confusing issue to clinicians. Argatroban, although it is monitored by using the partial thromboplastin time, affects the INR even when used alone. Both argatroban and warfarin prolong the INR; a schedule that called for argatroban to be discontinued when the INR reached 2.0 would not allow enough time for the desired anticoagulation effect of warfarin to be achieved. Therefore, it is recommended that argatroban be discontinued only after the INR is greater than 4.0. The INR should then be rechecked 4 to 6 hours after argatroban is stopped. If it is not in the therapeutic range of 2 to 3, argatroban should be restarted and continued until the INR attrributable to warfarin alone is in the therapeutic range. Some experts also recommend that argatroban be continued

Page 175: MKSAP13-Hematology and Oncology

for at least 3 days after starting warfarin, to ensure that a full anticoagulant effect of warfarin is achieved before argatroban is discontinued. It is also recommended that the argatroban infusion be reduced to 2 μg/kg perminute before warfarin is initiated. Discontinuing argatroban at the time warfarin is initiated, or 24 hours before warfarin initiation, would leave the patient without anticoagulation therapy for 3 to 4 days, the amount of time required for the anticoagulant effect of warfarin to be achieved. Furthermore, it would expose the patient to the transient procoagulant effect of warfarin that can occur soon after therapy is initiated (owing to a rapid fall in the levels of protein C, the vitamin K-dependent coagulation inhibitor). A loading dose of warfarin is unnecessary; it does not hasten the achievement of a stable therapeutic INR any more than does therapy initiated at a dose of 5 mg/d. Furthermore, initiation of therapy at high doses is associated with an increased incidence of bleeding complications.

Hematology and Oncology:Question 55 The correct answer is B Educational Objectives Determine the appropriate duration of anticoagulant therapy for a patient with a postoperative deep venous thrombosis.

Page 176: MKSAP13-Hematology and Oncology

Critique The appropriate period of anticoagulation for a symptomatic post-operative deep venous thrombosis is 3 months. Although low-molecular-weight heparin is effective in reducing the rate of deep venous thrombosis after total hip replacement, the incidence remains at 10% to 20% as assessed by bilateral venography after 7 to 10 days of therapy. The rate increases to 25% to 30% at 1 month if extended prophylaxis is not employed. Therapy for 6 weeks will result in a higher recurrence rate than therapy that lasts 3 to 6 months. Clearly, this patient’s thrombosis was provoked by the joint replacement surgery. Therefore the recurrence risk will be quite low, and anticoagulation for 1 year or for the rest of her life is not warranted.

Hematology and Oncology:Question 56 The correct answer is E Educational Objectives Diagnose platelet refractoriness. Critique The refractoriness observed in this patient is most probably caused by alloimmunization. Approximately 30% of pregnancies are associated with low levels of HLA alloimmunization, presumably through stimulation from placental tissue as well as unapparent fetal-maternal hemorrhage. These

Page 177: MKSAP13-Hematology and Oncology

HLA antibodies often decay to subdetectable levels quickly, but the womans immune system retains the ability to form an anamnestic response when rechallenged. Just like patients who have received many transfusions, multiparous women are at risk of having a high level of circulating HLA antibody. Although these alloantibodies do not themselves cause thrombocytopenia (because they do not destroy autologous platelets), they are a major cause of platelet refractoriness. When immune refractoriness exists, unmatched platelet transfusions are usually ineffective, and the patient requires specially ordered matched platelets, usually obtained from a single donor through apheresis. This patients catastrophic hemorrhage is a nonimmunologic complication of pregnancy. The post-transfusion platelet increment was less than 25% of the expected rise for someone of average body size and blood volume. The best discriminator of immune and nonimmune platelet destruction is the measurement of the immediate post-transfusion platelet count. Immune destruction occurs quickly, often within minutes of a transfusion and many times without evidence of a transfusion reaction. Nonimmune causes do not usually lower the platelet increment until well after 1 hour post-transfusion. The

Page 178: MKSAP13-Hematology and Oncology

observed poor platelet count increment in this patient suggests platelet transfusion refractoriness, which can have multiple causes.

Although moderate thrombocytopenia can develop through loss of platelets during vigorous bleeding and through the dilutional effect of massive transfusion, these mechanisms usually do not cause transfusion refractoriness. Mild thrombocytopenia of nonimmune origin can occur in many pregnancies, but the patients normal platelet count prior to delivery rules out this complication. Disseminated intravascular coagulation can lower platelet counts, but almost always causes abnormalities in the tests of clotting function. In the HELLP (hemolysis, elevated liver enzymes, and low platelet count) syndrome, the low platelet count is evident before delivery.

Hematology and Oncology:Question 57 The correct answer is B Educational Objectives Diagnose and treat iron overload Critique The most effective procedure to assess the possibility of iron overload in this patient is a diagnostic liver biopsy. The high serum indirect bilirubin and low serum haptoglobin levels in

Page 179: MKSAP13-Hematology and Oncology

association with the normal reticulocyte count are evidence of ineffective erythropoiesis. The elevated serum ferritin concentration and high transferrin saturation levels are consistent with iron overload. Given the patients ancestry and clinical picture, he could have iron overload related to 3-thalassemia intermedia or sideroblastic anemia. Homozygosity for the C282Y mutation in the HFE gene occurs in approximately 1 in 200 persons of northern European ancestry and is the most common cause of iron overload in whites. Mutations in other genes (TfR2, FPN 1) that lead to hereditary iron overload have recently been reported. Dietary iron overload is common in blacks from rural southern Africa and may have a genetic component. Nontransfusional iron overload also occurs in certain anemias characterized by high degrees of ineffective erythropoiesis. Such anemias include β-thalassemia major and intermedia, hemoglobin H disease, β-thalassemia/hemoglobin E, and hereditary or acquired sideroblastic anemias. A quantitative phlebotomy program is an alternative way to document iron overload but would be difficult to perform in an anemic patient and does not inform about the presence of iron-related hepatic damage. Staining a bone marrow aspirate for iron is the

Page 180: MKSAP13-Hematology and Oncology

gold standard for diagnosing iron deficiency but is not reliable for diagnosing iron overload. Treatment with desferrioxamine should be instituted only in an anemic patient with confirmed iron overload, because the toxicity of the agent is markedly increased if iron stores are not increased.

Hematology and Oncology:Question 58 The correct answer is E Educational Objectives Treat chronic myeloid leukemia in an elderly patient. Critique This patients presentation, examination findings, and laboratory test results are consistent with chronic-phase chronic myeloid leukemia (CML). Demonstration of a balanced translocation between chromosomes 9 and 22 (Philadelphia chromosome) confirms the diagnosis. Despite her advanced age and multiple medical problems, this patient can be successfully treated with imatinib mesylate (Gleevec). In 90% of patients treated with a daily oral dose of imatinib mesylate, blood counts become normal, and 30% to 40% of the patients no longer have detectable CML cells in their bone marrow. Transient leukopenia, mild nausea, and muscle cramps are common side effects of therapy

Page 181: MKSAP13-Hematology and Oncology

with imatinib mesylate. Rash, hepatotoxicity, and fluid retention may also occur. The optimal duration of therapy, long-term benefits, and toxicity of imatinib mesylate are under investigation. Without therapy, this patient is at risk of developing hyperleukocytosis and circulatory compromise, progressive splenomegaly, metabolic complications, and infections. Because there is no evidence of progression to acute leukemia, there is no need for systemic induction chemotherapy. Allogeneic bone marrow transplantation would not be an appropriate treatment option, especially given the patient’s age and history of cardiac problems. However, interferon-alfa therapy would be poorly tolerated and would not be a good choice for this patient. Daily subcutaneous interferon-alfa injections are more effective in decreasing CML cells in the bone marrow, and if not discontinued because of toxicity can result in 5-year survival rates of 50% to 70%. Hydroxyurea is commonly used to normalize blood counts quickly and results in a 5-year survival of 35% to 50%. Hydroxyurea is well tolerated and could be an alternative therapy for this patient, particularly if the cost of imatinib mesylate is prohibitive.

Page 182: MKSAP13-Hematology and Oncology

Hematology and Oncology:Question 59 The correct answer is B Educational Objectives Recognize and initiate therapy for the HELLP syndrome Critique This patient has the HELLP (hemolysis, elevated liver function tests, low platelets) syndrome. Some authorities consider it a variant of preeclampsia, because approximately 10% of patients with HELLP syndrome ultimately meet criteria for preeclampsia. As this case illustrates, however, a diagnosis of HELLP syndrome does not require hypertension and proteinuria. As with preeclampsia, definitive therapy for HELLP syndrome should be focused toward urgent delivery of the fetus, after which the disorder usually resolves. There is no role for intravenous immunoglobulin in management of HELLP syndrome or preeclampsia. Plasma exchange is indicated for thrombotic thrombocytopenic purpura, which must be considered in the differential diagnosis of this patient’s disorder.

Page 183: MKSAP13-Hematology and Oncology

Her presentation and laboratory studies, however, are most consistent with the HELLP syndrome. There is no role for aggressive diuresis or antiplatelet agents in the management of established preeclampsia or HELLP syndrome. Randomized trials of prophylactic aspirin to decrease the incidence of preeclampsia have not shown a benefit to treatment, and antiplatelet agents may also increase the incidence of thrombocytopenic bleeding complications.

Hematology and Oncology:Question 60 The correct answer is E Educational Objectives Recognize myelofibrosis. Critique Myelofibrosis is characterized by splenomegaly, hepatomegaly, normocytic anemia, circulating erythroblasts and myeloid precursors, teardrop cells, and marked bone marrow fibrosis. The splenomegaly and hepatomegaly result from extramedullary hematopoiesis. Other myeloproliferative syndromes have to be excluded in order to ascertain the diagnosis. Although this patient has circulating myeloid precursors, absence of an elevated leukocyte count and FISH analysis that is negative for t(9;22) exclude chronic myeloid leukemia.

Page 184: MKSAP13-Hematology and Oncology

Hairy cell leukemia can present with pancytopenia, splenomegaly, and dry bone marrow aspiration. Peripheral blood smear in this disorder shows characteristic hairy mononuclear cells, and leukoerythroblastosis is uncommon. Patients with myelodysplastic syndromes can have varying degrees of marrow fibrosis, but massive splenomegaly and hepatomegaly are uncommon. There are typically severe dysplastic abnormalities in one or more lineages in blood and marrow of patients with myelodysplastic syndrome. Disseminated tuberculosis is also associated with marrow fibrosis. However, caseating or noncaseating granulomas are typically observed on bone marrow biopsy.

Hematology and Oncology:Question 61 The correct answer is B Educational Objectives Diagnose an allergic transfusion reaction. Critique This patient is having an acute anaphylactic transfusion reaction. The presence of mucosal edema, bronchospasm, and hypotension make it more than a severe systemic allergic reaction. Bacterial contamination can lead to respiratory distress and shock, but fever is often present. Furthermore, because fresh frozen plasma is

Page 185: MKSAP13-Hematology and Oncology

stored frozen and used within hours of thawing, there is insufficient time for any contaminating bacteria to grow to a sufficient inoculum size. ABC-incompatible acute intravascular hemolytic reactions can cause symptoms like those of this patient, even before overt hemolysis is evident. The units given were group AB; therefore ABC plasma incompatibility cannot be the cause. Fresh frozen plasma contains virtually no intact red cells; therefore the recipient’s naturally occurring anti-B has no cellular target in the group AB plasma. The hallmark of transfusion-related acute lung injury is hypoxemia from pulmonary leukoagglutination and alveolitis rather than bronchospasm and edema, and allergic reactions like this patients urticaria and rash are uncommon. Patients taking angiotensin-converting enzyme inhibitors may have sudden acute hypotension due to disordered bradykinin metabolism, but these reactions are quick to resolve and cause few other symptoms.

Hematology and Oncology:Question 62 The correct answer is B Educational Objectives Recognize the risk posed by the prothrombin G20210A mutation. Critique

Page 186: MKSAP13-Hematology and Oncology

The prothrombin G2021 0A gene mutation is a definite risk factor for an initial episode of deep venous thrombosis, and she should encourage her first-degree relatives to be tested for the mutation. The incidence of recurrence in patients with a first unprovoked episode of deep venous thrombosis after adequate treatment (3 to 6 months of anticoagulation at therapeutic doses) is 5% to 15% per year. Heterozygosity for the prothrombin gene mutation is a definite risk factor for an initial episode of deep venous thrombosis in whites; most studies, however, have not shown the recurrence risk to be any greater in patients with an identifiable thrombophilic defect than in those without one. Lifelong anticoagulation is generally not undertaken for an uncomplicated first episode of deep venous thrombosis, although recent data from the PREVENT trial indicate that anticoagulation at a target INR of 1.5 to 2 will provide significant protection against recurrent venous thromboembolism without a significant risk of major bleeding.

Hematology and Oncology:Question 63 The correct answer is D Educational Objectives Identify patients with idiopathic thrombocytopenic purpura in whom anti-Rh(D) therapy is not appropriate.

Page 187: MKSAP13-Hematology and Oncology

Critique This patient has idiopathic thrombocytopenic purpura that has proved refractory to corticosteroid therapy and splenectomy. The use of intravenous anti-Rh(D) would not be indicated in this patient. Although the mechanisms of this agent in idiopathic thrombocytopenic purpura are complex, it is believed to act largely by blocking uptake of antibody-coated platelets by Fc receptors in the spleen. It is not efficacious in splenectomized patients. Initial therapy of idiopathic thrombocytopenic purpura with corticosteroids leads to a remission rate of 70% to 80%, although ultimately only 15% to 25% of these remissions can be sustained on an acceptable dosage of corticosteroids. Second-line therapy of idiopathic thrombocytopenic purpura may include intravenous immunoglobulin, anti-Rh(D), splenectomy, or a number of other approaches. Splenectomy induces remission in approximately 80% of such patients, although relapses, most of which occur in the first year after splenectomy, reduce the number of long-term responders to approximately 65%. Intravenous immunoglobulin is an acceptable agent to use in refractory idiopathic thrombocytopenic purpura after splenectomy and probably the most likely of the agents

Page 188: MKSAP13-Hematology and Oncology

mentioned to induce a response. Responses to intravenous immunoglobulin, however, are most often transient. Rituximab, an anti-CD20 monoclonal antibody, is associated with a response rate of 25% to 50% in patients with refractory idiopathic thrombocytopenic purpura. Danazol and cyclophosphamide are also of benefit in some patients, but their effect may require several weeks to become apparent.

Hematology and Oncology:Question 64 The correct answer is B Educational Objectives Recognize the late complications of breast cancer treatment. Critique This patients previous radiation therapy is the likely cause of her fibrosarcoma. Therapeutic radiation is associated with a 1% per year risk of a second solid tumor within the radiated field for at least 25 years after treatment. It is not clear whether the risk diminishes with longer time periods after therapy. Patients who receive therapeutic radiation, even in very small total doses, are at increased risk, and the radiation field should be carefully examined regularly each year after treatment. People cured of cancer remain at risk for other cancers. Second (and third) metachronous (that is, occurring at another time) cancers can

Page 189: MKSAP13-Hematology and Oncology

develop from a variety of causes. Approximately 15% of patients who are cured of a first cancer by a surgical procedure (no chemotherapy or radiation therapy) develop a second neoplasm in another organ. Typically the majority of these cases involve two common forms of cancer, for example, colon and prostate cancer in men or colon and breast cancer in women. Fibrosarcoma is a rare tumor; therefore, it is unlikely to be completely unrelated to the breast cancer and its treatment. Some tumors occur in distinct organs but have very similar risk factors; for example, head and neck cancer patients are at increased risk of lung cancer and both kinds of tumor occur with increased frequency in tobacco and alcohol abusers. Fibrosarcoma does not share risk factors with breast cancer; therefore, similar risk factors do not account for these metachronous cancers. Tamoxifen is associated with an increased risk of endometrial cancer and a decreased risk of a second breast cancer and colon cancer. It is not known to cause fibrosarcoma. Some organs are susceptible to multiple metachronous tumors, such as bladder cancer and breast cancer. Fibrosarcoma is not a tumor that routinely occurs as multiple metachronous tumors.

Page 190: MKSAP13-Hematology and Oncology

Hematology and Oncology:Question 65 The correct answer is C Educational Objectives Confirm staging test results before precluding surgery for non-small-cell lung cancer. Critique It is first necessary to determine whether the nodules in the liver are benign or malignant by percutaneous needle biopsy. Surgery with curative intent should be offered if the non-small-cell lung cancer appears localized to the area near the left hilum; it would be obviated if the nodules in the liver were found to be malignant. Radiation therapy alone would be inappropriate if liver metastases were confirmed because palliative chemotherapy would be the preferred approach. Chemotherapy would be inappropriate if the liver nodules are benign (outside of a clinical trial), because it would deny or delay potentially curative surgery. Measuring serum carcinoembryonic antigen (CEA) would not be helpful in any case; CEA may be elevated in localized or metastatic lung cancer. The positron emission tomography scan was unhelpful because it did not indicate abnormal uptake in the liver nodules. The false-negative rate of positron emission tomography scans in this setting is not yet known with certainty,

Page 191: MKSAP13-Hematology and Oncology

especially because faint activity can be interpreted as positive or negative, depending on the radiologist.

Hematology and OnCology:Question 66 The correct answer is A Educational Objectives Recognize modifiable risk factors related to cancer of the head and neck. Critique The patient should avoid excessive use of alcohol, a factor that has been linked to squamous cell carcinoma of the head and neck. Tobacco and alcohol abuse appear to be synergistic as causative agents for head and neck cancer. Benzene exposure is known to be a causative factor for leukemias, but is not known to cause lung or head and neck cancers. Daily vitamin supplementation may improve other health outcomes, but there is no evidence that this strategy will reduce cancer risk. There is no evidence that supplementation with β-carotene has any effect on the risk of head and neck cancer.

Hematology and Oncology:Question 67 The correct answer is D Educational Objectives Recognize the effects of radiation therapy in patients treated for clinically localized prostate

Page 192: MKSAP13-Hematology and Oncology

cancer. Critique Radiation therapy to the prostate gland has significant morbidity; for example, it is associated with acute proctitis in 18.7% of patients. Incontinence is more common with surgically treated patients (severe urinary incontinence in 9.6 % vs. 3.5 %), and impotence is common in both groups. Thrombosis occasionally occurs in the postoperative period, and it is not commonly related to radiation therapy.

Hematology and Oncology:Question 68 The correct answer is D Educational Objectives Make an appropriate referral for a patient with potentially resectable pancreatic cancer. Critique This case is a classic presentation of pancreatic cancer. CT results indicate that the patients tumor may be resectable. The best option is referral of the patient to a center with surgical expertise in the management of pancreatic cancer, because surgical resection offers the only potential for long-term survival. Futhermore, operative mortality is significantly lower at a center where many such procedures are performed than at centers with limited experience. Because the patient does not have

Page 193: MKSAP13-Hematology and Oncology

symptomatic jaundice, emergency biliary drainage is not indicated. Establishing a definitive histologic diagnosis is not necessary before the referral, in a patient with potentially resectable disease.

Hematology and OnCology:Question 69 The correct answer is A Educational Objectives Recognize which screening tests are associated with the strongest evidence of reducing the mortality rate in cancer Critique All three randomized trials of biennial fecal occult blood testing (FOBT) reported to date have shown a decrease in colorectal cancer mortality with screening compared with controls. One of the trials had an annual screening arm, which showed an even greater reduction in the number of deaths from colorectal cancer (Mandel et al). The reduction appears to be similar whether or not FOBT slides are rehydrated, although the false-positive rate of the tests is increased by rehydration. In one of the three trials of biennial screening, it was estimated that approximately five to ten deaths from colorectal cancer would be averted for every serious complication or postoperative death from the screening-treatment process (Robinson et al). It is

Page 194: MKSAP13-Hematology and Oncology

uncertain whether FOBT decreases the incidence as well as the mortality from colorectal cancer; results have been inconsistent. Most North American professional organizations recommend the use of sigmoidoscopy in screening for colorectal cancer every 5 years. The recommendation is supported by well-performed case-control studies that show an association between sigmoidoscopy within the previous 10 years and lower risk of death from colorectal cancer. Because this study design relies on prior choice to undergo the procedure and people who choose to undergo screening tests may have better underlying health than those who do not, its findings represent an intermediate strength of evidence. Furthermore, case-control study designs do not allow for a direct comparison of benefits and harms such as perforation or bleeding. A federally sponsored randomized trial of sigmoidoscopy versus a control arm in more than 150,000 volunteer participants is therefore being conducted, but results are unlikely to be known for several years. To date, the effects of human papillomavirus (HPV) DNA testing and Pap smears have never been tested in a randomized trial. In the case of Pap smears, however, virtually every other

Page 195: MKSAP13-Hematology and Oncology

study design has shown a substantial decrease in the incidence of cervical cancer as well as mortality associated with institution of Pap smears. Because Pap smears target preneoplastic lesions of the cervix, the test is in large measure a form of primary prevention, aimed at lowering the incidence of invasive cancer. Pap smears do detect a large number of noninvasive lesions such as low-grade squamous cell intraepithelial lesions and atypical squamous cells of unknown significance, whose natural history is ill defined. A randomized study is ongoing to determine if some of these low-grade lesions can be monitored without definitive therapy, and if HPV testing can aid in the decision. In a randomized trial of chest radiography plus sputum cytology every 4 months in male cigarette smokers, there was no decrease in the rate of death from lung cancer, even though more operable lung cancers were detected and survival time from diagnosis of lung cancer was increased in the screened group. These observations are most likely due to overdiagnosis—detection of non-life-threatening cancers—by the screening tests. Therefore, no major organizations recommend routine screening for lung cancer, even of current or former cigarette smokers.

Page 196: MKSAP13-Hematology and Oncology

Hematology and OnCology:Question 70 The correct answer is A Educational Objectives Evaluate adenocarcinoma presenting in an isolated axillary lymph node. Critique In a young, nonsmoking woman with adenocarcinoma of the axilla, breast cancer remains the most likely diagnosis. Optimal therapy for breast cancer should be initiated, especially because breast cancer is the most treatable type of adenocarcinoma (even with curative intent in this case). Options include modified radical mastectomy, which identifies an occult primary tumor 30% to 50% of the time, or radiation therapy for breast conservation. In either case, adjuvant chemotherapy would be recommended. The tissue from biopsy should be stained for HER2/neu; positive results would confirm the diagnosis of breast cancer. Although use of radiation therapy is certainly one option, it is not the only option, and some patients and clinicians might prefer modified radical mastectomy. Proceeding with resection alone would not be advised in such a young patient to whom potentially curative options

Page 197: MKSAP13-Hematology and Oncology

are available.

Hematology and Oncology:Question 71 The correct answer is A Educational Objectives Evaluate a breast mass. Critique One of the most common causes of malpractice in the United States is failure to diagnose breast cancer at an early stage. Several studies have demonstrated that prognosis and the degree of therapy required are both directly related to the size and stage at diagnosis, which are in turn directly related to early or delayed detection. Ultrasonography is the procedure of choice for this patient. It can be very helpful in evaluating a palpable abnormality of the breast, frequently increasing suspicion or providing evidence of benign conditions. The sensitivity of mammography is 75% to 90%; therefore, a negative mammogram is insufficient reason to discontinue thorough evaluation of a breast abnormality. No identifiable risk factor (other than being female and over 40 years of age) can be found in 75% of all patients with newly diagnosed breast cancer. MRI of the breast is probably more sensitive than mammography, but the specificity of MRI is quite low and this technique is reserved for special

Page 198: MKSAP13-Hematology and Oncology

circumstances, such as evaluation of a patient who has no evidence of breast cancer but has axillary lymph nodes containing adenocarcinoma. It is also used in the screening of women with BRCA-1 or BRCA-2 germ-line abnormalities. Measurement of circulating tumor markers, although helpful in selected patients with metastatic disease, is insufficiently sensitive or specific to be of use in evaluating a suspicious breast mass.

Hematology and Oncology:Question 72 The correct answer is A Educational Objectives Treat febrile neutropenia in a patient with cancer. Critique A thorough re-assessment should be performed to determine the causative agent. Because this patient’s condition is stable, and the neutrophil count is increasing, it is reasonable to continue the same therapy. For this reason, adding vancomycin or removing the central venous port is not indicated. Although often used in clinical practice, colony-stimulating factors are not recommended because results of randomized trials have failed to demonstrate consistent improvements in the duration of hospitalization, duration of antibiotic therapy, fever, total cost for managing the febrile neutropenic episode, or

Page 199: MKSAP13-Hematology and Oncology

infection-related mortality. The presence of a central venous port removes this patient from the low-risk category; otherwise, his disease might be treated with oral antibiotics. Intravenous ceftazidime remains the best choice.

Hematology and Oncology:Question 73 The correct answer is B Educational Objectives Recognize the long-term complications of mediastinal radiation therapy. Critique The most likely diagnosis is exercise-related myocardial ischemia in this young man. The delivery of therapeutic doses of radiation therapy to the anterior mediastinum includes large segments of the coronary arteries, and the radiation exposure promotes premature atherosclerosis. The Stanford group documented a threefold increase in the rate of fatal myocardial infarction in their patients receiving mantle-field radiation therapy. Thus, it is important that patients who receive mantle-field radiation therapy as a component of their treatment for Hodgkin’s disease be informed of the increased risk of myocardial infarction and counseled about the importance of maintaining a healthy weight and lowering cholesterol with a statin as prophylaxis. Recurrent Hodgkins disease is exceedingly

Page 200: MKSAP13-Hematology and Oncology

unlikely because the disease rarely recurs this late after diagnosis. Half of all cases of relapse of Hodgkin’s disease occur within the first year of completing treatment, and nearly all the rest occur 1 to 5 years after treatment. Furthermore, Hodgkins disease usually recurs in previously involved sites, particularly sites of bulky disease. Therefore, recurrent disease is most likely to be detected by the development of B symptoms according to the Ann Arbor criteria (unexplained fever, drenching night sweats in the previous month, unexplained weight loss of more than 10%) or dyspnea on exertion from a mediastinal recurrence that limits lung function, not by myocardial ischemia. Pulmonary fibrosis can occur in patients treated with bleomycin plus radiation therapy. The presenting signs of pulmonary fibrosis, however, are dyspnea on fibrosis and shortness of breath. Secondary marrow failure is so rare after doxorubicin, blemoycin, vinblastine, and dacarbazine plus radiation therapy that any case would be reportable. Furthermore, presenting symptoms would include pallor, fatigue, and exercise intolerance. Constrictive pericarditis is also unlikely given the time period (15 years ago) of this mans treatment. Twenty to 30 years ago, some

Page 201: MKSAP13-Hematology and Oncology

centers were administering mantle-field radiation therapy with anteriorly weighted ports such that two thirds of the radiation was administered to the front and one third to the back. This method produced a high rate of constrictive pericarditis that has since been dramatically reduced by administering the radiation therapy in equal doses front and back. The onset of constrictive pericarditis from radiation therapy in this setting was usually insidious and associated with progressive exercise intolerance.

Hematology and Oncology:Question 74 The correct answer is C Educational Objectives Recognize androgen ablation as the cornerstone of treatment in metastatic disease of the prostate. Critique Since the 1940s, androgen ablation has been the cornerstone of treatment in metastatic prostate cancer. Although the timing of the

Page 202: MKSAP13-Hematology and Oncology

commencement of therapy is somewhat a matter of controversy, most authorities agree that symptomatic patients should be treated with androgen ablation. Chemotherapy may palliate this patient’s bone pain and fatigue. Chemotherapy with docetaxel or mitoxantrone, corticosteroids, and radiation therapy may be used in the palliation of advanced disease. Outside of a clinical trial, however, such therapy should be used only after disease progression when the patient is receiving androgen blockade.

Hematology and Oncology:Question 75 The correct answer is B Educational Objectives Treat rectal cancer with adjuvant chemoradiation therapy. Critique This patient has stage II rectal cancer. Because the tumor has penetrated the bowel wall, she is at risk of developing a local recurrence, even though no lymph nodes are involved. Adjuvant therapy involving systemic chemotherapy and concurrent pelvic radiation therapy have been shown to reduce the risk of local recurrence and increase survival. Postoperative radiation therapy alone is inferior to the combination of chemotherapy and radiation therapy. Although regular colonoscopy is a reasonable surveillance

Page 203: MKSAP13-Hematology and Oncology

strategy, adjuvant combined-modality therapy should be offered to patients with stage II and III (regional lymph node involvement) disease who otherwise have good performance status.

Hematology and Oncology:Question 76 The correct answer is C Educational Objectives Recognize the relative risks and benefits of hormone replacement therapy (HRT) and tamoxifen to women at increased risk for breast cancer. Critique This patient should stop taking the HRT and start taking tamoxifen. Her risk of developing breast cancer is substantially elevated. It would be reasonable to encourage her to participate in a clinical trial for breast cancer prevention, such as the STAR (Study of Tamoxifen and Raloxifene) trial. The breast cancer risk criterion that qualified women for the Breast Cancer Prevention Trial (BCPT), a placebo-controlled randomized trial of tamoxifen, was an absolute risk of at least 1 .66% over the next 5-year period. The BCPT showed that administration of tamoxifen for as long as 5 years decreased the relative risk of breast cancer by approximately 50%, representing an absolute risk of approximately 5.2% over the next 5 years for this patient. In order to enter the BCPT study, women had

Page 204: MKSAP13-Hematology and Oncology

to have stopped taking HRT at least 3 months before randomization (and remain off it for the duration of the trial). Therefore, the efficacy of tamoxifen for the prevention of breast cancer is not known for women who are concurrently being treated with HRT. The risk of ischemic heart disease in the BCPT was similar in the tamoxifen and placebo arms, although the total number of ischemic heart disease events was small. (In therapeutic studies of tamoxifen for established breast cancer, cardiac disease has also been similar in the treatment and control groups.) Use of estrogen plus progestin HRT does, however, appear to increase the risk of ischemic heart disease in postmenopausal women. In the Women’s Health Initiative (WHI) trial, which studied the use of estrogen plus progestin by healthy postmenopausal women, the relative risk of coronary heart disease was elevated by 29% in the HRT arm. In that trial, HRT also increased the relative risk of breast cancer by 26%. The study was therefore halted because the harms of progestin plus estrogen were believed to outweigh the benefits in the preventive setting. The effect of estrogen alone on heart disease is still under study in the WHI trial, but estrogen alone is not generally given to women with an intact uterus, because it causes endometrial

Page 205: MKSAP13-Hematology and Oncology

cancer.

Hematology and Oncology:Question 77 The correct answer is D Educational Objectives Recognize the potential emergency of spinal cord compression in a cancer patient with new-onset back pain. Critique Spinal cord compression must be ruled out in a patient with cancer who presents with back pain of new onset, and MRI is currently the diagnostic test of choice. Nondiagnostic plain radiographs do not rule out the diagnosis, nor do bone scans, in which increased uptake is dependent on either an osteoblastic reaction or new bone deposition. Because the rate of progression cannot be accurately predicted, the MRI should be obtained promptly. CT scanning might be helpful in determining the degree of bone destruction and whether bone or tumor is impinging on the cord, but it is not the procedure of choice in this situtation.

Most patients with spinal cord compression have pain that is localized to the spine or radicular in nature. Left untreated, spinal cord compression associated with malignancy can progress to myelopathy over a period of weeks. Therefore, it would not be prudent to watch and wait under these circumstances.

Page 206: MKSAP13-Hematology and Oncology

Because the patient does not have any neurologic symptoms, there is no immediate need to administer dexamethasone before establishing the diagnosis.

Hematology and Oncology:Question 78 The correct answer is E Educational Objectives Diagnose intestinal obstruction in a patient treated for rectal cancer. Critique The patient presents with signs of intestinal obstruction, and the most urgent need is to define the cause. Because the distal colon and rectum are not filled with stool, rectal stricture is unlikely. Possibilities include either malignant obstruction (peritoneal carcinomatosis, for example, or metachronous primary tumor) or nonmalignant obstruction (caused by radiation enteritis or adhesions). CT scanning is the most useful diagnostic study to distinguish among the possibilities. It is more sensitive and specific than a gastrointestinal series and shows more of the intra-abdominal anatomy. Because colonoscopy done 1 year earlier was unremarkable, and the current radiographic studies suggest that the obstruction is at the level of the small bowel, another colonoscopy is unlikely to be helpful. The patient does not have signs of an acute abdomen; therefore a

Page 207: MKSAP13-Hematology and Oncology

surgical consultation can be deferred until additional diagnostic studies have been done to identify the cause of the obstruction. The results of a carcinoembryonic antigen test would not be available for a few days, and would have little influence on management of this case.

Hematology and Oncology:Question 79 The correct answer is E Educational Objectives Appropriately monitor a patient taking tamoxifen. Critique Women taking tamoxifen should be carefully questioned about abnormal gynecologic symptoms such as bleeding and discharge and given a careful pelvic examination, reserving diagnostic work-up for those with symptoms or physical abnormalities. Tamoxifen taken to prevent breast cancer increases the risk of endometrial cancer approximately two- to threefold in postmenopausal women with an intact uterus; the absolute risk is approximately 2 to 3 per 1000 women per year. Most of the endometrial cancers diagnosed in these women involve such symptoms as dysfunctional uterine bleeding or discharge, and most are localized (International Federation of Gynecology and Obstetrics stage I) and surgically curable.

Page 208: MKSAP13-Hematology and Oncology

Approximately 40% of women taking tamoxifen will develop endometrial thickening as assessed by ultrasound examination. The false-positive rate of ultrasononography of the uterus is high in asymptomatic women, however, leading to a high rate of invasive testing and attendant risk of uterine perforation. The diagnostic yield of routine endometrial cytologic sampling or biopsy for the detection of early endometrial cancer in asymptomatic women taking tamoxifen is too low to warrant the pain and risk of the procedure. Therefore, despite the increased risk of endometrial cancer in women receiving tamoxifen, routine screening tests such as ultrasound and biopsy or endometrial aspiration are not indicated.

Hematology and Oncology:Question 80 The correct answer is A Educational Objectives Recognize the clinical presentation of bronchoalveolar cell carcinoma. Critique This is a classic presentation of bronchoalveolar cell carcinoma. The fact that the patient is an older woman who is a nonsmoker suggests she has a type of adenocarcinoma, of which bronchoalveolar cell carcinoma is considered a subtype. The tumor cells of bronchoalveolar cell

Page 209: MKSAP13-Hematology and Oncology

carcinoma are better differentiated than those of the usual adenocarcinoma, and the disease follows a more indolent course. Many patients live for months without requiring palliative therapy. The disease is usually bilateral when multifocal or metastatic. Infiltrates and nodules are often seen. Palliative chemotherapy is usually offered when patients become symptomatic. Overt metastases to other organs are much less common for this type of cancer at presentation.

Hematology and OnCology:Question 81 The correct answer is B Educational Objectives Monitor a patient after primary and adjuvant systemic therapy for breast cancer. Critique Breast cancer survivors have special concerns. One of these is whether they should undergo special studies routinely to detect asymptomatic, occult metastases. No trials have demonstrated that doing so provides any benefit to the patient, and two prospective randomized trials demonstrated that frequent, routine restaging with chest radiographs and hepatic imaging failed to result in improved survival or quality of life. Therefore, the value of such testing is questionable at best, and harmful at worst. Women who are rendered postmenopausal by treatment require special

Page 210: MKSAP13-Hematology and Oncology

counseling regarding sexual dysfunction and prevention of osteoporosis. Although estrogen is the most effective treatment for these symptoms, it is now established that estrogen replacement therapy with progesterone modestly increases the risk of a new primary breast cancer.

Hematology and Oncology:Question 82 The correct answer is C Educational Objectives Recognize the role of second-line systemic chemotherapy in patients with metastatic colorectal cancer. Critique Data from randomized clinical trials have shown that second-line chemotherapy regimens using drugs with established activity in colorectal cancer are superior to best supportive care in patients with good performance status whose cancer has progressed on chemotherapy. Because this patient’s performance status is good, and she wants to continue receiving therapy, switching to a second-line chemotherapy regimen is reasonable, preferably in the context of a clinical trial. Oxaliplatin has recently been approved for use in combination with infusional 5-fluorouracil and leucovorin in patients whose colorectal cancer has progressed while they were taking

Page 211: MKSAP13-Hematology and Oncology

irinotecan given with 5-fluorouracil and leucovorin. Prospective studies have not shown metastatectomy to be beneficial for patients with more than three liver lesions, nor in patients with metastases to liver and lungs. There is no established role for marrow-ablative high-dose chemotherapy regimens in the treatment of patients with metastatic colorectal cancer.

Hematology and Oncology:Question 83 The correct answer is D Educational Objectives Diagnose and treat gastric mucosa-associated lymphoid tissue (MALT) lymphoma. Critique The first course of therapy should be eradication of the Helicobacter pylon organisms. In about half the cases of gastric MALT lymphoma associated with H. pylon infection, eradication of the infection induces a permanent complete remission. In half of these patients, the proliferation of the tumor has become dissociated from the presence of the H. pylon antigens. If the tumor does not regress within a year after eradication of the organism and the absence of the organism is confirmed and persistent, chemotherapy can be used effectively to treat the disease. Tumor progression after H. pylon eradication could indicate acceleration in its

Page 212: MKSAP13-Hematology and Oncology

natural history and transformation to diffuse large B-cell lymphoma. Total gastrectomy is not indicated in the management of gastric lymphoma. The main role of surgery in this patient would be diagnostic. The fluorouracil, doxorubicin, mitomycin (FAM) regimen has been used in patients with gastric carcinomas but is not used in lymphoma. Cyclophosphamide, hydroxydaunomycin, vincristine, and prednisone (CHOP) is effective therapy for some lymphomas, and the treatment of choice for localized nodal or extranodal diffuse large B-cell lymphomas is four cycles of CHOP plus involved-field radiation therapy. CHOP is generally not considered the first choice of therapy in the setting of gastric MALT lymphoma, however, because this form of lymphoma appears to be strongly influenced by antigens that are components of H. pylon. Hematology and Oncology:Question 84 The correct answer is C Educational Objectives Treat breast cancer with adjuvant radiation and systemic therapy. Critique Overall mortality from breast cancer has been declining over the past 10 years in the Western world, due in part to widespread application of adjuvant systemic therapy. However, these

Page 213: MKSAP13-Hematology and Oncology

therapies have toxicities and therefore must be applied judiciously. Chest wall radiation has been shown to decrease both local recurrences and systemic recurrences by 65% and 25%, respectively, of whatever they were destined to be. If this woman had a larger tumor or many positive lymph nodes, chest wall radiation would be appropriate. However, given her excellent prognosis, the benefits of chest wall radiation, like those of chemotherapy, are outweighed by the risks. Tamoxifen is the drug of choice in this situation. Aromatase inhibitors, like anastrozole, may be slightly more effective, but they appear to increase the risk of osteoporosis and bone fractures, and long-term side effects are not known. In a patient with such a favorable prognosis, these unknown risks are not worth what must be a very small incremental benefit for anastrozole. A recently reported prospective randomized trial has suggested that the combination of tamoxifen and anastrozole is no better than tamoxifen alone, and it may be worse than anastrozole alone. This patient has a very good prognosis. The chances that her disease will recur and that she will die over the next 10 years are no more than 10%. Although chemotherapy reduces

Page 214: MKSAP13-Hematology and Oncology

these odds, the proportional reduction in recurrence for a woman in her 60s is no more than 15%. Most guidelines panels, patients, and their physicians do not believe this small benefit outweighs the risks.

Hematology and Oncology:Question 85 The correct answer is E Educational Objectives Recognize the limitations of palliative chemotherapy in metastatic non-small-cell lung cancer Critique The patient’s performance status would be the limiting factor. Palliative chemotherapy offers modest prolongation of overall survival (8 to 16 weeks), but only in patients with adequate performance status. Patients in bed more than 50% of the day derive no benefit from palliative chemotherapy, and most large trials even suggest decreased survival for these patients with poor performance status who are exposed to cytotoxic agents. Patients whose status is adequate can be offered chemotherapy, and the choice of drugs can be modulated by factors such as jaundice. Hypercalcemia can be treated with bisphosphonates (intravenous pamidronate or zoledronate) concomitant with chemotherapy.

Page 215: MKSAP13-Hematology and Oncology

Hematology and Oncology:Question 86 The correct answer is C Educational Objectives Recommend interventions of proven benefit in decreasing the risk of lung cancer death in smokers. Critique The antidepressant bupropion has been shown to double the smoking cessation rate at 1 year in a placebo-controlled trial of cigarette smokers. Because smoking cessation is the only proven way to decrease smokers’ risk of dying of lung cancer, prescribing bupropion is the best option for this patient. Potential strategies to decrease the risk of lung cancer death in a patient who is having difficulty stopping smoking include more intensified smoking cessation efforts, medication to limit the cellular or DNA damage of the tobacco carcinogens, or screening to detect and cure small tumors. A number of studies have been undertaken. In placebo-controlled randomized clinical trials of 3-carotene to prevent lung cancer in smokers, the incidence and mortality rate of lung cancer were increased in the 13-carotene study groups. Therefore, on the basis of current evidence, 13-carotene supplementation is probably contraindicated for the prevention of lung cancer in smokers.

Page 216: MKSAP13-Hematology and Oncology

Although isotretinoin was shown to decrease the incidence of second primary tumors in patients treated for a primary squamous cell cancer of the head and neck, it did not improve the overall survival rate. In a study of patients who had been treated for early-stage non-small-cell lung cancer, those randomized to receive isotretinoin did no better than those randomized to receive placebo. Furthermore, there was a trend toward increased lung cancer recurrence and lung cancer death in subjects who were smokers at study baseline. In randomized trials, sputum cytology has not been shown to decrease the rate of death from lung cancer in smokers. In the Mayo Lung Project, monitoring of cigarette smokers by sputum cytology plus chest radiography every 4 months for 6 years was compared with usual care as a control arm. Although more surgically resectable cancers were detected in the screening arm, there was no decrease in the number of late-stage lung cancers. Furthermore, there was no decrease in lung cancer mortality in the screened arm. Spiral CT has been shown to detect early-stage lung cancers, but the false-positive rate is very high, and the effect on lung cancer mortality is unknown. Several randomized trials are in progress to test the net benefits and harms.

Page 217: MKSAP13-Hematology and Oncology

Hematology and Oncology:Question 87 The correct answer is D Educational Objectives Recognize the presenting features of hyponatremia of malignancy and the association with small-cell lung cancer. Critique This patient with mild hyponatremia is newly diagnosed with small-cell lung cancer. He has no symptoms that are clearly attributed to the hyponatremia (the fatigue may also be due to the underlying cancer), and therefore initiating nonspecific therapy such as fluid restriction with or without demeclocycline therapy is not warranted. The most important therapeutic intervention is to treat the underlying cancer. Combination chemotherapy would be the appropriate treatment. Because he has an established diagnosis of small-cell lung cancer, confirming that the laboratory studies meet the criteria for syndrome of inappropriate diuretic hormone secretion, ruling out other causes such as adrenal insufficiency and discontinuing drugs that might possibly lead to free water retention (such as thiazide diuretics) is for academic rather than practical reasons.

Page 218: MKSAP13-Hematology and Oncology

Hematology and Oncology:Question 88 The correct answer is D Educational Objectives Initiate treatment of testicular carcinoma. Critique This patient has testicular carcinoma as manifested by a mass and abnormal markers. He cannot have pure seminoma, because both his serum ct-fetoprotein and β -human chorionic gonadotropin levels are elevated. The next appropriate step would be to perform an inguinal orchiectomy. Retroperitoneal lymph node dissection or combination chemotherapy might be appropriate after a diagnosis is made and clinical staging is complete. Pelvic irradiation is not indicated in the management of patients with newly diagnosed nonseminomatous germ cell tumors.

Hematology and Oncology:Question 89 The correct answer is B Educational Objectives Recognize the most important risk factor for ovarian cancer. Critique This patient should be given standard medical care, including an annual rectovaginal pelvic

Page 219: MKSAP13-Hematology and Oncology

examination. Because this patients family history gives no evidence of ovarian cancer and because she is young, she should be reassured. Familial cancer syndromes account for only 5% to 10% of all cases. Apart from age, ovarian cancer in a first-degree relative is the most important risk factor for ovarian cancer. There is no established role for ovarian cancer screening. There is no need to screen for BRCA-1 and BRCA-2 because the family history shows no evidence of this genetic mutation.

Hematology and Oncology:Question 90 The correct answer is A Educational Objectives Improve the quality of life for breast cancer survivors. Critique Recently reported pilot and prospective randomized clinical trials have demonstrated that selective serotonin reuptake inhibitors, in particular fluoxetine, venlafaxine, and paroxetine, reduce the incidence and severity of hot flushes by more than half in 50% to 75% of patients, compared with similar reductions in only 20% to 30% of patients taking placebo. Over-the-counter, nonhormonal, nongreasy vaginal preparations (such as Replens® and Astroglide®) are frequently, although not always, helpful in facilitating intercourse.

Page 220: MKSAP13-Hematology and Oncology

Postmenopausal symptoms are quite common in women taking tamoxifen after chemotherapy. Atrophic vaginitis and dyspareunia as a consequence of menopause can be so severe that women sometimes discontinue taking tamoxifen, even though it provides clear-cut benefits in regard to preventing recurrence of incurable cancer. Sympathetic counseling is appropriate but not sufficient. Symptoms may lessen or resolve with time but sometimes they persist throughout treatment with tamoxifen. Although estrogen is the most effective treatment for these symptoms, it is now established that estrogen replacement therapy with progesterone modestly increases the risk of a new primary breast cancer. To date, no prospective randomized trials have demonstrated that estrogen replacement therapy increases the risk of breast cancer recurrence or death, but the most successful therapies in estrogen receptor-positive breast cancers are anti-estrogenic (selective estrogen receptor modulaters and aromatase inhibitors). Therefore, most oncologists are unwilling to prescribe estrogens, and patients with breast cancer are reluctant to take them unless nonhormonal therapies are ineffective. Tamoxifen has estrogenic agonist properties in the uterus, and, like unopposed estrogen,

Page 221: MKSAP13-Hematology and Oncology

increases the risk of uterine cancer two- to threefold. This modest relative risk translates to only a small absolute risk, and routine uterine screening is therefore not recommended. Although symptoms such as pelvic discomfort, pain, or vaginal bleeding should be thoroughly evaluated, this patients dyspareunia is a symptom of vaginal estrogen depletion and does not suggest endometrial cancer.

Hematology and Oncology:Question 91 The correct answer is B Educational Objectives Understand the role of screening for prostate cancer Critique The incidence of prostate cancer increased dramatically when PSA screening began to be used. At this time, it is unclear whether screening will lead to a change in prostate cancer mortality or overall mortality rates. It is hoped that the Prostate, Colorectal, Lung and Ovary randomized cancer screening trial will answer that question. This patient should be counseled that prostate cancer can be detected earlier with screening, but it is not known whether earlier treatment would improve his longevity. In addition, he should be advised of the significant morbidity that could accompany the treatment of early-stage

Page 222: MKSAP13-Hematology and Oncology

prostate cancer. Screening does lead to the earlier detection of prostate cancer. Most patients are asymptomatic; therefore a lack of symptoms does not exclude the possibility of an elevated PSA. The digital rectal examination is not as sensitive as a PSA screen. There are, however, tumors that will be detected on digital rectal examination in patients with a PSA lower than 4.0 ng/mL.

Hematology and Oncology:Question 92 The correct answer is A Educational Objectives Order appropriate further treatment for patients with small-cell lung cancer who achieve clinical complete remission. Critique Patients with small-cell lung cancer of limited or extensive stage should be offered prophylactic cranial irradiation. A meta-analysis of 7 randomized trials evaluating this therapy in patients who achieve a complete response demonstrated 15% to 21% improvement in disease-free and overall survival at 3 years. Additional chemotherapy, even with other agents, has not been shown to benefit complete responders with limited-stage disease. Surgery is rarely considered for small-cell lung cancer because micrometastatic disease is almost always present, and response

Page 223: MKSAP13-Hematology and Oncology

to chemotherapy is the key to survival.

Hematology and Oncology:Question 93 The correct answer is D Educational Objectives Recognize when palliative resection of the primary tumor should be considered in a patient who presents with metastatic colon cancer. Critique Systemic chemotherapy should be offered to this patient because he has symptoms related to his cancer and his performance status is still reasonably good. Clinical trials have shown that immediate institution of systemic 5-fluorouracil-based therapy is superior to best supportive care in the treatment of metastatic colorectal cancer, in terms of time to disease progression, time to symptom progression, and overall survival. The indications for palliative resection of the primary tumor in a patient with metastatic disease are obstruction (existing or imminent) and bleeding. The patient has no symptoms of obstruction and has only mild anemia. Therefore, resection of the primary tumor would not affect his overall survival, would subject him to the risks of surgery, and would delay the initiation of systemic therapy. Regional chemotherapy to the liver is not indicated because the patient’s disease is not

Page 224: MKSAP13-Hematology and Oncology

confined to the liver. There is no evidence to suggest that combined regional and systemic chemotherapy is beneficial in patients with metastatic disease involving the liver and extrahepatic sites. Because systemic chemotherapy is not curative, best supportive care can be considered if the patient chooses not to receive chemotherapy.

Hematology and Oncology:Question 94 The correct answer is C Educational Objectives Recognize the risk factors for metastasis in patients with a biochemical failure after surgery for prostate cancer. Critique The natural history of a rise in PSA levels after radical prostatectomy is poorly understood. The best series (Pound et al.) found that patients with a recurrent rise in PSA level within 2 years of surgery, with a high Gleason score and a rapid PSA doubling time, were at the greatest risk for the development of metastatic disease. This patient, with a late, slow rise in his PSA level, is at low risk for the rapid development of metastatic disease. There is no evidence that patients with recurrent disease treated with a prostatectomy have an outcome different from patients treated with other primary therapies. Neither

Page 225: MKSAP13-Hematology and Oncology

the age of the patient nor the PSA nadir has been shown to affect the length of time to metastatic disease in this population.

Hematology and Oncology:Question 95 The correct answer is E Educational Objectives Understand the role of adjuvant therapy in completely resected non-small-cell lung cancer. Critique A 50% to 70% cure rate with surgery alone would be predicted for early-stage disease. Randomized trials have failed to show any survival advantage as yet for any adjuvant chemotherapy. Meta-analyses confirm that additional radiation therapy confers no benefit in this setting and in fact may be deleterious to survival in early-stage disease. The fact that 30% to 50% of patients like this one will develop metastatic disease within a few years makes them good candidates for clinical trials involving novel therapies (such as immunomodulators or vaccines), with or without standard chemotherapy.

Page 226: MKSAP13-Hematology and Oncology

Hematology and Oncology:Question 96 The correct answer is A Educational Objectives Treat advanced ovarian cancer. Critique Randomized controlled trials have established that a combination of either cisplatin or carboplatin with paclitaxel is the standard of care for the treatment of advanced ovarian cancer (stage III/IV). Docetaxel is another taxane, but to date has not been shown to offer an advantage over paclitaxel in first-line therapy of advanced ovarian cancer. The addition of intraperitoneal chemotherapy may be beneficial in patients with optimally debulked stage III disease (no tumor larger than 1 cm) after completion of systemic chemotherapy, but this approach requires second-look laparotomy to confirm miminal residual disease and is not recommended if the patient has adhesions or the treatment center is not experienced with regional therapy. More aggressive approaches should be recommended only in the context of a clinical trial.

Hematology and Oncology:Question 97 The correct answer is B Educational Objectives Know the effects of tamoxifen, other than reduction of breast cancer risk, when used in

Page 227: MKSAP13-Hematology and Oncology

the cancer prevention setting. Critique The Breast Cancer Prevention Trial (BCPT) compared 5 years of tamoxifen use versus placebo in women at elevated risk for breast cancer. An important life-threatening adverse outcome associated with tamoxifen was an increased risk of deep venous thrombosis or pulmonary embolism. The rate of pulmonary embolism was 0.69 per 1000 women in the tamoxifen arm versus 0.23 per 1000 in the placebo arm (risk ratio of 3.01, 95% CI 1.15 to 9.27). The rates of deep venous thrombosis were 1.34 versus 0.84 versus 0.84 per 1000, respectively (RR 1.6, 95% CI 0.91 to 2.86). Although tamoxifen decreases plasma cholesterol levels, it has not been shown to have a statistically significant effect on ischemic heart disease rates in women with or without a history of heart disease. Changes in serum lipid levels appear to be a poor surrogate for the impact of hormonal manipulations on ischemic heart disease. For example, hormone replacement therapy (HRT) with estrogen plus progestin has recently been shown to increase the risk of coronary artery disease in healthy postmenopausal women despite the fact that HRT lowers serum lipid levels. The BCPT showed no increase in the incidence

Page 228: MKSAP13-Hematology and Oncology

of any cancer other than uterine endometrial cancer, with a risk ratio of about 2.5. This patient is, however, presumably not at risk for endometrial cancer, given her prior hysterectomy. There have been anecdotal reports of depression and mood disorders in association with tamoxifen. The BCPT found no difference in the incidence of depression between patients taking tamoxifen or placebo, irrespective of underlying risk of depression (on the basis of medical history obtained at study baseline).

Hematology and Oncology:Question 98 The correct answer is B Educational Objectives Treat adenocarcinoma presenting with diffuse peritoneal involvement in a woman. Critique When a woman presents with adenocarcinoma and diffuse peritoneal involvement, therapy should be directed toward ovarian cancer, even if no ovarian mass is evident. Patients with

Page 229: MKSAP13-Hematology and Oncology

ovarian cancer can have long-term disease-free survival at 10 years, even with this presentation. Management involves debulking surgery, followed by combination chemotherapy. Ovarian cancer rarely spreads to bone, and a bone scan would not be considered unless symptoms of bone involvement were evident. There is no established role for positron emission tomography scan in this setting, and a false-positive scan should not preclude a curative approach with surgery and chemotherapy. Radiation therapy plays no role in this setting. Although combination chemotherapy with a taxane and a platinum agent is an important treatment for this woman, the standard of care mandates debulking surgery first.

Hematology and Oncology:Question 99 The correct answer is D Educational Objectives Understand which patients with newly diagnosed breast cancer should have mastectomy vs. breast-conserving therapy and the therapy involved in the latter. Critique Breast-conserving (or preserving’) therapy is as effective as and cosmetically more acceptable than mastectomy. There are, however, some relative contraindications, mostly related to an

Page 230: MKSAP13-Hematology and Oncology

unacceptably high risk of subsequent recurrence of breast cancer (>15% over 5 to 10 years). They include the presence of grossly unresected breast cancer, relatively large lesions, and relatively small breasts. The presence of widespread ductal carcinoma in situ is also a high risk factor for recurrence in the breast. Several randomized trials have demonstrated that even with systemic therapy, recurrence of breast cancer is quite high in women who do not receive radiation therapy. In most studies, the risk of in-breast recurrence for patients who receive radiation therapy is less than 10%, whereas that for lumpectomy alone exceeds 40%. Therefore, because most recurrences in the breast are treated by mastectomy, an initial course of radiation therapy will result in fewer mastectomies in the long run. Patients who have previously had mediastinal or ipsilateral chest radiation (for example, women with a history of Hodgkin’s disease) and patients at high risk of toxicity from radiation (for example, women with lupus dermatitis) are also poor candidates for breast-conserving therapy. This patient appears to be an ideal candidate for breast-conserving therapy, which would be completed with radiation therapy. Because this patient has positive axillary lymph

Page 231: MKSAP13-Hematology and Oncology

nodes, there is no role for tamoxifen; she is unlikely to benefit, given her negative hormone receptors.

Hematology and Oncology:Question 100 The correct answer is E Educational Objectives Treat lymphadenopathy in a young smoker. Critique A chest radiograph is indicated because of the dyspnea on exertion and the night sweats. The age of the patient tends to favor observation as the standard approach, because lymphadenopathy in young people generally has a benign cause. The supraclavicular location, the size of the node (greater than 2 cm in diameter), and the presence of systemic symptoms, however, heighten the suspicion of more serious illness. Watching and waiting would not be appropriate under these circumstances. Referral to an otolaryngologist might be indicated because smoking-related head and neck cancers can present with

Page 232: MKSAP13-Hematology and Oncology

lymphadenopathy in the absence of an obvious mucosal lesion. The patient’s age, however, and the fact that she abstains from alcohol make head and neck cancer unlikely. There is no proven role for empiric use of antibiotics in the management of a patient with unexplained lymphadenopathy. Needle aspiration is not the diagnostic procedure of choice in a patient with undiagnosed lymphadenopathy because it fails to obtain adequate tissue for histologic, immunologic, and genetic analysis.

Hematology and Oncology:Question 101 The correct answer is A Educational Objectives Recognize the various nonspecific manifestations of multiple myeloma Critique The patient is older than 40 years, has unexplained anemia, renal dysfunction, and recurrent infections: This scenario is common in patients with multiple myeloma. In addition, his total protein level is increased in the face of a normal albumin level, meaning that the gamma globulin fraction is increased. Serum protein electrophoresis would document the presence of a monoclonal gammopathy and he would need to be staged with a bone marrow evaluation (to quantitate marrow plasma cells)

Page 233: MKSAP13-Hematology and Oncology

and skeletal survey (to search for lytic bone lesions). It would also be necessary to measure his serum 32 -microglobulin level. Anemia is not a common feature of chronic pyelonephritis until renal failure develops. Iron deficiency anemia is most often noted in the setting of chronic bleeding, of which this patient has no history or symptoms. When cancer of the colon is associated with anemia, the cause is chronic blood loss and the mechanism is iron deficiency, which would produce microcytic, hypochromic anemia. Small-cell lung cancer could produce similar generalized symptoms but is rarely associated with hypercalcemia and is not associated with recurrent urinary tract infection. The diagnosis of myeloma should be considered in any patient older than 40 years with unexplained anemia, renal failure, bone lesions, or recurrent infections.

Hematology and Oncology:Question 102 The correct answer is C

Page 234: MKSAP13-Hematology and Oncology

Educational Objectives Recognize the indications for surgical therapy after diagnostic testing for non-small-cell lung cancer. Critique Surgical therapy is offered in two situations for patients with non-small-cell lung cancer and obvious metastasis. First, surgery is considered if there is reasonable curative intent. In the four options presented, curative intent is most unlikely, especially because microscopic metastases are almost always present when an overt metastasis is demonstrated on chest radiograph. Second, surgical therapy is considered in selected situations of isolated single metastasis. A randomized study showed a 1-year survival advantage for patients with a single metastasis to the brain treated with surgery and radiation therapy compared with radiation therapy alone. A finding of bone metastasis would be best treated by palliative radiation therapy or chemotherapy or both. A finding of liver metastasis or cytologically positive pleural effusion would be best treated by palliative chemotherapy. Surgical therapy might also be called for to relieve an epidural compression if radiation therapy is contraindicated or if a second lung

Page 235: MKSAP13-Hematology and Oncology

nodule is found that is suspected of being cancerous. In the latter case, resection of both lesions might even offer the possibility of cure, especially if the second lesion is not malignant.

Hematology and Oncology:Question 103 The correct answer is D Educational Objectives Treat metastatic breast cancer. Critique A recently reported prospective randomized clinical trial in patients with metastatic breast cancer has demonstrated that the combination of trastuzumab and paclitaxel are at least additive, if not synergistic, in women with HER2-positive disease. Patients treated with this combination had significantly longer survival than those treated initially with chemotherapy alone. Few if any patients are cured of metastatic breast cancer. For many patients, however, systemic therapy probably does result in modest survival prolongation of several months to a few years compared with best supportive care, and the palliative benefits are well recognized. Therefore, this patient should receive treatment. The key to effective palliation is balancing the potential benefits with the expected side effects of specific therapies.

Page 236: MKSAP13-Hematology and Oncology

In this regard, endocrine therapy is almost always less toxic than chemotherapy. The steroid hormone receptors in her tumor, however, have been consistently negative. Therefore, she is very unlikely (less than 10%) to respond to endocrine therapy of any sort. Given that she has symptomatic visceral disease, she should be treated with therapy that has a higher chance of working, even though she will suffer more side effects from it.

Preclinical studies and some preliminary clinical data suggest that aromatase inhibitors are not likely to be effective and may be dangerous in premenopausal women.

Hematology and Oncology:Question 104 The correct answer is C Educational Objectives Recognize the clinical significance of the Gleason score in patients with newly diagnosed prostate cancer. Critique This patients Gleason score indicates that he would have a high risk of recurrence after prostatectomy. The Gleason score is obtained by grading from 1 to 5 the two predominant

Page 237: MKSAP13-Hematology and Oncology

histologic types of adenocarcinoma of the prostate and adding the grades. This score is an extremely important prognostic factor in predicting the biology of clinically localized prostate cancer. The scores range from 2 to 10, with the highest scores predictive of patients who will not have organ-confined disease. This patient with a high Gleason score (despite the fact that his examination reveals no abnormalities and his PSA is low) would have only about a 37% probability of having organ-confined disease. There is some variability among pathologists when scoring prostate cancer using this system. The concordance rate is high enough, however, that the Gleason score can be used as a prognostic variable. The Gleason score is not influenced by the absence of symptoms in a patient.

Hematology and OnCology:Question 105 The correct answer is A Educational Objectives Provide supportive care in the management of pancreatic cancer. Critique This patient almost certainly has metastatic pancreatic cancer. The median time to disease progression of patients with advanced pancreatic cancer-related symptoms is only a few months, even with the best available

Page 238: MKSAP13-Hematology and Oncology

chemotherapy. This patient is not a good candidate for chemotherapy; therefore, best supportive care and referral to a hospice is the best recommendation. Since the patient’s upper abdominal discomfort is vague and mild, a trial of narcotic analgesics is warranted. Even if the patients clinical status improves slightly as he recovers from the pulmonary embolism, his baseline performance status is poor because of his other medical conditions and cachexia caused by the cancer. As such, he is unlikely to benefit from palliative chemotherapy. Palliative radiation therapy is indicated only for the relief of local symptoms. Diagnostic biopsy is not needed because it would not influence the current management.

Hematology and Oncology:Question 106 The correct answer is B Educational Objectives Recognize the potential toxicities of androgen suppression. Critique It is important for this patient to realize that with long-term androgen ablation he will be at risk for osteopenia and potential nonpathologic fractures. The use of PSA screening has led to the earlier detection of prostate cancer recurrence after definitive local therapy. Many patients are asymptomatic and have no clinical manifestations of cancer except for an

Page 239: MKSAP13-Hematology and Oncology

abnormal PSA level. There is no standard therapy for these patients. Although there is no evidence that early hormone therapy can affect survival in this group, many patients will commence long-term androgen ablation. Leuprolide treatment should not impair urinary flow in this asymptomatic patient. Weight gain and loss of muscle mass can occur with testosterone suppression. In addition, many patients note loss of hair on their arms and legs with leuprolide therapy. Hematology and Oncology:Question 107 The correct answer is D Educational Objectives Treat limited-stage small-cell lung cancer. Critique Clinical staging for small-cell lung cancer classifies disease as limited-stage (unilateral lung and mediastinal involvement) or extensive-stage (bone, liver, brain, or bilateral lung metastases). Approximately 25% of patients with limited-stage small-cell lung cancer will be free of disease 5 years after combined-modality therapy. A meta-analysis of randomized trials documents that combined-modality therapy is the preferred option, with chemotherapy and radiation therapy administered concomitantly at the start of treatment. Surgery is not indicated because of the systemic nature of small-cell lung cancer.

Page 240: MKSAP13-Hematology and Oncology

Hematology and Oncology:Question 108 The correct answer is C Educational Objectives Treat symptomatic hypercalcemia of malignancy. Critique The patient has hypercalcemia, the most common metabolic complication of malignancy. She is profoundly dehydrated. Despite her history of congestive heart failure, the most appropriate initial therapy is to rehydrate as vigorously as possible with normal saline. Furosemide must not be given until the patient has first been adequately rehydrated. Administration of a bisphosphonate is not an appropriate initial therapeutic intervention in this patient because of her volume depletion; in addition, it would slow the necessary decrease in her serum calcium level. Although corticosteroids can be considered as an adjunctive therapy in a patient with a potentially hormone-responsive tumor such as breast cancer, the top priority in this seriously dehydrated patient is volume repletion.

Hematology and Oncology:Question 109 The correct answer is C Educational Objectives Select the best test to evaluate the histology of carcinoma of unknown primary site.

Page 241: MKSAP13-Hematology and Oncology

Critique When a poorly differentiated cancer is negative for leukocyte common antigen and cytokeratin, the most likely diagnosis is melanoma (which can be amelanotic) or sarcoma. An inguinal lymph node would be an expected site for melanoma but very unusual for sarcoma. The best test for establishing melanoma is a stain for S-100, although other immunoperoxidase stains can be employed (melan A, vimentin). If positive, an exhaustive search of the skin should be undertaken to locate the primary tumor, focusing on sites that drain to the right inguinal region. Melanoma can be further confirmed with electron microscopy. Sometimes the primary site may not be evident, even when the tumor is known to be a melanoma. In general, primary sites might include the eye or even mucosal areas that can receive sun exposure.

Hematology and Oncology:Question 110 The correct answer is B Educational Objectives Recognize the presentation of superior vena cava syndrome and identify the most useful

Page 242: MKSAP13-Hematology and Oncology

initial diagnostic test. Critique The presentation is consistent with obstruction of the superior vena cava (SVC), not pulmonary embolism. Because the patient does not have any life-threatening symptoms such as upper airway obstruction, he should undergo diagnostic procedures to confirm the cause and guide the most appropriate therapy. A contrast-enhanced CT is the most useful test to diagnose SVC syndrome and to detect the presence of a mediastinal mass; it may also provide an opportunity for CT-guided needle biopsy. CT findings indicating SVC obstruction may include intraluminal filling defects of the SVC, decreased opacification of the innominate vein or SVC below the level of obstruction, opacification of collateral vessels, and external compression of mediastinal venous channels. Initiating both pulmonary and thoracic surgery consultations early on is prudent to facilitate the work-up and obtain a tissue diagnosis. Administration of radiation therapy while the diagnostic work-up is occurring is not necessary because the patient’s symptoms do not require emergency intervention. A venous dye study would not add to the findings of the CT. Mediastinoscopy with biopsy is more invasive than necessary for this diagnosis. The clinical picture does not suggest pulmonary

Page 243: MKSAP13-Hematology and Oncology

embolism; ventilation/perfusion scan is not useful to establish a diagnosis in SVC syndrome

Hematology and Oncology:Question 111 The correct answer is B Educational Objectives Treat metastatic testicular carcinoma with chemotherapy. Critique Testicular carcinoma is the rare metastatic solid tumor that can be routinely cured with combination chemotherapy such as bleomycin, etoposide, and cisplatin. The introduction of cisplatin approximately 25 years ago has resulted in effective therapy, even for patients with advanced disease. Therefore, this patient, whose tumor has recurred in the lungs (a common location after retroperitoneal lymph node dissection), should be treated with curative intent. Surgical resection of lung lesions is sometimes appropriate in testicular cancer, generally after treatment with chemotherapy if there are remaining nodules. Teratoma is not chemoresponsive, and, after chemotherapy, residual tumor of this histologic type may need to be resected. Radiation is sometimes used for palliation, but chemotherapy would be the appropriate choice for multiple asymptomatic lesions. There is no advantage to watchful

Page 244: MKSAP13-Hematology and Oncology

waiting in a potentially curable patient.

Hematology and Oncology:Question 112 The correct answer is D Educational Objectives Recommend appropriate cancer risk assessment and testing. Critique Her family should be reassured that they are not at particularly high risk for cancer, especially the younger ones, and both she and her family should be encouraged to follow routine screening and risk reduction guidelines.

Although several members of this patients family have had cancer, in each case the cancer was one that occurs commonly in the age groups in which these individuals were diagnosed. Hereditary genetic cancer syndromes are usually associated with cancers in patients younger than 60 years, and they often result in multiple cancers in the same person. Furthermore, there is no specific pattern of organ-oriented cancers, as one might see in families that harbor BRCA-1 or BRCA-2 (breast and ovarian cancers). Even though there are two instances of breast cancer in the family, both occurred in elderly women. Furthermore, the patients ancestry is not one in which hereditary genetic cancer syndromes are more prevalent.

Page 245: MKSAP13-Hematology and Oncology

Taken together, these data suggest that the cancers observed in this family are sporadic and it is very unlikely that her family harbors a germ-line abnormality in any of the currently identified tumor suppressor genes. Testing is very expensive and can be associated with occasional false-positive results, which could have devastating consequences for her or members of her family.

Hematology and Oncology:Question 113 The correct answer is C Educational Objectives Apply the principles of supportive care in multiple myeloma. Critique The only one of the options that is contraindicated is intravenous gamma globulin. Patients with multiple myeloma have hypogammaglobulinemia when the paraprotein is not considered and are susceptible to infections. Intravenous gamma globulin, however, has not been shown to reduce the incidence of serious infections when used prophylactically. Monthly intravenous gamma globulin is not routinely administered to patients with myeloma. However, it can be used in patients who have recurrent episodes of bacterial infections or sepsis that are life-threatening. The supportive care of the patient with

Page 246: MKSAP13-Hematology and Oncology

myeloma is as important as the anticancer treatment to the patients quality of life. Erythropoietin has been shown to alleviate the anemia and improve quality of life. Radiation therapy may be necessary to control localized, symptomatic bony destructive lesions. Pain management is an important part of the care of many cancer patients. The goal of total relief of pain should not be sacrificed to fears of causing narcotic addiction in this population. Bisphosphonates such as pamidronate or zoledronate are effective agents at minimizing bony destruction and bone fractures. They also control hypercalcemia. Evidence is also emerging for a role for bisphosphonates in killing tumor cells, and randomized studies have found that bisphosphonate therapy improves patients’ length and quality of life.